Download as pdf or txt
Download as pdf or txt
You are on page 1of 165

PART–A : PHYSICS

1. A man grows into a giant such that his linear


dimensions increase by a factor of 9. Assuming that v
his density remains same, the stress in the leg will
change by a factor of t
(3)
1
(1) 9 (2)
9

1
(3) 81 (4) v
81
Answer (1)
(4) t
vf
Sol.  93
vi
Answer (3)
∵ Density remains same
Sol. Acceleration is constant and negative
So, mass  Volume
v
mf
 93
mi
t
(Area)f
 92
(Area)i

(Mass)  g
Stress = 3. A body of mass m = 10–2 kg is moving in a medium
Area
and experiences a frictional force F = –kv2. Its initial
2 ⎛ mf ⎞ ⎛ Ai ⎞ speed is v0 = 10 ms–1. If, after 10 s, its energy is

1 ⎜⎝ mi ⎟⎠ ⎜⎝ Af ⎟⎠ 1
mv 02 , the value of k will be
8
93
 9 (1) 10–3 kg m–1 (2) 10–3 kg s–1
92
(3) 10–4 kg m–1 (4) 10–1 kg m–1 s–1
2. A body is thrown vertically upwards. Which one of
Answer (3)
the following graphs correctly represent the velocity
vs time? 1
kf mv 02
8 1
Sol.  
ki 1 4
v mv 02
2
vf 1

(1) vi 2

v0
t vf 
2

v mdv
 kv 2 
dt
v0
(2) t t0
2
dv  kdt
∫ v2 ∫ 
m
v0 0

2
www.vedantu.com
v0
⎡ 1⎤ 2 k
⎢  v ⎥  m t0
⎣ ⎦v 0 Sol. 2R
l
1 2 k
   t0
v0 v0 m
mR 2 m2
I 
1 k 4 12
   t0
v0 m
m ⎡ 2 2 ⎤
I ⎢R  ⎥
m 4 ⎣⎢ 3 ⎥⎦
k
v 0 t0
m ⎡ v 2 ⎤
 ⎢  ⎥
102 4 ⎢⎣  3 ⎥⎦

10  10
dI m ⎡ v 2 ⎤
= 10–4 kg m–1   0
d  4 ⎢⎣ 2 3 ⎥⎦
4. A time dependent force F = 6t acts on a particle of
mass 1 kg. If the particle starts from rest, the work v 2
2

done by the force during the first 1 second will be  3

(1) 4.5 J (2) 22 J 23


v
(3) 9 J (4) 18 J 3

Answer (1) 23


R 2  
dv 3
Sol. 6t  1
dt
2 3

v R2 2
∫ dv  ∫ 6t dt
0  3

1
R 2
⎡t2 ⎤
v  6⎢ ⎥ 6. A slender uniform rod of mass M and length l is
⎣2⎦ 0 pivoted at one end so that it can rotate in a vertical
= 3 ms–1 plane (see figure). There is negligible friction at the
pivot. The free end is held vertically above the pivot
1 and then released. The angular acceleration of the
W = KE   1  9  4.5 J
2 rod when it makes an angle  with the vertical is
5. The moment of inertia of a uniform cylinder of length z
 and radius R about its perpendicular bisector is I.

What is the ratio such that the moment of inertia
R
is minimum?

3 3
(1) (2) x
2 2
3g 2g
(1) sin  (2) sin 
3 2 3
(3) 1 (4)
2
3g 2g
(3) cos  (4) cos 
Answer (1) 2 3

3
www.vedantu.com
Answer (1) Answer (4)
Sol. Torque at angle 
g

  Mg sin  
2
Sol. gs

,
d
(M, l) O d=R


Mg Variation of g inside earth surface

Gm
d Rg  d
= I R2

 M 2 Gm
I = Mg sin   I  d  R  gs 
2 3 R2

M 2  Gm
   Mg sin  d Rg 
3 2 d2
8. A copper ball of mass 100 gm is at a temperature T.
 sin 
g
3 2 It is dropped in a copper calorimeter of mass
100 gm, filled with 170 gm of water at room
3g sin  temperature. Subsequently, the temperature of the

2 system is found to be 75°C. T is given by :

7. The variation of acceleration due to gravity g with (Given : room temperature = 30°C, specific heat of
distance d from centre of the earth is best copper = 0.1 cal/gm°C)
represented by (R = Earth's radius) : (1) 800°C (2) 885°C
(3) 1250°C (4) 825°C
g
Answer (2)
(1) Sol. 100 × 0.1 × (t – 75) = 100 × 0.1 × 45 + 170 × 1 × 45
d
O 10t – 750 = 450 + 7650
10t = 1200 + 7650
g
10t = 8850
(2) d t = 885°C
O R 9. An external pressure P is applied on a cube at 0°C
so that it is equally compressed from all sides. K is
g the bulk modulus of the material of the cube and 
is its coefficient of linear expansion. Suppose we
(3) want to bring the cube to its original size by heating.
d The temperature should be raised by :
O R
P P
(1) (2)
g 3 K K

(4) d 3
(3) (4) 3PK
PK
O R
4
www.vedantu.com
Answer (1) Answer (4)
Sol. n1 = initial number of moles
P
Sol. K =
⎛ V ⎞ PV
5
10  30
⎜– V ⎟ n1 = 1 1
  1.24  10
3
⎝ ⎠ RT1 8.3  290

V P n2 = final number of moles


=
V K
5
 = 3 P2V2 10  30 3
 V = V0 (1 + t) =   1.20  10
RT2 8.3  300
V
V0 = t
Change of number of molecules :
nf – ni = (n2 – n1) × 6.023 × 1023
P P P
 = t  t = =  – 2.5 × 1025
K K 3 K
12. A particle is executing simple harmonic motion with
10. Cp and Cv are specific heats at constant pressure a time period T. At time t = 0, it is at its position of
and constant volume respectively. It is observed that equilibrium. The kinetic energy-time graph of the
Cp – Cv = a for hydrogen gas particle will look like :
Cp – Cv = b for nitrogen gas
The correct relation between a and b is :
(1) 0
1 T T T
(1) a  b (2) a = b 2
14
(3) a = 14b (4) a = 28b
Answer (3)
Sol. Let molar heat capacity at constant pressure = Xp (2) 0 T
and molar heat capacity at constant volume = Xv
Xp – Xv = R
MCp – MCv = R

R (3) 0
Cp – Cv = T T
M 2
R
For hydrogen; a =
2
R
For N2; b = (4)
28 0 T T T
4 2
a
= 14
b Answer (4)
a = 14b
1
11. The temperature of an open room of volume 30 m3 Sol. K.E  m2 A2 cos2 t
2
increases from 17°C to 27°C due to the sunshine.
The atmospheric pressure in the room remains
1 × 105 Pa. If ni and nf are the number of molecules
in the room before and after heating, then nf – ni will
be
(1) –1.61 × 1023 (2) 1.38 × 1023 T T
(3) 2.5 × 1025 (4) –2.5 × 1025 4 2

5
www.vedantu.com
13. An observer is moving with half the speed of light 
T2  ( p cos i  p sin j )  3E1j
towards a stationary microwave source emitting
waves at frequency 10 GHz. What is the frequency
of the microwave measured by the observer? (speed 
k  3 pE1 cos 
k …(ii)
of light = 3 × 108 ms–1) From (i) and (ii)
(1) 10.1 GHz (2) 12.1 GHz
pE sin   3 pE cos 
(3) 17.3 GHz (4) 15.3 GHz
Answer (3) tan   3
Sol. For relativistic motion
 = 60°
c v 15. A capacitance of 2 F is required in an electrical
f = f0 ; v = relative speed of approach
c v circuit across a potential difference of 1.0 kV. A large
number of 1 F capacitors are available which can
c withstand a potential difference of not more than
c
f = 10 2  10 3  17.3 GHz 300 V.
c
c The minimum number of capacitors required to
2
achieve this is

14. An electric dipole has a fixed dipole moment p , (1) 2 (2) 16
which makes angle  with respect to x-axis. When (3) 24 (4) 32

subjected to an electric field E 1  Ei , it experiences Answer (4)
 Sol. Following arrangement will do the needful :
a torque T 1  
k . When subjected to another electric
   8 capacitors of 1F in parallel with four such
field E 2  3E1j it experiences a torque T 2  T 1 . branches in series.
The angle  is
1 1 1 1
(1) 30° (2) 45° 2 2 2 2
3 3 3 3
(3) 60° (4) 90°
8 8 8 8
Answer (3)
Sol. y 1000 V

8 F 8 F 8 F 8 F

p 250 V 250 V 250 V 250 V


 x
1000 V
16. In the given circuit diagram when the current reaches
z
 steady state in the circuit, the charge on the
p  p cos i  p sin j capacitor of capacitance C will be :
 E r
E 1  Ei
r1
  
T 1  p  E1 C
r2

= ( p cos i  p sin j )  E i
r1
(1) CE (2) CE
(r2  r )

k  pE sin  
k   …(i)
r2 r1
 (3) CE (4) CE
E 2  3E1j (r  r2 ) (r1  r )

6
www.vedantu.com
Answer (3) Answer (1)
Sol. In steady state, flow of current through capacitor will
be zero. I
Sol. T = 2π
MB
E r

7.5  10 –6 2
i = 2π =  1.06
r1 6.7  10 –2
 0.01 10

C For 10 oscillations,
r2 t = 10T = 2π × 1.06

E = 6.6568 ≈ 6.65 s
i=
r  r2 19. When a current of 5 mA is passed through a
galvanometer having a coil of resistance 15 Ω, it
Er2C shows full scale deflection. The value of the
VC = i r2C = r  r resistance to be put in series with the galvanometer
2
to convert it into a voltmeter of range 0-10 V is
r2 (1) 1.985 × 103 Ω
VC = CE
r  r2
(2) 2.045 × 103 Ω
(3) 2.535 × 103 Ω
17. 2V 2V 2V
(4) 4.005 × 103 Ω
Answer (1)
1 1 1
Sol. ig = 5 × 10–3 A
G = 15 Ω
2V 2V 2V
Let series resistance be R.
In the above circuit the current in each resistance is V = ig (R + G)
(1) 1 A 10 = 5 × 10–3 (R + 15)
(2) 0.25 A R = 2000 – 15 = 1985 = 1.985 × 103 Ω
(3) 0.5 A 20. In a coil of resistance 100 Ω, a current is induced by
changing the magnetic flux through it as shown in
(4) 0 A
the figure. The magnitude of change in flux through
Answer (4) the coil is
Sol. The potential difference in each loop is zero.
10
∴ No current will flow.
Current
18. A magnetic needle of magnetic moment (amp.)
6.7 × 10–2 Am2 and moment of inertia 7.5 × 10–6 kg m2
is performing simple harmonic oscillations in a
magnetic field of 0.01 T. Time taken for 10 complete
0.5 s
oscillations is Time
(1) 6.65 s (1) 200 Wb

(2) 8.89 s (2) 225 Wb

(3) 6.98 s (3) 250 Wb

(4) 8.76 s (4) 275 Wb

7
www.vedantu.com
Answer (3) Answer (1)
Sol. In X-ray tube
d
Sol. ε =
dt hc
min 
eV
d
iR =
dt ⎛ hc ⎞
ln min  ln ⎜ ⎟  lnV
⎝ e ⎠
∫ d   R ∫ idt Slope is negative
Magnitude of change in flux = R × area under Intercept on y-axis is positive
current vs time graph
log min
1 1
= 100 × × × 10
2 2

= 250 Wb
21. An electron beam is accelerated by a potential
difference V to hit a metallic target to produce
X-rays. It produces continuous as well as
characteristic X-rays. If λmin is the smallest possible log V
wavelength of X-ray in the spectrum, the variation of
log λ minwith log V is correctly represented in 22. A diverging lens with magnitude of focal length
25 cm is placed at a distance of 15 cm from a
converging lens of magnitude of focal length 20 cm.
A beam of parallel light falls on the diverging lens.
The final image formed is
(1) log min
(1) Real and at a distance of 40 cm from convergent
lens
log V
(2) Virtual and at a distance of 40 cm from
convergent lens
(3) Real and at a distance of 40 cm from the
divergent lens
(2) log min
(4) Real and at a distance of 6 cm from the
convergent lens
log V
Answer (1)

f1 = 25 cm f2 = 20 cm
Sol.
(3) log min
I1
log V

25 cm 15 cm

For converging lens


(4) log min
u = –40 cm which is equal to 2f
 Image will be real and at a distance of 40 cm
log V
from convergent lens.

8
www.vedantu.com
23. In a Young's double slit experiment, slits are Answer (2)
separated by 0.5 mm, and the screen is placed m1  m
(m1  m2 )v
150 cm away. A beam of light consisting of two Sol. v1  0 m
m1  m2 m2 
wavelengths, 650 nm and 520 nm, is used to obtain 2
interference fringes on the screen. The least distance v
=
from the common central maximum to the point 3
where the bright fringes due to both the wavelengths
⎡v ⎤
coincide is  p1  m. ⎢ ⎥
⎣3 ⎦
(1) 1.56 mm
2m1v
(2) 7.8 mm v2  0
m1  m2
(3) 9.75 mm
4v
(4) 15.6 mm =
3
Answer (2)
m ⎡ 4v ⎤ 2mv
p2  
Sol. For 1 For 2 2 ⎢⎣ 3 ⎥⎦ 3

m1D n 2 D  A p2
y y  de-Broglie wavelength   2 :1
d d B p1

25. Some energy levels of a molecule are shown in the


m 2 4 figure. The ratio of the wavelengths r = 1/2, is given
  
n 1 5 by

For 1 –E
2
4
m1D  E
y , 1 = 650 nm 3
d 1

= 7.8 mm –2 E

24. A particle A of mass m and initial velocity v collides


–3 E
m
with a particle B of mass which is at rest. The
2
collision is head on, and elastic. The ratio of the 4 2
(1) r  (2) r 
de-Broglie wavelengths A to B after the collision is 3 3

3 1
A 1 (3) r  (4) r 
(1)  4 3
B 3
Answer (4)
Sol. From energy level diagram
A
(2) 2
B hc
1 
E

A 2 hc
(3)  2 
B 3 ⎛E ⎞
⎜3⎟
⎝ ⎠
A 1
(4)  1 1
B 2  
2 3

9
www.vedantu.com
26. A radioactive nucleus A with a half life T, decays into 28. In amplitude modulation, sinusoidal carrier frequency
a nucleus B. At t = 0, there is no nucleus B. At used is denoted by c and the signal frequency is
sometime t, the ratio of the number of B to that of denoted by m . The bandwidth ( m ) of the signal
A is 0.3. Then, t is given by
is such that m  c . Which of the following
frequencies is not contained in the modulated wave?
T log2
(1) t  2 log1.3 (1) m (2) c

(3) m  c (4) c – m
log1.3 Answer (1)
(2) t  T log2 Sol. Modulated wave has frequency range.
c ± m
(3) t  T log 1.3   Since c >> m
 m is excluded.
29. Which of the following statements is false?
T
(4) t  log 1.3  (1) Wheatstone bridge is the most sensitive when
all the four resistances are of the same order of
magnitude
Answer (2)
(2) In a balanced Wheatstone bridge if the cell and
the galvanometer are exchanged, the null point
N0  N0 e t is disturbed
Sol.  0.3
N0 e t (3) A rheostat can be used as a potential divider
(4) Kirchhoff’s second law represents energy
 et = 1.3 conservation
 t = ln 1.3 Answer (2)
Sol. In a balanced Wheatstone bridge, the null point
remains unchanged even if cell and galvanometer
⎛ ln 2 ⎞
⎜ T ⎟ t  ln 1.3 are interchanged.
⎝ ⎠
30. The following observations were taken for determining
surface tension T of water by capillary method:
ln(1.3) diameter of capillary, D = 1.25 × 10–2 m
t  T.
ln 2 rise of water, h = 1.45 × 10–2 m.
Using g = 9.80 m/s 2 and the simplified relation
log(1.3) rhg
t T T   103 N / m , the possible error in surface
log2 2
tension is closest to
27. In a common emitter amplifier circuit using an (1) 0.15% (2) 1.5%
n-p-n transistor, the phase difference between the (3) 2.4% (4) 10%
input and the output voltages will be Answer (2)
(1) 45° T D h
Sol.  100   100   100
(2) 90° T D h

(3) 135° 0.01 0.01


=  100   100
1.25 1.45
(4) 180°
100 100
Answer (4) = 
125 145
Sol. In common emitter configuration for n-p-n transistor, = 0.8 + 0.689
phase difference between output and input voltage is = 1.489
180°.
 1.5%

10
www.vedantu.com
PART–B : CHEMISTRY

31. Given 33. U is equal to


C(graphite) + O2(g)  CO2(g); (1) Adiabatic work
rHº = –393.5 kJ mol–1 (2) Isothermal work
1 (3) Isochoric work
H2(g) + O2 (g)  H2O(l);
2 (4) Isobaric work
rHº = –285.8 kJ mol–1 Answer (1)
CO2(g) + 2H2O(l)  CH4(g) + 2O2(g); Sol. For adiabatic process, q = 0
rHº = +890.3 kJ mol–1  As per 1st law of thermodynamics,
Based on the above thermochemical equations, the
U = W
value of rHº at 298 K for the reaction
34. The Tyndall effect is observed only when following
C(graphite) + 2H2(g)  CH4(g) will be
conditions are satisfied
(1) –74.8 kJ mol–1 (2) –144.0 kJ mol–1
(a) The diameter of the dispersed particles is much
(3) +74.8 kJ mol–1 (4) +144.0 kJ mol–1 smaller than the wavelength of the light used.
Answer (1)
(b) The diameter of the dispersed particle is not
Sol. C(graphite) + O2(g)  CO2(g); much smaller than the wavelength of the light
rH° = –393.5 kJ mol–1 ...(i) used

1 (c) The refractive indices of the dispersed phase and


H2 (g)  O2 (g)  H2O(l); dispersion medium are almost similar in
2
rH° = –285.8 kJ mol–1 ...(ii) magnitude

CO2(g) + 2H2O(l)  CH4(g) + 2O2(g); (d) The refractive indices of the dispersed phase and
dispersion medium differ greatly in magnitude
rH° = 890.3 kJ mol–1 ...(iii)
(1) (a) and (c)
By applying the operation
(i) + 2 × (ii) + (iii), we get (2) (b) and (c)

C(graphite) + 2H2(g)  CH4(g); (3) (a) and (d)


rH° = –393.5 –285.8 × 2 + 890.3 (4) (b) and (d)
= –74.8 kJ mol–1
32. 1 gram of a carbonate (M2CO3) on treatment with Sol. For Tyndall effect refractive index of dispersion phase
excess HCl produces 0.01186 mole of CO2. The and dispersion medium must differ significantly.
molar mass of M2CO3 in g mol–1 is Secondly, size of dispersed phase should not differ
(1) 118.6 (2) 11.86 much from wavelength used.
(3) 1186 (4) 84.3 35. A metal crystallises in a face centred cubic
Answer (4) structure. If the edge length of its unit cell is 'a', the
closest approach between two atoms in metallic
Sol. M2CO3 + 2HCl  2MCl + H2O + CO2 crystal will be
nM2CO3  nCO2
(1) 2a
1
 0.01186 a
MM2CO3 (2)
2
1
MM2CO3 = (3) 2a
0.01186
= 84.3 g/mol (4) 2 2a

11
www.vedantu.com
Answer (2) Answer (2)
Sol. In FCC, one of the face is like
0.2 / 60
Sol. 0.45 = i(5.12)  1000
20
A
 i = 0.527

a   CH3COOH2


2CH3 COOH 
1–  
2
C B

 i  1–
By ABC, 2
2a2 = 16r2

 0.527 = 1 –
1 2
 r 2  a2
8
1 
 r a  = 0.473
2 2 2

a   = 0.946
Distance of closest approach = 2r =
2  % association = 94.6%
36. Given 38. The radius of the second Bohr orbit for hydrogen
º º
atom is
ECl –  1.36 V, ECr 3  –0.74 V
2 /Cl /Cr (Planck's Const. h = 6.6262 × 10–34 Js;
º º mass of electron = 9.1091 × 10–31 kg;
ECr O2– /Cr 3 
 1.33 V, EMnO –
/Mn2 
 1.51 V
2 7 4
charge of electron e = 1.60210 × 10–19 C;
Among the following, the strongest reducing agent is permittivity of vacuum
(1) Cr3+ (2) Cl– 0 = 8.854185 × 10–12 kg–1 m–3 A2)
(3) Cr (4) Mn2+ (1) 0.529 Å
Answer (3) (2) 2.12 Å

E°Cr 3+ /Cr O2– = – 1.33 V (3) 1.65 Å


Sol. For Cr3+ , 2 7
(4) 4.76 Å

For Cl–, E°Cl– /Cl = – 1.36 V Answer (2)


2

n2
For Cr, E°Cr/Cr 3 = 0.74 V Sol. r = a0 = 0.529 ×4
Z

For Mn2 , E°Mn2 /MnO – = – 1.51 V = 2.12 Å


4

Positive E° is for Cr, hence it is strongest reducing 39. Two reactions R 1 and R 2 have identical pre-
agent. exponential factors. Activation energy of R1 exceeds
that of R2 by 10 kJ mol–1. If k 1 and k2 are rate
37. The freezing point of benzene decreases by 0.45ºC constants for reactions R1 and R2 respectively at
when 0.2 g of acetic acid is added to 20 g of 300 K, then ln(k2/k1) is equal to
benzene. If acetic acid associates to form a dimer
in benzene, percentage association of acetic acid in (R = 8.314 J mole–1 K–1)
benzene will be (1) 6
(Kf for benzene = 5.12 K kg mol–1) (2) 4
(1) 74.6% (2) 94.6% (3) 8
(3) 64.6% (4) 80.4% (4) 12

12
www.vedantu.com
Answer (2) 42. Which of the following species is not paramagnetic?

–Ea /RT (1) O2


Sol. k1 = Ae 1

(2) B2
–Ea /RT
k2 = Ae 2 (3) NO

1
(4) CO
k2 Ea – Ea2 
e RT 1
k1 = Answer (4)

Sol. CO has 14 electrons (even)  it is diamagnetic


k Ea – Ea2
ln 2 = 1 NO has 15e–(odd)  it is paramagnetic and has
k1 RT
1 unpaired electron in 2p molecular orbital.

B2 has 10e– (even) but still paramagnetic and has


10  103 two unpaired electrons in 2p x and 2p y
= 4
8.314  300 (s-p mixing).

40. pKa of a weak acid (HA) and pKb of a weak base O2 has 16 e– (even) but still paramagnetic and has
(BOH) are 3.2 and 3.4, respectively, The pH of their two unpaired electrons in *2px and *2py molecular
salt (AB) solution is orbitals.

(1) 7.0 43. Which of the following reactions is an example of a


redox reaction?
(2) 1.0
(1) XeF6 + H2O  XeOF4 + 2HF
(3) 7.2

(4) 6.9 (2) XeF6 + 2H2O  XeO2F2 + 4HF

Answer (4) (3) XeF4 + O2F2  XeF6 + O2

(4) XeF2 + PF5  [XeF]+ PF6–


1
Sol. pH = 7 +  pK a – pK b 
2 Answer (3)

Sol. Xe is oxidised from +4(in XeF4) to +6(in XeF6)


1
= 7 +  3.2 – 3.4
2 Oxygen is reduced from +1 (in O2F2) to zero (in O2)

= 6.9 44. A water sample has ppm level concentration of


following anions
41. Both lithium and magnesium display several similar
properties due to the diagonal relationship, however, F– = 10; SO42– = 100; NO3– = 50
the one which is incorrect, is
The anion/anions that make/makes the water sample
(1) Both form nitrides unsuitable for drinking is/are
(2) Nitrates of both Li and Mg yield NO2 and O2 on (1) Only F–
heating
(2) Only SO42–
(3) Both form basic carbonates
(3) Only NO3–
(4) Both form soluble bicarbonates
(4) Both SO42– and NO3–
Answer (3)

Sol. Mg forms basic carbonate Answer (1)



Sol. Permissible limit of F in drinking water is upto
3MgCO3 · Mg  OH2 ·3H2O but no such basic –
1 ppm. Excess concentration of F  10 ppm
carbonate is formed by Li. causes decay of bones.
13
www.vedantu.com
45. The group having isoelectronic species is Answer (2)

(1) O2–, F–, Na, Mg2+ Sol. Na2C2O4  H2SO4 


 Na2SO4  H2 C2 O4
( X) Conc. oxalic acid
(2) O–, F–, Na+, Mg2+

(3) O2–, F–, Na+, Mg2+ Conc. H SO


H2 C2 O 4 
2 4
 CO  CO2 
 
–H2O (effervescence)
(4) O–, F–, Na, Mg+

Answer (3)
Na2 C2 O 4  CaCl2  CaC2 O 4   2NaCl
( X) white ppt.
Sol. Mg2+, Na+, O2– and F– all have 10 electrons each.

 2
46. The products obtained when chlorine gas reacts with 2MnO4–  5C2O2–
4  16H  2Mn  10CO2  8H2O
cold and dilute aqueous NaOH are
49. The most abundant elements by mass in the body
(1) Cl– and ClO– (2) Cl– and ClO2–
of a healthy human adult are :
(3) ClO– and ClO3– (4) ClO2– and ClO3– Oxygen (61.4%); Carbon (22.9%); Hydrogen (10.0%)
Answer (1) and Nitrogen (2.6%).
The weight which a 75 kg person would gain if all
Sol. Cl2  2NaOH  NaCl  NaOCl  H2 O 1H atoms are replaced by 2H atoms is
Cold & dilute Sodium
hypochlorite
(1) 7.5 kg
47. In the following reactions, ZnO is respectively acting (2) 10 kg
as a/an
(3) 15 kg
(a) ZnO + Na2O  Na2ZnO2 (4) 37.5 kg
(b) ZnO + CO2  ZnCO3 Answer (1)
(1) Acid and acid
10
Sol. Mass of hydrogen =  75 = 7.5 kg
(2) Acid and base 100

(3) Base and acid Replacing 1H by 2H would replace 7.5 kg with 15 kg

(4) Base and base  Net gain = 7.5 kg

Answer (2) 50. On treatment of 100 mL of 0.1 M solution of


CoCl3  6H2O with excess AgNO3; 1.2 × 1022 ions are
Sol. In (a), ZnO acts as acidic oxide as Na2O is basic precipitated. The complex is
oxide.
(1) [Co(H2O)6]Cl3
In (b), ZnO acts as basic oxide as CO2 is acidic (2) [Co(H2O)5Cl]Cl2  H2O
oxide.
(3) [Co(H2O)4Cl2]Cl  2H2O
48. Sodium salt of an organic acid 'X' produces
effervescence with conc. H2SO4. 'X' reacts with the (4) [Co(H2O)3Cl3]  3H2O
acidified aqueous CaCl2 solution to give a white Answer (2)
precipitate which decolourises acidic solution of
KMnO4. 'X' is 1.2  1022
Sol. Millimoles of AgNO3 =  1000 = 20
6  1023
(1) CH3COONa
Millimoles of CoCl3·6H2O = 0.1 × 100 = 10
(2) Na2C2O4
 Each mole of CoCl3·6H2O gives two chloride
(3) C6H5COONa ions.
(4) HCOONa  [Co(H2O)5Cl]Cl2·H2O

14
www.vedantu.com
51. Which of the following compounds will form Answer (3)
significant amount of meta product during
mono-nitration reaction? CH3
NH2 +
Na O

C CH3
O
(1) CH3
Sol. Br
NHCOCH3
CH3
(2)
O O C CH3

OH CH3
(Product)

(3)
The above product does not have any C = C or
C  C bond, so, it will not give Br2-water test.
OCOCH3 53. The formation of which of the following polymers
involves hydrolysis reaction?
(4)
(1) Nylon 6, 6
(2) Terylene
Answer (1)
(3) Nylon 6

NH2 NH3 (4) Bakelite


Answer (3)
Sol. H+ NO2
Sol. Caprolactam is hydrolysed to produce caproic acid
which undergoes condensation to produce Nylon-6.
NH3 NH3 NH3
NO2 O O
+ +
NH H3O+ C
NO2
(51%) (2%) HO (CH2)5 – NH2
NO2
(Caprolactam) (Caproic acid)
(47%)

52. Which of the following, upon treatment with 54. Which of the following molecules is least resonance
tert-BuONa followed by addition of bromine water, stabilized?
fails to decolourize the colour of bromine?
O
(1)
(1) N
Br
O
(2)
(2)
Br
O

O
(3)
Br (3)

C6H5
(4) (4)
Br O
15
www.vedantu.com
Answer (2) Answer (4)
Sol. However, all molecules given in options are stabilised
Br
by resonance but compound given in option (2) is H t-BuOK C6H5
least resonance stabilised (other three are aromatic) Sol. C6H5

C6H5 (E-2) C6H5

(+)
O O
57. Which of the following compounds will behave as a
reducing sugar in an aqueous KOH solution?

HOH2C O CH2OH

55. The increasing order of the reactivity of the following (1) HO OCH
halides for the SN1 reaction is 3

I. CH 3CHCH 2CH 3 OH

Cl
HOH2C
II. CH3CH2CH2Cl O CH OCH
2 3
III. p–H3CO – C6H4 – CH2Cl
(2) OH
(1) (I) < (III) < (II) (2) (II) < (III) < (I) OH
(3) (III) < (II) < (I) (4) (II) < (I) < (III) OH
Answer (4)
Sol. Rate of SN1 reaction  stability of carbocation HOH2C O CH2OH

I. CH3 – CH – CH2 – CH3 CH3 – CH – CH2 – CH3


(3) HO OCOCH
3

Cl
OH
II. CH3 – CH2 – CH2 – Cl CH3 – CH2 – CH2
HOH2C O CH2OH

CH2 – Cl CH2 (4)


HO

III. OH

OCH3 OCH3 Answer (3)

So, II < I < III Sol. Sugars in which there is free anomeric –OH group
are reducing sugars
Increase stability of carbocation and hence increase
OH
reactivity of halides.
56. The major product obtained in the following reaction CH2 O CH2 – OH
O KOH(aq.)
is
HO O – C– CH
3
Br
H OH
t-BuOK
C6H5

C6H5 OH
(+)
CH2 O CH2 – OH
(1) (+)C6H5CH(OtBu)CH2C6H5
(2) (–)C6H5CH(OtBu)CH2C6H5 HO
OH + CH3COOK
(3) (±)C6H5CH(OtBu)CH2C6H5
OH
(4) C6H5CH = CHC6H5 Free anomeric group

16
www.vedantu.com
58. 3-Methyl-pent-2-ene on reaction with HBr in presence
of peroxide forms an addition product. The number of O (i) CH3MgBr HO CH3
possible stereoisomers for the product is (3 moles)
(ii) H2O
(1) Two

(2) Four CH3


C – OCH3 HO – C
O CH3
(3) Six

(4) Zero
60. The major product obtained in the following reaction
Answer (2) is
O
CH3 O
HBr
Sol. CH3 – CH = C – CH2 – CH3 R2O2
3-methyl pent-2-ene DIBAL-H

CH3 COOH
CH3 – *CH –*C – CH2 – CH3
Br H CHO
(1)
Product (X)
COOH

Since product (X) contains two chiral centres and it


is unsymmetrical.
CHO
So, its total stereoisomers = 22 = 4. (2)
CHO
59. The correct sequence of reagents for the following
conversion will be
OH

O HO CH3
(3) CHO

COOH

CHO HO CH3
CH3 OH

(1) CH3MgBr, [Ag(NH3)2]+OH–, H+/CH3OH (4) CHO

(2) [Ag(NH3)2]+OH–, CH3MgBr, H+/CH3OH CHO

(3) [Ag(NH3)2]+OH–, H+/CH3OH, CH3MgBr


Answer (4)
(4) CH3MgBr, H+/CH3OH, [Ag(NH3)2]+OH–
Sol. DIBAL — H reduces esters and carboxylic acids
Answer (3) into aldehydes

O O O O
O OH H
[Ag(NH3)2]+OH– CH3 – OH/H+
Sol. Esterification DIBAL-H

C=O C=O COOH CHO


H HO
17
www.vedantu.com
PART–A : MATHEMATICS

⎡ 1 1⎤ Given difference of roots = 1


61. The function f : R  ⎢ , ⎥ defined as
⎣ 2 2⎦  || = 1
x  D1
f (x)  2 , is
1 x
4 2
(1) Injective but not surjective  n2  (n  31)  1
3
(2) Surjective but not injective
(3) Neither injective nor surjective So, n = 11

(4) Invertible 63. Let  be a complex number such that 2 + 1 = z


Answer (2) where z  3 . If

x 1 1 1
Sol. f ( x ) 
1 x2
1 2  1 2  3k , then k is equal to
2 2
(1  x )  1  x  2 x 1 x 1 2 7
f ( x )  2 2
 2 2
(1  x ) (1  x )
f(x) changes sign in different intervals. (1) z (2) –1
 Not injective. (3) 1 (4) –z
x Answer (4)
y
1 x2
Sol. 2 + 1 = z , z  3 i
yx 2  x  y  0
For y  0 1  3i
 Cube root of unity.
2
⎡ 1 1⎤
D  1  4 y 2  0 ⇒ y  ⎢  , ⎥  {0} C1  C1 + C2 + C3
⎣ 2 2⎦
For, y = 0 x = 0 1 1 1 1 1 1 3 1 1
 Part of range 2
1 1     1    0  2
2 2

⎡ 1 1⎤
 Range : ⎢  , ⎥ 1 2 7 1 2  0 2 
⎣ 2 2⎦
 Surjective but not injective. = 3 (2 – 4)
62. If, for a positive integer n, the quadratic equation,
x( x  1)  ( x  1)( x  2)  ...  ( x  n  1)( x  n )  10n ⎡⎛ 1  3i ⎞ ⎛ 1  3i ⎞ ⎤
= 3 ⎢⎜ ⎟⎜ ⎟⎥
has two consecutive integral solutions, then n is ⎣⎝ 2 ⎠ ⎝ 2 ⎠⎦
equal to
= 3 3i
(1) 9 (2) 10
(3) 11 (4) 12 = –3z
Answer (3)  k = –z
Sol. Rearranging equation, we get
⎡ 2 3 ⎤
64. If A  ⎢ 2
⎥ , then adj (3A + 12A) is equal to
nx 2  1  3  5  ....  (2n  1) x ⎣ 4 1 ⎦
 1 2  2  3  ...  ( n  1)n  10n
⎡ 51 63 ⎤ ⎡ 51 84 ⎤
(n  1)n(n  1) (1) ⎢ ⎥ (2) ⎢ ⎥
⇒ nx 2  n 2 x   10n ⎣84 72 ⎦ ⎣ 63 72 ⎦
3
⎡ 72 63 ⎤ ⎡ 72 84 ⎤
⎛ n 2  31 ⎞ (3) ⎢ ⎥ (4) ⎢ ⎥
⇒ x 2  nx  ⎜ ⎟0 ⎣ 84 51 ⎦ ⎣ 63 51 ⎦
⎝ 3 ⎠
18
www.vedantu.com
Answer (1) Eq. (1) & (2) are identical i.e.,x + y + z = 1
To have no solution with x + by + z = 0.
⎡ 2 3 ⎤
Sol. A  ⎢ ⎥ b=1
⎣ 4 1 ⎦
66. A man X has 7 friends, 4 of them are ladies and 3
2 3 are men. His wife Y also has 7 friends, 3 of them
A  I 
4 1  are ladies and 4 are men. Assume X and Y have no
common friends. Then the total number of ways in
= (2 – 2– + 2) – 12 which X and Y together can throw a party inviting 3
ladies and 3 men, so that 3 friends of each of X and
f ( )   2  3  10 Y are in this party, is
(1) 468 (2) 469
∵ A satisfies f ( )
(3) 484 (4) 485
 A2 – 3A –10I = 0 Answer (4)
A2 – 3A = 10I Sol. X(4 L 3 G) Y(3 L 4 G)
3A2 – 9A = 30I 3L0G 0L3G

3A2 + 12A = 30I + 21A 2L1G 1L2G


1L2G 2L1G
⎡30 0 ⎤ ⎡ 42 63 ⎤ 0L3G 3L0G
⎢ ⎥⎢ ⎥
⎢⎣ 0 30 ⎥⎦ ⎢⎣ 84 21 ⎥⎦ Required number of ways

⎡ 72 63 ⎤
     C 
2 2 2
= 4C3  4C3  4
C2  3C1 4
C1  3C2 3
⎢ ⎥ 3

⎣⎢ 84 51 ⎦⎥ = 16 + 324 + 144 + 1


⎡ 51 63 ⎤ = 485
adj(3 A2  12 A)  ⎢ ⎥
⎢⎣84 72 ⎥⎦ 67. The value of

65. If S is the set of distinct values of b for which the ( 21C1  10C1 )  ( 21C2  10C2 )  ( 21C3  10C3 ) 
following system of linear equations
( 21C4  10C4 )  ...  ( 21C10  10C10 ) is
x y z 1
(1) 221 – 210
x  ay  z  1 (2) 220 – 29
(3) 220 – 210
ax  by  z  0 (4) 221 – 211
has no solution, then S is Answer (3)
(1) An infinite set
Sol.
21
C1  21C2  ...  21C10 
1 21
2

C0  21C1  ...  21C21  1 
(2) A finite set containing two or more elements
= 220 – 1
(3) A singleton
(4) An empty set  10

C1  10C2  ...  10C10  210  1

Answer (3)  Required sum = (220 – 1) – (210 – 1)


Sol. = 220 – 210
68. For any three positive real numbers a, b and c,
1 1 1
9(25a2  b2 )  25(c 2  3ac )  15b(3a  c ).
1 a 10
Then
a b 1
(1) b, c and a are in A.P.
 –(1 – a)2 = 0 (2) a, b and c are in A.P.
 a=1 (3) a, b and c are in G.P.
For a = 1 (4) b, c and a are in G.P.

19
www.vedantu.com
Answer (1) cot  cos
70. lim equals
Sol. 9(25a2  b2 )  25(c 2  3ac )  15b (3a  c ) x
 (   2)3
2

1 1
 (15a )2  (3b)2  (5c )2  45ab  15bc  75ac  0 (1) (2)
16 8
 (15a  3b)2  (3b  5c )2  (15a  5c )2  0 1 1
(3) (4)
4 24
It is possible when Answer (1)
15a  3b  0 and 3b  5c  0 and 15a  5c  0 cot x  cos x
Sol. lim
x
 (   2 x )3
15a  3b  5c 2


a b c Put, x t
  2
1 5 3
tan t  sin t
lim
 b, c, a are in A.P. t 0 8t 3
69. Let a, b, c  R. If f(x) = ax2 + bx + c is such that t
sin t  2 sin2
a + b + c = 3 and 2
= lim
t 0 8t 3
f ( x  y )  f ( x )  f ( y )  xy ,  x, y  R,
1
= .
10 16
then ∑ f (n ) is equal to
1⎞ 1 ⎛ 6 x x ⎞
n 1 ⎛
71. If for x  ⎜ 0, ⎟ , the derivative of tan ⎜ ⎟ is
⎝ 4⎠ ⎝ 1  9x 3 ⎠
(1) 165 (2) 190
x  g ( x ) , then g(x) equals
(3) 255 (4) 330
3x x 3x
Answer (4) (1) (2)
1 9x 3
1 9x3
Sol. As, f ( x  y )  f ( x )  f ( y )  xy 3 9
(3) 3 (4)
Given, f (1)  3 1  9x 1  9x 3
Answer (4)
Putting, x  y  1  f (2)  2f (1)  1  7 ⎛ 1⎞
Sol. f ( x )  2 tan1(3 x x ) For x  ⎜ 0, ⎟
⎝ 4⎠
Similarly, x  1, y  2  f (3)  f (1)  f (2)  2  12
9 x
f ( x ) 
10
1 9x3
Now, ∑ f (n ) = f (1)  f (2)  f (3)  ...  f (10)
9
n 1
g( x ) 
1 9x3
= 3 + 7 + 12 + 18 + ... = S (let)
72. The normal to the curve y ( x  2)( x  3)  x  6 at
Now, Sn  3  7  12  18  ...  t n the point where the curve intersects the y-axis
passes through the point
Again, Sn  3  7  12  ...  t n 1  t n
⎛ 1 1⎞ ⎛ 1 1⎞
(1) ⎜ , ⎟ (2) ⎜ ,  ⎟
We get, t n  3  4  5  ... n terms ⎝2 2⎠ ⎝2 3⎠
⎛ 1 1⎞ ⎛ 1 1⎞
n (n  5) (3) ⎜ , ⎟ (4) ⎜  ,  ⎟
= ⎝2 3⎠ ⎝ 2 2⎠
2 Answer (1)
n
n (n  1)( n  8) Sol. y ( x  2)( x  3)  x  6
∑ tn = 2 ∑ n 2  5 ∑ n =
1
i.e., Sn =
n 1 6 At y-axis, x = 0, y = 1
Now, on differentiation.
10  11 18 dy
So, S10 =  330 ( x  2)( x  3)  y (2 x  5)  1
6 dx
20
www.vedantu.com
dy Answer (1)
(6)  1( 5)  1
dx n
Sol. In  ∫ tan xdx, n  1
dy 6
 1
dx 6 I4  I6  ∫ (tan4 x  tan6 x )dx
Now slope of normal = –1
Equation of normal y – 1 = –1(x – 0)  ∫ tan4 x sec 2 xdx
y+x–1=0 ... (i) Let tanx = t
⎛ 1 1⎞ sec2x dx = dt
Line (i) passes through ⎜ , ⎟
⎝2 2⎠
73. Twenty meters of wire is available for fencing off a  ∫ t 4 dt
flower-bed in the form of a circular sector. Then the
maximum area (in sq. m) of the flower-bed, is t5
 C
5
(1) 10 (2) 25
1
(3) 30 (4) 12.5  tan5 x  C
5
Answer (2)
r 1
Sol. a ,b 0
5

3
r  r 4
dx
75. The integral ∫ 1  cos x is equal to
2r  r  20 ... (i) 
4
 r 2
A = area =  r 2  ... (ii) (1) 2 (2) 4
2 2
(3) –1 (4) –2
r 2 ⎛ 20  2r ⎞
A ⎜ ⎟ Answer (1)
2⎝ r ⎠
3 3
⎛ 20r  2r 2 ⎞ 2
4
dx 1 4
x
A⎜ ⎟  10r  r 2
⎝ 2 ⎠ Sol. ∫ x 2
dx 
2 ∫ sec 2
dx
 2cos 
A to be maximum 4 2 4

3
dA
 10  2r  0 ⇒ r  5 ⎡ x⎤4
tan ⎥
dr 1⎢ 2
 ⎢ ⎥
2⎢ 1 ⎥
d2A ⎣ 2 ⎦
 2  0
dr 2 4

3 
Hence for r = 5, A is maximum  tan  tan
8 8
Now, 10 + ·5 = 20  = 2 (radian)
⎡ 
2 ⎢ 1  cos
  5 2  25 sq m  4  2 1 2 1
Area =
2 ⎢ tan  
⎢ 8  2 1 1
1  cos
n
⎢⎣ 4
74. Let In  ∫ tan xdx,(n  1) . If
3 ⎤
I4  I6  a tan5 x  bx 5  C, where C is a constant 1  cos ⎥
3 4  2 1
of integration, then the ordered pair (a, b) is equal to tan   2  1⎥
8 3 2 1 ⎥
1  cos
⎛1 ⎞ ⎛1 ⎞ 4 ⎥⎦
(1) ⎜ ,0 ⎟ (2) ⎜ , 1⎟
⎝5 ⎠ ⎝5 ⎠
 ( 2  1)  ( 2  1)
⎛ 1 ⎞ ⎛ 1 ⎞
(3) ⎜  ,0 ⎟ (4) ⎜  ,1⎟
⎝ 5 ⎠ ⎝ 5 ⎠ 2
21
www.vedantu.com
76. The area (in sq. units) of the region (1  1)  2  C  C = 4
{(x, y) : x  0, x + y  3, x2  4y and y  1  x }
Now, ( y  1)(2  sin x )  4
is

3 7 For, x 
(1) (2) 2
2 3
( y  1)(2  1)  4
5 59
(3) (4) 4
2 12 y 1
3
Answer (3)
y 4 1
Sol. y 1
3 3
) 78. Let k be an integer such that the triangle with
,2
(1
vertices (k, –3k), (5, k) and (–k, 2) has area
(2, 1) 28 sq. units. Then the orthocentre of this triangle is
(0, 1)
at the point
x x
O x=1 x=2 x+
y=
3 ⎛ 3⎞ ⎛ 3⎞
(1) ⎜⎝ 1, ⎟⎠ (2) ⎜ 1,  ⎟
4 ⎝ 4⎠
x=0

y ⎛ 1⎞ ⎛ 1⎞
(3) ⎜ 2, ⎟ (4) ⎜ 2,  ⎟
⎝ 2⎠ ⎝ 2⎠
Area of shaded region
1 2 Answer (3)
⎛ x2 ⎞ ⎛ x2 ⎞
 ∫ ⎜ x  1 ⎟ dx  ∫ ⎜ (3  x )  ⎟ dx
0
⎝ 4 ⎠ 1
⎝ 4 ⎠ k 3 k 1
1
Sol. Area = 5 k 1  28
5 2
 sq. unit k 2 1
2
dy k  5 4k 0
77. If  2  sin x    y  1 cos x  0 and y(0) = 1, then 5  k k  2 0   56
dx
k 2 1
⎛ ⎞
y ⎜ ⎟ is equal to
⎝ 2⎠ (k 2  7k  10)  4k 2  20k   56

2 1 5k 2  13k  10   56
(1)  (2) 
3 3
5k 2  13k  46  0 5K 2  13K  66  0
4 1
(3) (4) 5k 2  13k  46  0
3 3
13  169  920
Answer (4) k =
10
dy
Sol. (2  sin x )  ( y  1)cos x  0 = 2, –4.6
dx reject
⎛⎞ For k = 2
y (0)  1, y ⎜ ⎟  ?
⎝2⎠ A (2, –6)

1 cos x
8

dy  dx  0
=

y 1 2  sin x E
m

m= –2
ln| y  1|  ln(2  sin x )  ln C
(5, 2) B
D C (–2, 2)
( y  1)(2  sin x )  C
m=0
Put x = 0, y = 1

22
www.vedantu.com
Equation of AD,
k
x=2 ...(i)  r 
2
Also equation of BE,
k2
1 Equation of circle is x 2  ( y  k )2 
y  2  ( x  5) 2
2
It passes through point P
2y  4  x  5
2
x  2y  1  0 t2 ⎛ t2 ⎞ k2
...(ii)  ⎜⎜ 4   k ⎟⎟ 
4 ⎝ 4 ⎠ 2
Solving (i) & (ii), 2y = 1

1 t 4  t 2 (8k  28)  8k 2  128k  256  0 ...(ii)


y
2
For t = 0  k 2  16k  32  0
⎛ 1⎞
Orthocentre is ⎜ 2, ⎟ k 84 2
⎝ 2⎠
k
79. The radius of a circle, having minimum area, which  r   4( 2  1) (discarding 4( 2  1) ) ...(iii)
touches the curve y = 4 – x2 and the lines, y = |x| 2
is
For t   14  4k
(1) 2  2  1 (2) 4  2  1
(14  4k )2  (14  4k )(8k  28)  8k 2  128k  256  0
(3) 4  2  1 (4) 2  2  1
2k 2  4k  15  0
Answer (2)
y 2  34
Sol. k
2
P
 k 17  2
r  (Ignoring negative ...(iv)
2 2
x value of r)
O
From (iii) & (iv),

17  2
rmin 
x 2  ( y  4) 2

⎛t t2 ⎞ But from options, r  4( 2  1)


Let a point on the parabola P ⎜⎜ , 4  ⎟⎟
⎝2 4 ⎠
y
Equation of normal at P is
(0, 4)
2
t 1⎛ t⎞
y 4  ⎜x  ⎟
4 t⎝ 2⎠
x
(0, 0)
t3 7
 x  ty   t  0
4 2
It passes through centre of circle, say (0, k) 80. The eccentricity of an ellipse whose centre is at the
1
t3 7 origin is . If one of its directrices is x = – 4, then
tk   t  0 ...(i) 2
4 2
⎛ 3⎞
t = 0, t 2  14  4k the equation of the normal to it at ⎜ 1, ⎟ is
⎝ 2⎠
0k
Radius = r  (Length of perpendicular (1) 4x – 2y = 1 (2) 4x + 2y = 7
2
from (0, k) to y = x) (3) x + 2y = 4 (4) 2y – x = 2

23
www.vedantu.com
Answer (1) y2
 x2  1
Sol. 3

 Tangent at P ( 2, 3) is y
2x  1
3

Clearly it passes through (2 2, 3 3)


x = –4
82. The distance of the point (1, 3, –7) from the plane
1
e passing through the point (1, –1, –1), having normal
2 x 1 y  2 z  4
perpendicular to both the lines  
a 1 2 3
 4 x  2 y 1 z  7
e and   , is
2 1 1
a  4  e
10
(1)
a2 83

Now, b2  a2 (1  e2 )  3 5
(2)
83
Equation to ellipse
10
x2 y 2 (3)
 1 74
4 3
20
Equation of normal is (4)
74
3
y
x 1 2  4 x  2y  1  0 Answer (1)

1 3
Sol. Let the plane be
4 23

81. A hyperbola passes through the point P ( 2, 3) a( x  1)  b( y  1)  c( z  1)  0


and has foci at (±2, 0). Then the tangent to this It is perpendicular to the given lines
hyperbola at P also passes through the point
a – 2b + 3c = 0
(1) (2 2, 3 3)
2a – b – c = 0
(2) ( 3, 2)
Solving, a : b : c = 5 : 7 : 3
(3) (  2,  3)
 The plane is 5x + 7y + 3z + 5 = 0
(4) (3 2, 2 3) 10
Distance of (1, 3, –7) from this plane =
Answer (1) 83

x2 y 2 83. If the image of the point P(1, –2, 3) in the plane,


Sol.  1 2x + 3y – 4z + 22 = 0 measured parallel to the line,
a2 b2
x y z
  is Q, then PQ is equal to
a2  b2  4 1 4 5
2 3 (1) 2 42
and  1
a2 b2
2 3 (2) 42
2
 2
1
4b b
(3) 6 5
 b 3 2

 a2  1 (4) 3 5

24
www.vedantu.com
Answer (1) (1) 6

x 1 y  2 z  3 (2) 4
Sol. Equation of PQ,  
1 4 5
6
(3)
Let M be (  1, 4  2, 5  3) 25
P
12
(4)
5
M Answer (4)
Sol. n = 10
15
p(Probability of drawing a green ball) =
Q 25
3 2
As it lies on 2x + 3y – 4z + 22 = 0  p , q
5 5
=1 var(X) = n.p.q
For Q,  = 2 6 12
= 10 

25 5
Distance PQ  2 12  42  52  2 42 86. For three events A, B and C, P (Exactly one of A or
 B occurs) = P(Exactly one of B or C occurs)
 
84. Let a  2iˆ  jˆ  2kˆ and b  iˆ  ˆj . Let c be a vector 1
= P (Exactly one of C or A occurs) = and
     4
such that | c  a | 3, (a  b )  c  3 and the angle
     1
between c and a  b be 30°. Then a  c is equal to P(All the three events occur simultaneously) = .
16
Then the probability that at least one of the events
(1) 2 occurs, is
(2) 5 7 7
(1) (2)
1 16 64
(3)
8 3 7
(3) (4)
25 16 32
(4)
8 Answer (1)
Answer (1) 1
Sol. P ( A)  P (B )  P ( A  B ) 
     4
Sol. | (a  b )  c |  3 a  b  2iˆ  2 ˆj  kˆ
1
      P (B )  P (C )  P (B  C ) 
⇒ | a  b | | c | sin 30  3 |a |  3  ab 4
 1
⇒ |c | 2 P (C )  P ( A)  P ( A  C ) 
4
 
|c a | 3
P ( A)  P (B )  P (C )  P ( A  B )  P (B  C )
    3
⇒ | c |2  | a |2 2(a  c )  9  P( A  C ) 
8
  932
a c  2
2 1
∵ P( A  B  C ) 
16
85. A box contains 15 green and 10 yellow balls. If 10
balls are randomly drawn, one-by-one, with 3 1 7
replacement, then the variance of the number of  P( A  B  C )   
8 16 16
green balls drawn is

25
www.vedantu.com
87. If two different numbers are taken from the set 1
{0, 1, 2, 3, ......, 10}; then the probability that their = 
3
sum as well as absolute difference are both multiple
cos4x = 2 cos2 2x – 1
of 4, is
2
12 = 1
(1) 9
55
7
14 = 
(2) 9
45
89. Let a vertical tower AB have its end A on the level
7 ground. Let C be the mid-point of AB and P be a
(3) point on the ground such that AP = 2AB. If
55
BPC =  then tan  is
6 1
(4)
55 (1)
4
Answer (4)
2
Sol. Total number of ways = 11C (2)
2 9
= 55
4
Favourable ways are (3)
9
(0, 4), (0, 8), (4, 8), (2, 6), (2, 10), (6, 10)
6
(4)
6 7
Probability =
55 Answer (2)
88. If 5 (tan2x – cos2x) = 2cos 2x + 9, then the value of 1
cos 4x is Sol. tan   B
4
x
1
(1) 1 C
3 tan       x
2
2 
P A
(2) 1 4x
9  tan  1
4 
7  1 2
(3)  1  tan 
9 4
2
3 Solving tan  
(4)  9
5
90. The following statement (p q) [(~ p q) q]
Answer (3)
is
Sol. 5 tan2x = 9 cos2x + 7 (1) Equivalent to ~ p q
5 sec2x – 5 = 9 cos2x + 7 (2) Equivalent to p ~ q
Let cos2x =t (3) A fallacy
(4) A tautology
5
 9t  12 Answer (4)
t
9t2 + 12t – 5 = 0
Sol. p q p  q (~p  q) (~p  q)  q (p  q)  [(~p  q)  q]
1 5 T T T T T T
t as t T F F T F T
3 3
F T T T T T
1 F F T F T T
cos2 x  , cos 2x = 2cos2x – 1
3 (a tautology)

  
26
www.vedantu.com
Answers & Solutions For JEE MAIN- 2015

(Code-A)

Time Duration : 3 hrs. Maximum Marks : 360

(Physics, Chemistry and Mathematics)


Important Instructions :

1. The test is of 3 hours duration.


2. The Test Booklet consists of 90 questions. The maximum marks are 360.
3. There are three parts in the question paper A, B, C consisting of Physics,
Chemistry and Mathematics having 30 questions in each part of equal weightage. Each
question is allotted 4 (four) marks for each correct response.
4. Candidates will be awarded marks as stated above in Instructions No. 3 for correct response
of each question. ¼ (one-fourth) marks will be deducted for indicating incorrect response
of each question. No deduction from the total score will be made if no response is
indicated for an item in the answer sheet.
5. There is only one correct response for each question. Filling up more than one response
in each question will be treated as wrong response and marks for wrong response will be
deducted accordingly as per instruction 4 above.
6. Use Blue/Black Ball Point Pen only for writing particulars/marking responses on Side-1 and
Side-2 of the Answer Sheet. Use of pencil is strictly prohibited.
7. No candidate is allowed to carry any textual material, printed or written, bits of papers,
pager, mobile phone, any electronic device, etc. except the Admit Card inside the
examination room/hall.
8. The CODE for this Booklet is A. Make sure that the CODE printed on Side-2 of the
Answer Sheet and also tally the serial number of the Test Booklet and Answer Sheet are
the same as that on this booklet. In case of discrepancy, the candidate should immediately
report the matter to the Invigilator for replacement of both the Test Booklet and the Answer
Sheet.
PART–A : PHYSICS

1. Two stones are thrown up simultaneously from the 2. The period of oscillation of a simple pendulum is
edge of a cliff 240 m high with initial speed of L
T  2
10 m/s and 40 m/s respectively. Which of the g . Measured value of L is 20.0 cm known
following graph best represents the time variation of to 1 mm accuracy and time for 100 oscillations of
relative position of the second stone with respect to
the pendulum is found to be 90 s using a wrist
the first?
watch of 1 s resolution. The accuracy in the
(Assume stones do not rebound after hitting the determination of g is
ground and neglect air resistance, take g = 10 m/s2)
(1) 2% (2) 3%
(The figures are schematic and not drawn to scale)
(3) 1% (4) 5%

(y2 – y1) m Answer (2)


240
2 l
Sol. g  4 .
T2
(1) g l T
  100   100  2  100
t (s) g l T
t 8 12
l t
=  100  2.  100
(y2 – y1) m l t
240 0.1 1
=  100  2   100
20.0 90
100 200 1 20
(2) =     3%
200 90 2 9
t (s)
12
F
3. A B
(y2 – y1) m
240

(3) Given in the figure are two blocks A and B of


t (s) weight 20 N and 100 N, respectively. These are
8 12 being pressed against a wall by a force F as shown.
If the coefficient of friction between the blocks is 0.1
and between block B and the wall is 0.15, the
(y2 – y1) m frictional force applied by the wall on block B is
240
(1) 100 N (2) 80 N
(3) 120 N (4) 150 N
(4)
Answer (3)
t (s)
8 12
fs
Answer (3)
A B
Sol. Till both are in air (From t = 0 to t = 8 sec) Sol. F N
x = x2 – x1 = 30t
20 N 100 N
 x  t
When second stone hits ground and first stone is in Clearly fs = 120 N (for vertical equilibrium of the
air x decreases. system)

2
4. A particle of mass m moving in the x direction with 6. From a solid sphere of mass M and radius R a cube
speed 2v is hit by another particle of mass 2m of maximum possible volume is cut. Moment of
moving in the y direction wth speed v. If the
inertia of cube about an axis passing through its
collision is perfectly inelastic, the percentage loss in
the energy during the collision is close to center and perpendicular to one of its faces is
(1) 44% (2) 50% MR 2 MR 2
(1) (2)
(3) 56% (4) 62% 32 2  16 2 
Answer (3) 4 MR 2 4 MR 2
(3) (4)
Sol. m 2v 9 3 3 3
v v' Answer (3)
= 2mv 2
 v'
3m Sol. d  2 R  a 3
2m

1 1
m  2 v    2m  v 2
2
KE loss =
2 2

2
1 ⎛ 2mv 2 ⎞ 5 2
   3m  ⎜⎜ ⎟⎟  mv
2 ⎝ 3m ⎠ 3
2
5  a R
mv 2 3
Required % = 3  100  56%
2mv 2  mv 2
4 3
R
M 3
 3
5. Distance of the centre of mass of a solid uniform
 
cone from its vertex is z0. If the radius of its base is M  ⎛ 2 ⎞3 2
R and its height is h then z0 is equal to ⎜ R⎟
⎝ 3 ⎠
h2 3h
(1) (2) 2M
4R 4  M' 
3
5h 3h 2
(3) (4)
8 8R M ' a2 2 M 4 2 1
I   R 
6 3 3 6
Answer (2)

Sol. dm  r 2 .dy. 4 MR 2
I
9 3
7. From a solid sphere of mass M and radius R, a
 R
y spherical portion of radius is removed, as
2
r h shown in the figure. Taking gravitational potential
V = 0 at r = , the potential at the centre of the
cavity thus formed is
(G = gravitational constant)

R
h

yCM 
∫ ydm  ∫ 0
r 2 dy   y
1 2
∫ dm 3
R h GM GM
(1) (2)
2R R
3h
 2GM 2GM
4 (3)
3R
(4)
R
3
Answer (2) Answer (3)
Sol. V = V1 – V2
1⎛U ⎞ 1 4
Sol. P  ⎜ ⎟  kT ...(i)
GM ⎡ 2⎤ 3⎝V ⎠ 3
V1   3 ⎢ 3R 2  ⎛⎜ R ⎞⎟ ⎥
2R ⎣ ⎝2⎠ ⎦ PV = RT ...(ii)
M RT 1 4
3G ⎛⎜ ⎞⎟  kT
V2   ⎝ 8 ⎠ V 3
R
2 ⎜ ⎞⎟

 V  T–3
⎝2⎠
1
GM R
 V T
R
8. A pendulum made of a uniform wire of cross- 10. A solid body of constant heat capacity 1 J/°C is
sectional area A has time period T. When an being heated by keeping it in contact with reservoirs
additional mass M is added to its bob, the time in two ways :
period changes to TM. If the Young's modulus of the (i) Sequentially keeping in contact with 2
reservoirs such that each reservoir supplies
1
material of the wire is Y then is equal to same amount of heat.
Y
(ii) Sequentially keeping in contact with 8
(g = gravitational acceleration) reservoirs such that each reservoir supplies
same amount of heat.
⎡⎛ TM ⎞ 2 ⎤ A ⎡⎛ TM ⎞ 2 ⎤ Mg
(1) ⎢⎜ T ⎟  1⎥ Mg (2) ⎢⎜ T ⎟  1⎥ A In both the cases body is brought from initial
⎢⎣⎝ ⎠ ⎥⎦ ⎢⎣⎝ ⎠ ⎥⎦ temperature 100°C to final temperature 200°C.
Entropy change of the body in the two cases
⎡ ⎛ TM ⎞ 2 ⎤ A ⎡ ⎛ T ⎞2 ⎤ A respectively is
(3) ⎢1  ⎜ T ⎟ ⎥ Mg (4) ⎢1  ⎜ T ⎟ ⎥ Mg (1) ln 2, 4ln 2 (2) ln 2, ln 2
⎢⎣ ⎝ ⎠ ⎥⎦ ⎢⎣ ⎝ M ⎠ ⎥⎦
(3) ln 2, 2ln 2 (4) 2ln 2, 8ln 2
Answer (1) Answer (None)
l dQ dT
Sol. T  2  g ...(1) Sol. ds   ms
T T

l  l dT T 473
TM  2  s  ∫ ds  ms ∫ 1log e 2  log e
g ...(2) T T1 373
11. Consider an ideal gas confined in an isolated closed
Fl Mgl
Y ⇒ l  ...(3) chamber. As the gas undergoes an adiabatic
Al AY expansion, the average time of collision between
2
molecules increases as Vq, where V is the volume of
1 A ⎡⎛ TM ⎞ ⎤ the gas. The value of q is
⇒  ⎢⎜ ⎟  1⎥
Y Mg ⎢⎣⎝ T ⎠ ⎥⎦ ⎛ CP ⎞
⎜  ⎟
9. Consider a spherical shell of radius R at ⎝ Cv ⎠
temperature T. The black body radiation inside it 3  5 3  5
can be considered as an ideal gas of photons with (1) (2)
6 6
U
internal energy per unit volume u   T 4 and 1 1
V (3) (4)
2 2
1 U Answer (3)
pressure P  ⎛⎜ ⎞⎟ . If the shell now undergoes an
3⎝V ⎠  1
Sol.    ...(i)
adiabatic expansion the relation between T and R is 3 RT
2 d 2 ⎛⎜ ⎞⎟
vrms N
(1) T  e–R (2) T  e–3R ⎝V ⎠ M

1 V
(3) T  1 (4) T   ...(ii)
R R3 T
4
TV – 1 = k ...(iii) Answer (2)
 1 ⎡ v ⎤ ⎡ 320 ⎤ 320
  V 2 Sol. f 1  f ⎢ v  v ⎥  f ⎢ 320  20 ⎥  f  300 Hz
⎣ s ⎦ ⎣ ⎦
12. For a simple pendulum, a graph is plotted between ⎡ v ⎤ 320
its kinetic energy (KE) and potential energy (PE) f2  f ⎢ ⎥ f Hz
⎣ v  vs ⎦ 340
against its displacement d. Which one of the
following represents these correctly? ⎛f ⎞ ⎛ f  f1 ⎞
100  ⎜ 2  1 ⎟  ⎜ 2 ⎟  100
(Graphs are schematic and not drawn to scale) ⎝ f1 ⎠ ⎝ f1 ⎠
E ⎡ 300 ⎤
 100 ⎢  1⎥  12%
KE ⎣ 340 ⎦
14. A long cylindrical shell carries positive surface
(1) PE charge  in the upper half and negative surface
d charge – in the lower half. The electric field lines
around the cylinder will look like figure given in
E
(figures are schematic and not drawn to scale)
PE

(2) KE ++++
d + +
–– ––
(1) –– ––
E KE

d +
++
+
++
(3) –– ––
(2) –– ––

PE

E
++ ++
+ +
(3) –– ––
PE –– ––

(4) KE

(4)
Answer (2)
1
Sol. KE  m2 ( A2  d 2 ) Answer (1)
2
1 Sol. The field line should resemble that of a dipole.
PE  m2 d 2
2 15. A uniformly charged solid sphere of radius R has
At d = ± A, potential V0 (measured with respect to ) on its
PE = maximum while KE = 0. surface. For this sphere the equipotential surfaces
3V0 5V0 3V0 V
13. A train is moving on a straight track with speed with potentials , , and 0 have radius
20 ms–1. It is blowing its whistle at the frequency of 2 4 4 4
R1, R2, R3 and R4 respectively. Then
1000 Hz. The percentage change in the frequency
heard by a person standing near the track as the (1) R1 = 0 and R2 > (R4 – R3)
train passes him is (speed of sound = 320 ms–1) (2) R1  0 and (R2 – R1) > (R4 – R3)
close to
(3) R1 = 0 and R2 < (R4 – R3)
(1) 6% (2) 12%
(4) 2R < R4
(3) 18% (4) 24%
5
Answer (3, 4) Charge
Q
Sol. V0  k ...(i) Q2
R
kQ (4)
VI   3R 2  r 2 
2 R3 C
1 F 3 F
3
V  V0 R1 = 0 Answer (2)
2
3C
5 kQ  2 2
 kQ 3R 3 r Sol. C aq = ...(i)
3C
4 R 2R
R ⎛ 3C ⎞
R2  Total charges q  ⎜ ⎟E ...(ii)
 ⎝ 3C ⎠
2
3 kQ kQ Charge upon capacitor 2 F,
 3
4 R R
2 3CE 2CE 2E
4R q'    
 R3  3 (3  C ) 3  C 1  3
3 C
1 kQ kQ

4 R R4
dQ dQ 2
 R4 = 4R  R4 > 2R Now,  0, 2  0
dC dC
16. In the given circuit, charge Q2 on the 2 F capacitor
changes as C is varied from 1 F to 3 F. Q2 as a 17. When 5 V potential difference is applied across a
function of C is given properly by : (Figures are wire of length 0.1 m, the drift speed of electrons is
drawn schematically and are not to scale) 2.5 × 10–4 ms–1. If the electron density in the wire is
8 × 1028 m–3, the resistivity of the material is close to
(1) 1.6 × 10–8 m (2) 1.6 × 10–7 m
1 F
(3) 1.6 × 10–6 m (4) 1.6 × 10–5 m
C
Answer (4)
2 F
l
Sol. V  IR  I 
A
VA VA V
⇒   
E Il ln eA  d l n  e  d
Charge 5
⇒
0.1  2.5  10 19  1.6  10 19  8  10 28
Q2
= 1.6 × 10–5 m
(1) 18. In the circuit shown, the current in the 1  resistor
C is
1 F 3 F
6V 2
P
Charge

Q2
1 9V
(2)
C 3
1 F 3 F 3
Charge Q
(1) 1.3 A, from P to Q
Q2 (2) 0 A
(3) (3) 0.13 A, from Q to P
C (4) 0.13 A, from P to Q
1 F 3 F
6
Answer (3) Answer (2)
Sol. From KVL,

9 = 6I1 – I2 …(1)
6 = 4I2 – I1 …(2) T
Solving, I1 – I2 = –0.13A
Sol.
6V F
I2 P 2
I1
(l)g
I1–I2 Tcos = gl …(1)
3 9V
1
0 I  Il
Tsin = . …
2   2 L sin  
Q
3
19. Two coaxial solenoids of different radii carry gL
 ⇒ I  2 sin 
current I in the same direction. Let F1 be the  0 cos 
magnetic force on the inner solenoid due to the
 21. A rectangular loop of sides 10 cm and 5 cm carrying
outer one and F2 be the magnetic force on the outer a current I of 12 A is placed in different orientations
solenoid due to the inner one. Then as shown in the figures below:
  z
(1) F1 = F2 = 0
  I
(2) F1 is radially inwards and F2 is radially B
outwards I I
(a)
  y
(3) F1 is radially inwards and F2 = 0 I
x
  z
(4) F1 is radially outwards and F2 = 0

Answer (1)
B
Sol. Net force on each of them would be zero.
(b) I
20. Two long current carrying thin wires, both with I y
I
current I, are held by insulating threads of length L x I
and are in equilibrium as shown in the figure, with z
threads making an angle  with the vertical. If wires
have mass  per unit length then the value of I is
I
B
(g = gravitational acceleration) I
(c) I
y
I
x
 z
L

B
(d) I
I I I y
I
x I
gL gL
(1) sin   cos  (2) 2 sin   cos  If there is a uniform magnetic field of 0.3 T in the
0 0

positive z direction, in which orientations the loop


gL gL would be in (i) stable equilibrium and (ii) unstable
(3) 2 tan  (4) tan 
0 0 equilibrium?

7
(1) (a) and (b), respectively 23. A red LED emits light at 0.1 watt uniformly around
it. The amplitude of the electric field of the light at
(2) (a) and (c), respectively a distance of 1 m from the diode is
(1) 1.73 V/m (2) 2.45 V/m
(3) (b) and (d), respectively
(3) 5.48 V/m (4) 7.75 V/m
(4) (b) and (c), respectively Answer (2)

Answer (3) P
Sol. I   U av  c ...(1)
4 r 2
 
Stable equilibrium M||B
1
U av   0 E02 ...(2)
z 2

P 1
⇒   0 E02  c
B 4 r 2 2
I
I y 2P
I ⇒ E0   2.45 V/m
x I 4 r 2  0 c

 
 
24. Monochromatic light is incident on a glass prism of
Unstable equilibrium M|| B
angle A. If the refractive index of the material of the
prism is , a ray, incident at an angle , on the face
z
AB would get transmitted through the face AC of the
prism provided.
B
I A
I y
I
x I 
22. An inductor (L = 0.03 H) and a resistor (R = 0.15
k) are connected in series to a battery of 15 V EMF
in a circuit shown below. The key K1 has been kept B C
closed for a long time. Then at t = 0, K1 is opened
1
⎡ ⎛ 1 ⎛ 1 ⎞ ⎞

and key K 2 is closed simultaneously. At (1)   sin ⎢ sin ⎜ A  sin ⎜  ⎟ ⎟ ⎥
t = 1 ms, the current in the circuit will be ( e  150) ⎣⎢ ⎝ ⎝ ⎠ ⎠ ⎦⎥

0.03 H 0.15 k 1
⎡ ⎛ 1 ⎛ 1 ⎞ ⎞

(2)   sin ⎢ sin ⎜ A  sin ⎜  ⎟ ⎟ ⎥
⎣⎢ ⎝ ⎝ ⎠ ⎠ ⎦⎥
K2 ⎡ ⎛ ⎤
1 1 ⎛ 1 ⎞ ⎞
(3)   cos ⎢ sin ⎜ A  sin ⎜  ⎟ ⎟ ⎥
⎣⎢ ⎝ ⎝ ⎠ ⎠ ⎦⎥

K1 1
⎡ ⎛ 1 ⎛ 1 ⎞ ⎞

15 V (4)   cos ⎢ sin ⎜ A  sin ⎜  ⎟ ⎟ ⎥
⎢⎣ ⎝ ⎝ ⎠ ⎠ ⎥⎦
(1) 100 mA (2) 67 mA
Answer (1)
(3) 6.7 mA (4) 0.67 mA
Answer (4)
t
 L
Sol. I  I 0 e  ,    r
R Sol. 1 r2

1103
15  1/5103
 e  0.67 mA
150
sin  =  sin r1

8
26. Assuming human pupil to have a radius of 0.25 cm
sin  and a comfortable viewing distance of 25 cm, the
 sin r1 =
 minimum separation between two objects that
human eye can resolve at 500 nm wavelength is
1 ⎛ sin  ⎞ (1) 1 m (2) 30 m
 r1 = sin ⎜ ⎟
⎝  ⎠
(3) 100 m (4) 300 m

⎛ sin  ⎞ Answer (2)


r2 = A – sin 1 ⎜ ⎟
⎝  ⎠
1.22  1.22  (500  10 9 m)
Sol. RP  
1 ⎛ 1 ⎞ 2 sin  ⎛ 1 ⎞
 r2 < sin ⎜ ⎟ 2  1 ⎜ ⎟
⎝⎠ ⎝ 100 ⎠


⎛ sin  ⎞ 1 ⎛ 1 ⎞
0.25 cm
A  sin 1 ⎜ ⎟  sin ⎜ ⎟
⎝  ⎠ ⎝⎠
25 cm

⎛1⎞ ⎛ sin  ⎞ = 3.05 × 10–5 m


 A  sin 1 ⎜ ⎟  sin 1 ⎜ ⎟
⎝⎠ ⎝  ⎠ = 30 m
27. As an electron makes a transition from an excited
⎛ 1 ⎛ 1 ⎞ ⎞ sin  state to the ground state of a hydrogen-like atom/ion
 sin ⎜ A  sin ⎜  ⎟ ⎟  
⎝ ⎝ ⎠⎠
(1) Its kinetic energy increases but potential energy
and total energy decrease
⎛ ⎛ 1 ⎛ 1 ⎞ ⎞

  ⎜⎜ sin ⎜ A  sin ⎜  ⎟ ⎟ ⎟⎟  sin 
(2) Kinetic energy, potential energy and total energy
⎝ ⎝ ⎝ ⎠⎠⎠ decrease
(3) Kinetic energy decreases, potential energy
1
⎛ ⎛ 1 ⎛ 1 ⎞ ⎞
⎞ increases but total energy remains same
 sin ⎜⎜  sin ⎜ A  sin ⎜  ⎟ ⎟ ⎟⎟  
⎝ ⎝ ⎝ ⎠⎠⎠ (4) Kinetic energy and total energy decrease but
potential energy increases
25. On a hot summer night, the refractive index of air is
smallest near the ground and increases with height Answer (1)
form the ground. When a light beam is directed
horizontally, the Huygen's principle leads us to z2
Sol. PE  27.2 eV
conclude that as it travels, the light beam n2
(1) Becomes narrower
13.6 z 2
(2) Goes horizontally without any deflection TE   eV
n2
(3) Bends downwards
(4) Bends upwards 13.6 z 2
KE  eV
Answer (4) n2
Sol. Consider a plane wavefront travelling horizontally.
As it moves, its different parts move with different 13.6
KE  eV , As n decreases, KE
speeds. So, its shape will change as shown n2
 Light bends upward
27.2
PE   eV , as n decreases, PE
n2

13.6
TE   eV , as n decreases, TE
n2

9
28. Match List-I (Fundamental Experiment) with List-II R L
(its conclusion) and select the correct option from
the choices given below the list:

List -I List-II
(A) Franck-Hertz (i) Particle nature C
experiment of light
(B) Photo-electric (ii) Discrete energy
experiment levels of atom If a student plots graphs of the square of maximum
(C) Davison-Germer (iii) Wave nature of
experiment electron charge QMax
2
 on the capacitor with time (t) for two
(iv) Structure of different values L1 and L2 (L1 > L2) of L then which
atom
of the following represents this graph correctly?
(1) (A) - (i) (B) - (iv) (C) - (iii) (Plots are schematic and not drawn to scale)

(2) (A) - (ii) (B) - (iv) (C) - (iii)


2
QMax
(3) (A) - (ii) (B) - (i) (C) - (iii) L1
(1)
(4) (A) - (iv) (B) - (iii) (C) - (ii) L2
t
Answer (3)

Sol. Franck-Hertz exp.– Discrete energy level.


2
QMax
Photo-electric effect– Particle nature of light L2

Davison-Germer exp.– Diffraction of electron beam. (2)


L1
t
29. A signal of 5 kHz frequency is amplitude modulated
on a carrier wave of frequency 2 MHz. The
frequencies of the resultant signal is/are 2
QMax
L1
(1) 2 MHz only
(3)
(2) 2005 kHz and 1995 kHz L2
t
(3) 2005 kHz, 2000 kHz and 1995 kHz

(4) 2000 kHz and 1995 kHz


2
QMax
Answer (3) Q0 (For both L1 and L2)

Sol. Frequencies of resultant signal are (4)

t
fe + fm, fe and fe – fm

(2000 + 5) kHz, 2000 kHz, (2000 – 5) kHz,


Answer (1)
2005 kHz, 2000 kHz, 1995 kHz
Sol. For a damped pendulum, A = A0e–bt/2m
30. An LCR circuit is equivalent to a damped
R⎞
pendulum. In an LCR circuit the capacitor is ⎛⎜ ⎟t
 A  A0 e ⎝ 2L ⎠
charged to Q0 and then connected to the L and R as
shown below : (Since L plays the same role as m)

10
PART–B : CHEMISTRY

31. The molecular formula of a commercial resin used –13.6


for exchanging ions in water softening is = = 4 =2
–3.4
C8H 7SO 3Na (mol. wt. 206). What would be the
maximum uptake of Ca2+ ions by the resin when 34. The intermolecular interaction that is dependent on
expressed in mole per gram resin? the inverse cube of distance between the molecules
is
1 1
(1) (2) (1) Ion-ion interaction (2) Ion-dipole interaction
103 206
(3) London force (4) Hydrogen bond
2 1
(3) (4) Answer (4)
309 412
Answer (4) Sol. H-bond is one of the dipole-dipole interaction and
dependent on inverse cube of distance between the
Sol. Ca+2 + 2C8H7SO3–Na+  Ca(C8H7SO3–)2 + 2Na+ molecules.
1 mol 2 mol
35. The following reaction is performed at 298 K.
1 1
The maximum uptake =  mol/g  2NO (g)
2NO(g) + O2(g) 
206  2 412 2

32. Sodium metal crystallizes in a body centred cubic The standard free energy of formation of NO(g) is
86.6 kJ/mol at 298 K. What is the standard free
lattice with a unit cell edge of 4.29 Å. The radius of energy of formation of NO2(g) at 298 K?
sodium atom is approximately (Kp = 1.6 × 1012)
(1) 1.86 Å (2) 3.22 Å (1) R(298) ln(1.6 × 1012) – 86600
(3) 5.72 Å (4) 0.93 Å (2) 86600 + R(298) ln(1.6 × 1012)
Answer (1) 
ln 1.6  10 12 
(3) 86600 
Sol. Edge length of BCC is 4.29 Å. R  298 
In BCC,
(4) 0.5 ⎡⎣ 2  86, 600  R  298  ln1.6  10 ⎤⎦
12

4
edge length = r Answer (4)
3
 2NO (g)
Sol. 2NO(g) + O2(g) 
4 2
4.29  r
3

4.29
 G  
reaction ⎣ 
 ⎡⎢  G 
formation ⎥

⎦ product
r 3  1.86 Å
4  ⎡⎢   G 
 ⎤
⎥ reactant
⎣ formation ⎦
33. Which of the following is the energy of a possible
excited state of hydrogen?
⇒ RT ln K P  2   G    2  G  
(1) +13.6 eV NO 2 NO

(2) –6.8 eV
⇒  G    2  G    RT ln K P
NO 2 NO
(3) –3.4 eV
(4) +6.8 eV
2  86600  R  298  ln K P
Answer (3) ⇒ G 
 NO 2

2
Sol. Energy of excited state is negative and correspond to
n > 1. 2  86600  R  298  ln1.6  10 12

2
–13.6
n=
E excited state  0.5 ⎡⎣ 2  86, 600  R  298  ln 1.6  1012 ⎤⎦

11
36. The vapour pressure of acetone at 20°C is 185 torr. ⎛1⎞
When 1.2 g of a non-volatile substance was C B ⎜ ⎟ 2
2
dissolved in 100 g of acetone at 20°C, its vapour Now Q C  ⎝ ⎠ 2 4
pressure was 183 torr. The molar mass (g mol–1) of A 2 ⎛1⎞
⎜ ⎟
the substance is ⎝2⎠
(1) 32 as QC > KC, hence reaction will shift in backward
direction.
(2) 64
38. Two faraday of electricity is passed through a
(3) 128
solution of CuSO4. The mass of copper deposited at
(4) 488 the cathode is (at. mass of Cu = 63.5 amu)
Answer (2) (1) 0 g
Sol. Vapour pressure of pure acetone P°A  185 torr (2) 63.5 g
(3) 2 g
Vapour pressure of solution, PS = 183 torr
(4) 127 g
Molar mass of solvent, MA = 58 g/mole
Answer (2)
P°A  PS n B Sol. Cu+2 + 2e  Cu
as we know 
PS nA So, 2 F charge deposite 1 mol of Cu. Mass deposited
185  183 WB M A = 63.5 g.
⇒  
183 M B WA 39. Higher order (>3) reactions are rare due to
2 1.2 58 (1) Low probability of simultaneous collision of all
⇒   the reacting species
183 M B 100
(2) Increase in entropy and activation energy as
1.2 58
⇒ MB    183 more molecules are involved
2 100
(3) Shifting of equilibrium towards reactants due to
 63.68 g/mole elastic collisions
(4) Loss of active species on collision
37. The standard Gibbs energy change at 300 K for the
Answer (1)


reaction 2A 
 B + C is 2494.2 J. At a given time,
Sol. Higher order greater than 3 for reaction is rare
the composition of the reaction mixture is because there is low probability of simultaneous
1 1 collision of all the reacting species.
A   , B   2 and  C   . The reaction proceeds 40. 3 g of activated charcoal was added to 50 mL of
2 2
in the : [R = 8.314 J/K/mol, e = 2.718] acetic acid solution (0.06N) in a flask. After an hour
it was filtered and the strength of the filtrate was
(1) Forward direction because Q > KC found to be 0.042 N. The amount of acetic acid
(2) Reverse direction because Q > KC adsorbed (per gram of charcoal) is
(1) 18 mg
(3) Forward direction because Q < KC
(2) 36 mg
(4) Reverse direction because Q < KC
(3) 42 mg
Answer (2)
(4) 54 mg


 B + C, G° = 2494.2 J
Sol. 2A  Answer (1)
Sol. Number of moles of acetic acid adsorbed
As we know G° = –2.303 RT logKC
⎛ 50 50 ⎞
 2494.2 = –2.303 × 8.314 × 300 log KC  ⎜ 0.06   0.042  ⎟
⎝ 1000 1000 ⎠
 –0.434 = log KC
0.9
 KC = anti log (–0.434)  moles
1000
 KC = 0.367  Weight of acetic acid adsorbed = 0.9 × 60 mg
1 1 = 54 mg
Now A   , B   2 and  C  
2 2 Hence, the amount of acetic acid adsorbed per g of

12
54 Answer (2)
charcoal = mg
3 Sol. BeSO4 has hydration energy greater than its lattice
= 18 mg energy.
Hence, option (1) is correct. 45. Which among the following is the most reactive?
41. The ionic radii (in Å) of N 3– , O 2– and F – are (1) Cl2
respectively (2) Br2
(1) 1.36, 1.40 and 1.71 (3) I2
(2) 1.36, 1.71 and 1.40 (4) ICl
(3) 1.71, 1.40 and 1.36 Answer (4)
(4) 1.71, 1.36 and 1.40 Sol. Because of polarity and weak bond interhalogen
compounds are more reactive.
Answer (3)
46. Match the catalysts to the correct processes :
Sol. Radius of N3–, O2– and F– follow order
Catalyst Process
N3– > O2– > F–
a. TiCl3 (i) Wacker process
As per inequality only option (3) is correct
b. PdCl2 (ii) Ziegler-Natta
that is 1.71 Å, 1.40 Å and 1.36 Å polymerization
42. In the context of the Hall-Heroult process for the c. CuCl2 (iii) Contact process
extraction of Al, which of the following statement is
d. V2O5 (iv) Deacon's process
false?
(1) a(iii), b(ii), c(iv), d(i)
(1) CO and CO2 are produced in this process
(2) a(ii), b(i), c(iv), d(iii)
(2) Al2O3 is mixed with CaF2 which lowers the
melting point of the mixture and brings (3) a(ii), b(iii), c(iv), d(i)
conductivity (4) a(iii), b(i), c(ii), d(iv)
(3) Al 3+ is reduced at the cathode to form Al Answer (2)
(4) Na3AlF6 serves as the electrolyte Sol. TiCl3 - Ziegler-Natta polymerisation
Answer (4) V2O5 - Contact process
Sol. In Hall-Heroult process Al2O3 (molten) is electrolyte. PdCl2 - Wacker process
43. From the following statement regarding H 2O 2, CuCl2 - Deacon's process
choose the incorrect statement
47. Which one has the highest boiling point?
(1) It can act only as an oxidizing agent
(1) He
(2) It decomposes on exposure to light (2) Ne
(3) It has to be stored in plastic or wax lined glass (3) Kr
bottles in dark.
(4) Xe
(4) It has to be kept away from dust
Answer (4)
Answer (1) Sol. Down the group strength of van der Waal's force of
Sol. H2O2 can be reduced or oxidised. Hence, it can act attraction increases hence Xe have highest boiling
as reducing as well as oxidising agent. point.

44. Which one of the following alkaline earth metal 48. The number of geometric isomers that can exist for
sulphates has its hydration enthalpy greater than square planar [Pt(Cl)(py)(NH 3 )(NH 2 OH)] + is
its lattice enthalpy? (py = pyridine)

(1) CaSO4 (1) 2

(2) BeSO4 (2) 3

(3) BaSO4 (3) 4

(4) SrSO4 (4) 6

13
Answer (2) 52. Which of the following compounds will exhibit
geometrical isomerism?
a b a c a b
(1) 1 - Phenyl - 2 - butene
Sol. Pt Pt Pt
d c d b c d (2) 3 - Phenyl - 1 - butene
as per question a = Cl, b = py, c = NH 3 and (3) 2 - Phenyl - 1 - butene
d = NH2OH are assumed. (4) 1, 1 - Diphenyl - 1 propane
49. The color of KMnO4 is due to Answer (1)
(1) M  L charge transfer transition Sol. For geometrical isomerism doubly bonded carbon
must be bonded to two different groups which is
(2) d - d transition only satisfied by 1 - Phenyl - 2 - butene.
(3) L  M charge transfer transition H H H CH3
(4)  - * transition C=C C=C
Ph – CH2 CH3 Ph – CH2 H
Answer (3)
cis trans
Sol. Charge transfer spectra from ligand (L) to metal (M)
is responsible for color of KMnO4. 53. Which compound would give 5-keto-2-methyl
hexanal upon ozonolysis?
50. Assertion : Nitrogen and Oxygen are the main
components in the atmosphere but these CH3 CH3
do not react to form oxides of nitrogen.
CH3
Reason : The reaction between nitrogen and (1) (2)
oxygen requires high temperature.
(1) Both assertion and reason are correct, and the CH3
reason is the correct explanation for the
assertion CH3 CH3
(2) Both assertion and reason are correct, but the H3C
reason is not the correct explanation for the (3) (4)
assertion
(3) The assertion is incorrect, but the reason is CH3
correct
Answer (2)
(4) Both the assertion and reason are incorrect
Sol. 5-keto-2-methylhexanal is
Answer (1)
O O
Sol. N2 + O2  2NO
H
Required temperature for above reaction is around
3000°C which is a quite high temperature. This
reaction is observed during thunderstorm. O3 Zn
O O
+
51. In Carius method of estimation of halogens, 250 mg H
H
of an organic compound gave 141 mg of AgBr. The
percentage of bromine in the compound is (At. mass
Ag = 108; Br = 80) 54. The synthesis of alkyl fluorides is best
accomplished by
(1) 24 (2) 36
(1) Free radical fluorination
(3) 48 (4) 60
Answer (1) (2) Sandmeyer's reaction

Sol. Percentage of Br (3) Finkelstein reaction


(4) Swarts reaction
Weight of AgBr Mol. mass of Br
= Mol. mass of AgBr  Weight of O.C.  0 Answer (4)
Sol. Swart's reaction
141 80
=   100 = 24% 
188 250 CH 3  Cl  AgF  CH 3 F  AgCl

14
55. In the following sequence of reactions : 57. Which polymer is used in the manufacture of paints
KMnO SOCl H /Pd
Toluene 
4
 A 
2
 B 
2
BaSO
 C, and lacquers?
4

the product C is (1) Bakelite

(1) C6H5COOH (2) Glyptal


(2) C6H5CH3 (3) Polypropene
(3) C6H5CH2OH
(4) Poly vinyl chloride
(4) C6H5CHO
Answer (2)
Answer (4)
CH3 COOH COCl CHO Sol. Glyptal is used in manufacture of paints and

KMnO4 SOCl2 H2/Pd


lacquires.
Sol. BaSO4
58. Which of the vitamins given below is water soluble?
(A) (B) (C)
56. In the reaction (1) Vitamin C

NH2 (2) Vitamin D

NaNO2/HCl CuCN/KCN (3) Vitamin E


0-5°C
D 
E + N2 ,
(4) Vitamin K
CH3
Answer (1)
the product E is
Sol. Vitamin C is water soluble vitamin.
COOH
59. Which of the following compounds is not an
(1) antacid?

(1) Aluminium Hydroxide


CH3
(2) Cimetidine
(2) H3C CH3 (3) Phenelzine

(4) Ranitidine
CN
Answer (3)
(3)
Sol. Phenelzine is not antacid, it is anti-depressant.
CH3 60. Which of the following compounds is not colored
yellow?
CH3
(1) Zn2[Fe(CN)6]
(4)
(2) K3[Co(NO2)6]

Answer (3) (3) (NH4)3[As (Mo3O10)4]


+ –
NH2 N2Cl CN (4) BaCrO4
NaNO2/HCl CuCN/KCN Answer (1)
Sol.
0°C - 5°C 
+ N2
Sol. (NH4)3[As (Mo3O10)4], BaCrO4 and K3[Co(NO2)6] are
CH3 CH3 CH3
yellow colored compounds but Zn2[Fe(CN)6] is not
(D) (E) yellow colored compound.

15
PART–C : MATHEMATICS

61. Let A and B be two sets containing four and two


63. Let  and  be the roots of equation x2 – 6x – 2 = 0.
elements respectively. Then the number of subsets of
a10 – 2 a8
the set A × B, each having at least three elements is If an = n – n, for n  1, then the value of 2 a9
(1) 219 (2) 256 is equal to
(1) 6 (2) –6
(3) 275 (4) 510
(3) 3 (4) –3
Answer (1) Answer (3)
Sol. n(A) = 4, n(B) = 2 Sol. From equation,
+=6
n(A × B) = 8
 = –2
Required numbers = 8C3 + 8C4 + ...... + 8C8 10 10 8 8
a10 – 2 a8      (   )
The value of
= 28 – (8C0 + 8C1 + 8C2) 2 a9 2( 9  9 )

= 256 – 37 9 (  )  9 (  )

2(9  9 )
= 219
 6
62. A complex number z is said to be unimodular if   3
|z| = 1. Suppose z1 and z2 are complex numbers 2 2

z1  2 z2 ⎡1 2 2 ⎤
such that is unimodular and z 2 is not
2  z1 z2 64. If A = ⎢⎢ 2 1 2 ⎥⎥ is a matrix satisfying the
unimodular. Then the point z1 lies on a ⎢⎣ a 2 b ⎥⎦
(1) Straight line parallel to x-axis equation AAT = 9I, where I is 3 × 3 identity matrix,
then the ordered pair (a, b) is equal to
(2) Straight line parallel to y-axis
(1) (2, –1) (2) (–2, 1)
(3) Circle of radius 2
(3) (2, 1) (4) (–2, –1)
(4) Circle of radius 2
Answer (4)
Answer (3)
⎡1 2 2 ⎤ ⎡ 1 2 a⎤ ⎡9 0 0⎤
⎛ z1  2 z2 ⎞ ⎢ 2 1 2 ⎥ ⎢ 2 1 2 ⎥⎥  ⎢⎢ 0 9 0 ⎥⎥
Sol. ⎜ 2  z z ⎟  1 Sol. ⎢ ⎥⎢
⎝ 1 2 ⎠ ⎢⎣ a 2 b ⎥⎦ ⎢⎣ 2 2 b ⎥⎦ ⎢⎣0 0 9 ⎥⎦

⎛ z1  2 z2 ⎞ ⎛ z1  2 z2 ⎞ a  4  2b  0
⎜ ⎟⎜ ⎟1
⎝ 2  z1 z2 ⎠ ⎝ 2  z1 z2 ⎠ 2 a  2  2b  0

z1 z1  2 z1 z2  2 z2 z1  4 z2 z2 a1b  0
2 a  2 b  2
 4  2 z1 z2  2 z1 z2  z1 z1 z2 z2
a  2 b  4
z1 z1  4 z2 z2  4  z1 z1 z2 z2
3 a  6
zz1  1  z2 z2   4  1  z2 z2   0
a  2
 z1 z1  4  1  z2 z2   0 2  1  b  0
 z1 z1  4 b=–1
a=–2
|z| = 2 i.e. z lies on circle of radius 2.
(–2, –1)
16
65. The set of all values of  for which the system of 5 digit numbers
linear equations
2x1 – 2x2 + x3 = x1
2x1 – 3x2 + 2x3 = x2
5
–x1 + 2x2 = x3
5 × 4 × 3 × 2 × 1 = 120
has a non-trivial solution
Total number of integers = 72 + 120 = 192
(1) Is an empty set
(2) Is a singleton 67. The sum of coefficients of integral powers of x in

 
50
(3) Contains two elements the binomial expansion of 1  2 x is
(4) Contains more than two elements
Answer (3) 1 50 1 50
(1) (3  1) (2) (3 )
Sol. x1 (2   )  2 x2  x3  0 2 2

2 x1  x2 (   3)  2 x3  0 1 50 1 50
(3) (3  1) (4) (2  1)
 x1  2 x2  x3  0 2 2
2 2 1 Answer (1)
2   3 2 0
1  2 x   
50 50 50 1
Sol.  C0  C1 2 x  50 C 2 (2 x )2  .....
1 2 
 50 C 50 ( 2 x )50
(2   )(  2  3  4)  2 ( 2   2)  (4    3)  0
Sum of coefficient of integral power of x
2 3 2
2   6   8    3  4   4   4    1  0 50
 C0 2 0  50 C 2  2 2  50 C 4  2 4    50 C 50  2 50
  3   2  5  3  0
We know that
  3   2  5  3  0
50
(1 + 2)50 = C0  50 C1  2  .....  50 C 50  2 50
 3   2  2  2  2   3  3  0
 2 (   1)  2  (   1)  3(   1)  0 Then,

(   1)(  2  2   3)  0 50 350  1
C0  50 C 2  2 2  .....  50 C 50  2 50 
(   1)(   3)(   1)  0 2
68. If m is the A.M. of two distinct real numbers l and
   1, 1,  3 n (l, n > 1) and G1, G2 and G3 are three geometric
Two elements. means between l and n, then G14  2G24  G34 equals.
66. The number of integers greater than 6,000 that can (1) 4 l2mn (2) 4 lm2n
be formed, using the digits 3, 5, 6, 7 and 8, without
(3) 4 lmn2 (4) 4 l2m2n2
repetition, is
Answer (2)
(1) 216
(2) 192 ln
Sol. m
2
(3) 120
l + n = 2m …(i)
(4) 72
1
Answer (2) ⎛ n⎞ 4
G1  l ⎜ ⎟
Sol. 4 digit numbers ⎝ l⎠
2
3, 5, 6, 7, 8
⎛ n⎞ 4
G2  l ⎜ ⎟
678 ⎝ l⎠
3
⎛ n⎞ 4
3 4 5 2 = 72 G3  l ⎜ ⎟
⎝ l⎠
17
Now G14  2G24  G33 71. If the function.

⎪⎧ k x  1 , 0x3
2 3
n ⎛ n⎞ ⎛ n⎞
l4   2  (l 2 ) ⎜ ⎟  l 4 ⎜ ⎟ g( x )  ⎨
l ⎝ l⎠ ⎝ l⎠ ⎪⎩ mx  2 , 3x5
= nl3 + 2n2l2 + n3l is differentiable, the value of k + m is
= 2n2l2 + nl(n2 + l2)
16
= 2n2l2 + nl((n + l)2 – 2nl) (1) 2 (2)
5
= nl(n + l)2 10
= nl (2m)2 (3) (4) 4
3
=4 nlm2
Answer (1)
69. The sum of first 9 terms of the series
⎧⎪ k x  1 , 0x3
1 3
1 2 3 3
1 2 3 3 3 3 Sol. g( x )  ⎨
   ........ is ⎪⎩ mx  2 , 3x5
1 1 3 1 3 5
R.H.D.
(1) 71 (2) 96 g(3  h )  g(3)
lim
(3) 142 (4) 192 h0 h
Answer (2) m(3  h )  2  2 k
= lim
h0 h
2
⎡ n  n  1 ⎤ (3m  2 k )  mh  2
⎢ ⎥  lim m
Sol. ⎣ 2 ⎦ h0 h
tn 
2
n and 3m – 2k + 2 = 0
L.H.D.
 n  1 2
 k (3  h )  1  2 k
4 lim
h0 h
1
 ⎡⎣n2  2n  1⎤⎦  k[ 4  h  2]
4 lim
h0 h
1 ⎡ n  n  1  2n  1  2  n  n  1  ⎤ lim  k 
4h4

k
 ⎢   1⎥
4⎣ 6 2 ⎦
h0
h( 4  h  2) 4
From above,
1 ⎡ 9  10  19 ⎤
 ⎢  9  10  9 ⎥ k
4⎣ 6 ⎦  m and 3m – 2k + 2 = 0
4
= 96 2 8
m and k 
 1  cos 2 x  3  cos x  5 5
70. lim is equal to
x 0 x tan 4x 8 2 10
km   2
(1) 4 5 5 5
Alternative Answer
(2) 3
(3) 2 ⎪⎧ k x  1 , 0x3
g( x )  ⎨
⎪⎩ mx  2 , 3x5
1
(4) g is constant at x = 3
2
Answer (3) k 4  3m  2
2k = 3m + 2 …(i)
2 sin 2 x   3  cos x  x 2
Sol. lim  =2
tan 4 x x ⎛ k ⎞
x2
x0
 4x Also ⎜⎝ ⎟ m
4x 2 x  1 ⎠ x3
18
k Now, f (x) = 4x + 3a3x2 + 4a4x3
m
4 = x[4 + 3a3x + 4a4x2]
k=4m …(ii) Given, f (1) = 0 and f (2) = 0
8m=3m+2  3a3 + 4a4 + 4 = 0 …(i)
2 8
m ,k and 6a3 + 16a4 + 4 = 0 …(ii)
5 5
2 8 1
m k    2 Solving, a4  , a = –2
5 5 2 3
72. The normal to the curve, x2 + 2xy – 3y2 = 0 at (1,1) 2 3 1 4
i.e., f ( x )  2 x – 2 x  x
(1) Does not meet the curve again 2
(2) Meets the curve again in the second quadrant i.e., f (2)  0
(3) Meets the curve again in the third quadrant dx
(4) Meets the curve again in the fourth quadrant
74. The integral ∫x 2
( x 4  1)3/4
equals

Answer (4) 1
Sol. Curve is x2 + 2xy – 3y2 = 0 ⎛ x4  1⎞ 4 1
(1) ⎜ 4 ⎟  c (2) ( x 4  1) 4  c
⎡ dy ⎤ dy ⎝ x ⎠
Differentiate wr.t. x, 2 x  2 ⎢ x  y ⎥  6y  0
⎣ dx ⎦ dx 1
1
⎛ x4  1⎞ 4
⎛ dy ⎞ (3) ( x  1)  c
4 4
(4)  ⎜ 4 ⎟  c
 ⎜ ⎟ 1 ⎝ x ⎠
⎝ dx ⎠ (1, 1)
Answer (4)
So equation of normal at (1, 1) is
dx dx
Sol. I  ∫
x 2 ( x 4  1)3/4 ∫
y – 1 = – 1 (x – 1)  3/4
⎛ 1⎞
 y=2–x x5 ⎜ 1  4 ⎟
⎝ x ⎠
Solving it with the curve, we get
x2 + 2x(2 – x) – 3(2 – x)2 = 0  4
Let 1  4
 t ⇒ 5 dx  dt
 –4x2 + 16x – 12 = 0 x x
 x2 – 4x + 3 = 0 1 dt 1 3/4
 x = 1, 3
So, I  ∫
4 t 3/4

4 ∫
t dt

So points of intersections are (1, 1) & (3, –1) i.e.


1 ⎛ t 1/4 ⎞
normal cuts the curve again in fourth quadrant.  c
4 ⎜⎝ 1 / 4 ⎟⎠
73. Let f(x) be a polynomial of degree four having extreme
1/4
⎡ f (x) ⎤ ⎛ 1 ⎞
values at x = 1 and x = 2. If lim ⎢ 1  ⎥  3 , then = ⎜1  4 ⎟ c
x 0 ⎣ x2 ⎦ ⎝ x ⎠
f(2) is equal to So, option (4).
(1) –8 (2) –4
4
log x 2
(3) 0 (4) 4
75. The integral ∫ 2
 log(36 – 12 x  x 2 )
dx is equal
Answer (3) 2 log x
to
Sol. Let f(x) = a0 + a1x + a2x2 + a3x3 + a4x4
(1) 2 (2) 4
⎡ f (x) ⎤
Using lim ⎢ 1  2 ⎥  3 (3) 1 (4) 6
x 0 ⎣ x ⎦
f (x) Answer (3)
 lim 2  2
x 0 x 4
log x 2 dx
a  a x  a2 x 2  a3 x 3  a4 x 4 Sol. I  ∫ 2
lim 0 1 2 2 log x  log(36 – 12 x  x 2 )

x 0 x2 4
So, a0 = 0, a1 = 0, a2 = 2 log(6 – x )2 dx
I∫
2
i.e., f(x) = 2x2 + a3x3 + a4x4 2 log x  log(6 – x )2

19
4 Answer (3)*
2 I  ∫ 1 dx
Sol. It is best option. Theoretically question is wrong,
2
because initial condition is not given.
2I = 2
I=1 dy
x log x + y = 2x logx If x = 1 then y = 0
76. The area (in sq. units) of the region described by dx
{(x, y) : y2  2x and y  4x – 1} is dy y
 2
7 5 dx x log x
(1) (2)
32 64 1
∫ x log x dx
I.F.  e  e log log x  log x
15 9
(3) (4)
64 32 Solution is y  log x  ∫ 2 log x dx  c
Answer (4)
y log x  2( x log x  x )  c
y=1 1 y=1
Sol. x = 1, y = 0
Then, c = 2, y(e) = 2

1 78. The number of points, having both co-ordinates as


integers, that lie in the interior of the triangle with
2
vertices (0, 0), (0, 41) and (41, 0), is
(1) 901 (2) 861
(3) 820 (4) 780
After solving y = 4x – 1 and y2 = 2x
Answer (4)
2
y
y  4 1
2 Sol. (0, 41)
2y2 –y–1=0

1 A39
1 18 13
y  y  1,
4 4 2
1 1 2
⎛ y1⎞ y A2 B2
A ∫ ⎜⎝ 4 ⎟⎠ dy  ∫ 2 dy
1/2 1/2 B1
A1
1 3 ⎤1
1 ⎡ y2 ⎤ 1 ⎡y (0, 0) (41, 0)
 ⎢  y⎥  ⎢ ⎥
4 ⎢⎣ 2 ⎥⎦ 1/2 2 ⎢⎣ 3 ⎥⎦ 1/2

1 ⎡ 4  8  1  4 ⎤ 1 ⎡8  1⎤ Total number of integral coordinates as required


 ⎥  2 ⎢ 24 ⎥
4 ⎢⎣ 8 ⎦ ⎣ ⎦ = 39 + 38 + 37 + ....... + 1
1 ⎡ 15 ⎤ 9 39  40
    780
4 ⎢⎣ 8 ⎥⎦ 48 2

15 6 9 79. Locus of the image of the point (2, 3) in the line


=  
32 32 32 (2x – 3y + 4) + k(x – 2y + 3) = 0, k  R, is a
77. Let y(x) be the solution of the differential equation (1) Straight line parallel to x-axis
dy
 x log x   y  2 x log x , ( x  1). (2) Straight line parallel to y-axis
dx
Then y(e) is equal to (3) Circle of radius 2
(1) e (2) 0
(3) 2 (4) 2e (4) Circle of radius 3

20
Answer (3) 82. Let O be the vertex and Q be any point on the
Sol. After solving equation (i) & (ii) parabola, x2 = 8y. If the point P divides the line
segment OQ internally in the ratio 1 : 3, then the
2x – 3y + 4 = 0 ...(i) locus of P is
2x – 4y + 6 = 0 ...(ii)
(1) x 2  y (2) y 2  x
x = 1 and y = 2
A (3) y 2  2 x (4) x 2  2 y
Slope of AB × Slope of MN = – 1
(2, 3)
b3 Answer (4)
2 ⎛ a  2 b  3⎞
b3 M⎜ ,
 2  1 ⎝ 2 2 ⎠

Sol. x2 = 8y
a2 a2 N
1 (1, 2)
2 Let Q be (4t, 2t2)
(y – 3)(y – 1) = –(x – 2)x y
(a, b) ⎛ t2 ⎞ 2
y2 – 4y + 3 = –x2 + 2x  P  ⎜ t, Q(4t, 2t )
B (Image of A) ⎝ 2 ⎟⎠ 3
x2 + y2 – 2x – 4y + 3 = 0 P
1
Let P be (h, k) x
Circle of radius = 2 O
80. The number of common tangents to the circles t2
 h = t, k 
x2 + y2 – 4x – 6y – 12 = 0 and 2
x2 + y2 + 6x + 18y + 26 = 0, is
 2k  h 2
(1) 1 (2) 2
(3) 3 (4) 4  Locus of (h, k) is x2 = 2y.
Answer (3) 83. The distance of the point (1, 0, 2) from the point of
Sol. x2 + y2 – 4x – 6y – 12 = 0 x2 y1 z2
intersection of the line   and the
3 4 12
C1(center) = (2, 3), r = 2 2  32  12  5
plane x – y + z = 16, is
x2 + y2 + 6x + 18y + 26 = 0
C2(center) (– 3, –9), r  9  81  26 (1) 2 14 (2) 8

 64  8 (3) 3 21 (4) 13
C1C2 = 13, C1C2 = r1 + r2 Answer (4)
Number of common tangent is 3.
x2 y1 z2
81. The area (in sq. units) of the quadrilateral formed by Sol.   
the tangents at the end points of the latera recta to 3 4 12

x2 y2 P  3  2, 4  1, 12   2 
the ellipse   1 , is
9 5 Lies on plane x – y + z = 16
27 Then,
(1) (2) 18
4
3  2  4   1  12   2  16
27
(3) (4) 27 11  5  16
2
Answer (4)
1 P  5, 3, 14
2 2
x y
Sol. Ellipse is  1 Distance = 16  9  144  169  13
9 5
i.e., a2 = 9, b2 = 5 84. The equation of the plane containing the line
2 2x – 5y + z = 3; x + y + 4z = 5, and parallel to the
So, e 
3 plane, x + 3y + 6z = 1, is
2 (1) 2 x  6 y  12 z  13 (2) x  3y  6 z  7
As, required area  2a  2  9  27
e (2/3)
(3) x  3y  6 z  7 (4) 2 x  6 y  12 z  13

21
Answer (3) Answer (1)*

Sol. Required plane is Sol. Question is wrong but the best suitable option is (1).
11
(2x – 5y + z – 3) + (x + y + 4z – 5) = 0 12 29 55 ⎛ 2 ⎞
Required probability = C3 = ⎜ ⎟
It is parallel to x + 3y + 6z = 1 312 3 ⎝3⎠
87. The mean of the data set comprising of 16
2   5   1  4 
   observations is 16. If one of the observation valued
1 3 6 16 is deleted and three new observations valued 3,
4 and 5 are added to the data, then the mean of the
11 resultant data, is
Solving  =
2 (1) 16.8
 Required plane is (2) 16.0
(3) 15.8
11
(2x – 5y + z – 3) – (x + y + 4z – 5) = 0 (4) 14.0
2
Answer (4)
 x + 3y + 6z – 7 = 0
Sol. Mean = 16

85. Let a , b and c be three non-zero vectors such that Sum = 16 × 16 = 256
no two of them are collinear and
New sum = 256 – 16 + 3 + 4 + 5 = 252
   1   
( a  b )  c  |b ||c | a . If  is the angle between 252
3 Mean = = 14
 18
vectors b and c , then a value of sin  is
88. If the angles of elevation of the top of a tower from
three collinear points A, B and C, on a line leading
2 2  2 to the foot of the tower, are 30º, 45º and 60º
(1) (2)
3 3 respectively, then the ratio, AB : BC, is

2 2 3 (1) 3 : 1
(3) (4)
3 3
(2) 3 : 2
Answer (1)
(3) 1 : 3
      1   
Sol. ( a  c ) b  (b  c ) a  |b ||c | a (4) 2 : 3
3
Answer (1)
  1   P
  (b  c )  |b ||c | Sol. AO = h cot30º
3
h 3
1
 cos    h
3 BO = h

2 2 h
CO  30º 45º 60º
 sin   3
3 A B C O
86. If 12 identical balls are to be placed in 3 identical AB AO  BO
boxes, then the probability that one the boxes  
BC BO  CO
contains exactly 3 balls is
11 10
55 ⎛ 2 ⎞ 2
(1) ⎜ ⎟ (2) 55 ⎛⎜ ⎞⎟ 
h 3 h
3 ⎝3⎠ ⎝3⎠ h
h
12 11 3
⎛1⎞ ⎛1⎞
(3) 220 ⎜ ⎟ (4) 22 ⎜ ⎟
⎝3⎠ ⎝3⎠
 3
22
⎛ 2x ⎞ 3x  x 3
89. Let tan 1 y  tan 1 x  tan 1 ⎜ y
⎝ 1  x 2 ⎟⎠ 1  3x 2
1 90. The negation of ~ s  (~ r s) is equivalent to
where |x| . Then a value of y is
3 (1) s  ~ r
3x  x 3 3x  x 3 (2) s  (r  ~ s)
(1) (2)
1  3x 2 1  3x 2
(3) s  (r  ~ s)
3x  x 3 3x  x 3
(3) (4) (4) s  r
1  3x 2 1  3x 2
Answer (4)
Answer (1)
Sol. ∼ ( ∼ s  ( ∼ r  s ))
1 1 ⎛ 2x ⎞ 1
Sol. tan y  tan x  tan ⎜
⎝ 1  x 2 ⎟⎠ = s  (r  ∼ s )

1 ⎛ 3x  x ⎞
3
= (s  r )  (s  ∼ s )
3tan–1 x = tan ⎜ 2 ⎟
⎝ 1  3x ⎠ = s r

  

23
Answers & Solutions For JEE MAIN- 2014
(Code-E)

Time Durations : 3 hrs. Maximum Marks: 360

(Physics, Chemistry and Mathematics)

Important Instructions :

1. The test is of 3 hours duration.


2. The Test Booklet consists of 90 questions. The maximum marks are 360.
3. There are three parts in the question paper A, B, C consisting of Physics,
Chemistry and Mathematics having 30 questions in each part of equal weightage. Each
question is allotted 4 (four) marks for each correct response.
4. Candidates will be awarded marks as stated above in Instructions No. 3 for correct response
of each question. ¼ (one-fourth) marks will be deducted for indicating incorrect response
of each question. No deduction from the total score will be made if no response is
indicated for an item in the answer sheet.
5. There is only one correct response for each question. Filling up more than one response
in each question will be treated as wrong response and marks for wrong response will be
deducted accordingly as per instruction 4 above.
6. Use Blue/Black Ball Point Pen only for writing particulars/marking responses on Side-1 and
Side-2 of the Answer Sheet. Use of pencil is strictly prohibited.
7. No candidate is allowed to carry any textual material, printed or written, bits of papers,
pager, mobile phone, any electronic device, etc. except the Admit Card inside the
examination hall/room.
8. The CODE for this Booklet is E. Make sure that the CODE printed on Side-2 of the Answer
Sheet is the same as that on this booklet. In case of discrepancy, the candidate should
immediately report the matter to the Invigilator for replacement of both the Test Booklet and
the Answer Sheet.

www.vedantu.com
PART–A : PHYSICS

1. The current voltage relation of diode is given by 2g g


I = (e1000V/T – 1) mA, where the applied V is in volts (1) (2)
3 2
and the temperature T is in degree kelvin. If a
student makes an error measuring ± 0.01 V while 5g
(3) (4) g
measuring the current of 5 mA at 300 K, what will 6
be the error in the value of current in mA? Answer (2)
(1) 0.2 mA (2) 0.02 mA Sol. a = Rα α
(3) 0.5 mA (4) 0.05 mA
mg – T = ma R
Answer (1)
T×R= mR2α
Sol. I = (e1000 V/T – 1)mA T
or T = ma
When I = 5 mA, e1000 V/T = 6 mA T
1000 V /T 1000 g m a
Also, dI = ( e )× ⋅ dV ⇒ a=
T 2 mg
1000 4. A block of mass m is placed on a surface with a
= (6 mA) × × (0.01)
300 x3
= 0.2 mA vertical cross-section given by y = . If the
6
2. From a tower of height H, a particle is thrown coefficient of friction is 0.5, the maximum height
vertically upwards with a speed u. The time taken above the ground at which the block can be placed
by the particle, to hit the ground, is n times that without slipping is
taken by it to reach the highest point of its path. The
1 2
relation between H, u and n is: (1) m (2) m
6 3
(1) 2gH = n2u2 (2) gH = (n – 2)2u2
(3) 2gH = nu2 (n – 2) (4) gH = (n – 2)u2 1 1
(3) m (4) m
Answer (3) 3 2
Answer (1)
u
Sol. Time taken to reach highest point is t1 =
g dy x 2
Sol. tanθ = =
Speed on reaching ground = u2 + 2 gh dx 2
Now, v = u + at At limiting equilibrium,
m
u μ = tanθ
⇒ 2
u + 2 gh = –u + gt y
θ
2
x
u + u2 + 2 gH nu H 0.5 =
⇒ t= = 2
g g
⇒ x = ±1
⇒ 2 gH = n(n – 2)u2 2
u + 2gH
3. A mass m is supported by a massless string wound 1
Now, y =
around a uniform hollow cylinder of mass m and 6
radius R. If the string does not slip on the cylinder, 5. When a rubber-band is stretched by a distance x, it
with what acceleration will the mass fall on release? exerts a restoring force of magnitude F = ax + bx2
where a and b are constants. The work done in
stretching the unstretched rubber-band by L is :
R 1 2
m (1) aL2 + bL3 (2) ( aL + bL3 )
2

aL2 bL3 1 ⎛ aL2 bL3 ⎞


+ +
2 ⎜⎝ 2 3 ⎟⎠
(3) (4)
2 3
m
Answer (3)

(2)

www.vedantu.com
∫ dW = ∫ F ⋅ dl
8. The pressure that has to be applied to the ends of a
Sol.
steel wire of length 10 cm to keep its length constant
L 2 3
L when its temperature is raised by 100°C is :
W= ∫ bx 2 dx = aL + bL .
ax dx + ∫
0 0 2 3 (For steel Young's modulus is 2 × 1011 Nm–2 and
6. A bob of mass m attached to an inextensible string coefficient of thermal expansion is 1.1 × 10–5 K–1)
of length l is suspended from a vertical support. (1) 2.2 × 108 Pa (2) 2.2 × 109 Pa
The bob rotates in a horizontal circle with an
angular speed ω rad/s about the vertical. About the (3) 2.2 × 107 Pa (4) 2.2 × 106 Pa
point of suspension Answer (1)
(1) Angular momentum is conserved Sol. As length is constant,
(2) Angular momentum changes in magnitude but
not in direction ΔL
Strain = = αΔQ
(3) Angular momentum changes in direction but L
not in magnitude Now pressure = stress = Y × strain
(4) Angular momentum changes both in direction
= 2 × 1011 × 1.1 × 10–5 × 100
and magnitude
= 2.2 × 108 Pa
Answer (3)
Sol. τ = mg × l sin θ. (Direction parallel to plane of 9. There is a circular tube in a vertical plane. Two
rotation of particle) liquids which do not mix and of densities d1 and d2
are filled in the tube. Each liquid subtends 90° angle
ω at centre. Radius joining their interface makes an
d1
θ angle α with vertical. Ratio d is
2
l l


as τ is perpendicular to L , direction of L changes d2
but magnitude remains same. α
7. Four particles, each of mass M and equidistant from
each other, move along a circle of radius R under d1
the action of their mutual gravitational attraction.
The speed of each particle is 1 sin α 1 + cos α
(2)
1 – sin α 1 – cos α
GM GM
(1) (2) 2 2 1 + tan α 1 + sin α
R R (3) (4)
1 – tan α 1 – cos α
GM 1 GM
(3) (1 + 2 2 ) (4) (1 + 2 2 ) Answer (3)
R 2 R
Sol. Equating pressure at A
Answer (4)

F F Mv 2
Sol. + + F′ = M F M
2 2 R R
F′ d2 R sinα
v R α
2 × GM 2 GM 2 Mv 2 F
2
+ 2
= O α R
2( R 2 ) 4R R α Rcosα
(Rcosα – Rsinα)
2 M M
GM ⎡ 1 1 ⎤
⎢ + ⎥ = Mv 2 d1 A
R ⎣4 2⎦ (Rcosα + Rsinα)d2g = (Rcosα – Rsinα)d1g
Gm ⎛ 2 + 4 ⎞ 1 Gm
v= ⎜ ⎟ =
R ⎜⎝ 4 2 ⎟⎠ 2 R
1+ 2 2 ( ) ⇒
d1 cos α + sin α 1 + tan α
= =
d2 cos α – sin α 1 – tan α

(3)

www.vedantu.com
10. On heating water, bubbles being formed at the Answer (3)
bottom of the vessel detatch and rise. Take the Sol. 100°C
bubbles to be spheres of radius R and making a
circular contact of radius r with the bottom of the
Cu
vessel. If r << R, and the surface tension of water is
T, value of r just before bubbles detatch is T
(Density of water is ρw) B Brass
Steel
0°C
0°C
Q = Q1 + Q2
0.92 × 4(100 − T ) 0.26 × 4 × (T − 0) 0.12 × 4 × T
R = +
46 13 12
⇒ 200 – 2T = 2T + T
2r ⇒ T = 40°C
2 ρw g 2 ρw g 0.92 × 4 × 60
(1) R (2) R ⇒ Q= = 4.8 cal/s
3T 6T 46
ρw g 3ρw g 12. One mole of diatomic ideal gas undergoes a cyclic
2 2
(3) R (4) R process ABC as shown in figure. The process BC is
T T
adiabatic. The temperatures at A, B and C are 400 K,
Answer (No answer) 800 K and 600 K respectively. Choose the correct
Sol. When the bubble gets detached, statement
Buoyant force = force due to surface tension B
800 K

P
600 K
A C
R θ 400 K
V
r θ (1) The change in internal energy in whole cyclic
T × dl process is 250R
4 3 (2) The change in internal energy in the process CA
∫ T × dl sin θ = 3 πR ρw g is 700R
r 4 3 (3) The change in internal energy in the process AB
⇒ T × 2 πr × = πR ρ w g
R 3 is –350R
(4) The change in internal energy in the process BC
2 2 R 4 ρw g
⇒ r = is – 500R
3
Answer (4)
2 2ρ w g
⇒ r=R 5R
3T Sol. ΔU = nCV ΔT = 1 × ΔT
2
11. Three rods of copper, brass and steel are welded For BC, ΔT = –200 K
together to form a Y-shaped structure. Area of cross-
⇒ ΔU = –500R
seciton of each rod = 4 cm2. End of copper rod is
maintained at 100°C whereas ends of brass and 13. An open glass tube is immersed in mercury in such
steel are kept at 0°C. Lengths of the copper, brass a way that a length of 8 cm extends above the
and steel rods are 46, 13 and 12 cm respectively. mercury level. The open end of the tube is then
The rods are thermally insulated from surroundings closed and sealed and the tube is raised vertically
except at ends. Thermal conductivities of copper, up by additional 46 cm. What will be length of the
brass and steel are 0.92, 0.26 and 0.12 CGS units air column above mercury in the tube now?
respectively. Rate of heat flow through copper rod is (Atmospheric pressure = 76 cm of Hg)
(1) 1.2 cal/s (2) 2.4 cal/s (1) 16 cm (2) 22 cm
(3) 4.8 cal/s (4) 6.0 cal/s (3) 38 cm (4) 6 cm

(4)

www.vedantu.com
Answer (1) 15. A pipe of length 85 cm is closed from one end. Find
Sol. the number of possible natural oscillations of air
column in the pipe whose frequencies lie below
(54 – x) 1250 Hz. The velocity of sound in air is 340 m/s.
54 cm P
8 cm (1) 12 (2) 8
x
(3) 6 (4) 4
Answer (3)
(2 n − 1)v
Sol. f = ≤ 1250
4L
(2 n − 1) × 340
P + x = P0 ⇒ ≤ 1250
0.85 × 4
P = (76 – x)
⇒ 2n – 1 ≤ 12.5
8 × A × 76 = (76 – x) × A × (54 – x)
∴ Answer is 6.
x = 38

Length of air column = 54 – 38 = 16 cm. 16. Assume that an electric field E = 30 x 2 iˆ exists in
14. A particle moves with simple harmonic motion in a space. Then the potential difference VA – VO, where
straight line. In first τ s, after starting from rest it VO is the potential at the origin and VA the potential
travels a distance a, and in next τ s it travels 2a in at x = 2 m is
same direction then (1) 120 J (2) –120 J
(1) Amplitude of motion is 3a (3) –80 J (4) 80 J
(2) Time period of oscillations is 8τ Answer (3)
(3) Amplitude of motion is 4a  
Sol. dV = −E ⋅ dx
(4) Time period of oscillations is 6τ VA 2
Answer (4) ∫ dV = − ∫ 30 x 2 dx
VO 0
Sol. As it starts from rest, we have
x = Acosωt. At t = 0, x = A VA − VO = −[10 x 3 ]02 = −80 J
when t = τ, x = A – a 17. A parallel plate capacitor is made of two circular
when t = 2τ, x = A – 3a plates separated by a distance 5 mm and with a
⇒ A – a = Acosωτ dielectric of dielectric constant 2.2 between them.
When the electric field in the dielectric is 3 × 104
A – 3a = Acos2ωτ V/m, the charge density of the positive plate will be
As cos2ωτ = 2cos2ωτ – 1 close to
2 (1) 6 × 10–7 C/m2 (2) 3 × 10–7 C/m2
A – 3a ⎛ A– a ⎞
⇒ = 2⎜ ⎟ –1
A ⎝ A ⎠ (3) 3 × 104 C/m2 (4) 6 × 104 C/m2

A – 3a 2 A2 + 2 a 2 – 4 Aa – A2 Answer (1)
=
A A2 σ
A2 – 3aA = A2 + 2a2 – 4Aa Sol. E =
Kε0
a2 = 2aA σ = Kε0 E
A = 2a = 2.2 × 8.85 ×10–12 × 3 ×104 ≈ 6 × 10–7 C/m2
Now, A – a = Acosωτ 18. In a large building, there are 15 bulbs of 40 W,
5 bulbs of 100 W, 5 fans of 80 W and 1 heater of
1
⇒ cos ωτ = 1 kW. The voltage of the electric mains is 220 V. The
2 minimum capacity of the main fuse of the building
2π π will be :
τ=
T 3 (1) 8 A (2) 10 A
⇒ T = 6τ (3) 12 A (4) 14 A

(5)

www.vedantu.com
Answer (3) Answer (3)
Sol. 15 × 40 + 5 × 100 + 5 × 80 + 1000 = V × I
Sol. B = μ0 n i
600 + 500 + 400 + 1000 = 220 I
B
2500 = ni
I= = 11.36 μ0
220
I = 12 A. NI 100 × i
3 × 10 3 = =
19. A conductor lies along the z-axis at –1.5 ≤ z < 1.5 m L 10 × 10 −2
and carries a fixed current of 10.0 A in – aˆ z direction I = 3 A.
 −4 –0.2 x
(see figure). For a field B = 3.0 × 10 e aˆ y T,
21. In the circuit shown here, the point 'C' is kept
find the power required to move the conductor at
constant speed to x = 2.0 m, y = 0 m in 5 × 10–3 s. connected to point 'A' till the current flowing through
Assume parallel motion along the x-axis the circuit becomes constant. Afterward, suddenly,
z point 'C' is disconnected from point 'A' and connected
1.5 to point 'B' at time t = 0. Ratio of the voltage across
I
resistance and the inductor at t = L/R will be equal
y to
B
2.0
A C R
x –1.5

B L
(1) 1.57 W (2) 2.97 W
(3) 14.85 W (4) 29.7 W
Answer (2) e
(1) (2) 1
work 1– e
Sol. Average Power =
time 1– e
2 (3) –1 (4)
e
W = ∫0 Fdx
2
Answer (3)
= ∫0 3.0 × 10–4 e–0.2 x × 10 × 3 dx
Sol. Applying Kirchhoff's law in closed loop, –VR – VC = 0
2
= 9 × 10–3 e–0.2 x
∫ dx ⇒ VR/VC = –1
0
–4 –0.2x
9 × 10–3 ⎡ –0.2×2 B = 3.0 × 10 e Note : The sense of voltage drop has not been
= –e + 1⎤⎦
0.2 ⎣ defined. The answer could have been 1.
l=3m 22. During the propagation of electromagnetic waves in
9 × 10–3 ⎡ z
= × ⎣1 – e–0.4 ⎤⎦ I = 10 A a medium
0.2
x (1) Electric energy density is double of the
= 9 × 10–3 × (0.33)
magnetic energy density
= 2.97 × 10–3 J
(2) Electric energy density is half of the magnetic
2.97 × 10–3 energy density
P= = 2.97 W
(0.2) × 5 × 10–3
(3) Electric energy density is equal to the magnetic
20. The coercivity of a small magnet where the energy density
ferromagnet gets demagnetized is 3 × 103 A m–1.
The current required to be passed in a solenoid of (4) Both electric and magnetic energy densities are
length 10 cm and number of turns 100, so that the zero
magnet gets demagnetized when inside the
solenoid, is Answer (3)

(1) 30 mA (2) 60 mA Sol. Energy is equally divided between electric and


(3) 3 A (4) 6 A magnetic field

(6)

www.vedantu.com
⎛ 3⎞ 25. Two beams, A and B, of plane polarized light with
23. A thin convex lens made from crown glass ⎜ μ = ⎟ mutually perpendicular planes of polarization are
⎝ 2⎠
seen through a polaroid. From the position when the
has focal length f . When it is measured in two
beam A has maximum intensity (and beam B has
4 5 zero intensity), a rotation of polaroid through 30°
different liquids having refractive indices and ,
3 3 makes the two beams appear equally bright. If the
initial intensities of the two beams are IA and IB
it has the focal lengths f 1 and f 2 respectively. The
IA
correct relation between the focal lengths is respectively, then I equals
B
(1) f1 = f2 < f (1) 3
(2) f 1 > f and f 2 becomes negative 3
(2)
2
(3) f 2 > f and f 1 becomes negative
(3) 1
(4) f 1 and f 2 both become negative
1
(4)
Answer (2) 3
Sol. By Lens maker's formula Answer (4)

1 ⎛ 3/2 ⎞⎛ 1 1 ⎞ Sol. By law of Malus, I = I0cos2θ


=⎜ – 1⎟⎜ – ⎟
f 1 ⎝ 4/3 ⎠ ⎝ R1 R2 ⎠ Now, IA′ = IAcos230

1 ⎛ 3/2 ⎞⎛ 1 1 ⎞ IB′ = IBcos260


=⎜ – 1⎟⎜ – ⎟
f 2 ⎝ 5/3 ⎠ ⎝ R1 R2 ⎠ As IA′ = IB′

1 ⎛ 3 ⎞⎛ 1 1 ⎞ ⇒ 3 1
= ⎜ – 1⎟⎜ – ⎟ IA × = IB ×
f ⎝ 2 ⎠ ⎝ R1 R2 ⎠ 4 4
⇒ f1 = 4f & f2 = –5f IA 1
=
24. A green light is incident from the water to the air - IB 3
water interface at the critical angle(θ). Select the 26. The radiation corresponding to 3→2 transition of
correct statement hydrogen atoms falls on a metal surface to produce
(1) The entire spectrum of visible light will come photoelectrons. These electrons are made to enter a
out of the water at an angle of 90° to the normal magnetic field of 3 × 10–4 T. If the radius of the
largest circular path followed by these electrons is
(2) The spectrum of visible light whose frequency
is less than that of green light will come out to 10.0 mm, the work function of the metal is close to
the air medium (1) 1.8 eV
(3) The spectrum of visible light whose frequency (2) 1.1 eV
is more than that of green light will come out to
the air medium (3) 0.8 eV
(4) The entire spectrum of visible light will come (4) 1.6 eV
out of the water at various angles to the normal
Answer (2)
Answer (2)
mv
1 Sol. r =
Sol. sin θc = µ
qB
air 2m eV
=
eB
θc Water
1 2m
= V
B e
For greater wavelength (i.e. lesser frequency) μ is less
So, θ c would be more. So, they will not suffer B2 r 2 e
⇒ V= = 0.8 V
reflection and come out at angles less then 90°. 2m

(7)

www.vedantu.com
For transition between 3 to 2, 29. Match List-I (Electromagnetic wave type) with
⎛1 1⎞ List - II (Its association/application) and select the
E = 13.6 ⎜ − ⎟
⎝4 9⎠ correct option from the choices given below the lists :

13.6 × 5
= = 1.88 eV
36
(a) (i)
Work function = 1.88 eV – 0.8 eV
= 1.08 eV = 1.1 eV (b) (ii)
27. Hydrogen (1H1), Deuterium (1H2), singly ionised (c) (iii)
Helium (2He4)+ and doubly ionised lithium (3Li6)++
all have one electron around the nucleus. Consider (d) (iv)
an electron transition from n = 2 to n = 1. If the wave
lengths of emitted radiation are λ1, λ2, λ3 and λ4
respectively then approximately which one of the
(a) (b) (c) (d)
following is correct?
(1) (iv) (iii) (ii) (i)
(1) 4λ1 = 2λ2 = 2λ3 = λ4
(2) (i) (ii) (iv) (iii)
(2) λ1 = 2λ2 = 2λ3 = λ4
(3) (iii) (ii) (i) (iv)
(3) λ1 = λ2 = 4λ3 = 9λ4
(4) (i) (ii) (iii) (iv)
(4) λ1 = 2λ2 = 3λ3 = 4λ4
Answer (4)
Answer (3)
Sol. (a) Infrared rays are used to treat muscular strain
1 2
⎡1 1⎤ (b) Radiowaves are used for broadcasting
Sol. λ = RZ ⎢ 2 – 2 ⎥
⎣⎢ n1 n ⎦⎥ (c) X-rays are used to detect fracture of bones

(d) Ultraviolet rays are absorbed by ozone


1
⇒ λ∝ for given n1 & n2 30. A student measured the length of a rod and wrote
Z2
it as 3.50 cm. Which instrument did he use to
⇒ λ1 = λ2 = 4λ3 = 9λ4 measure it?

28. The forward biased diode connection is (1) A meter scale

(2) A vernier calliper where the 10 divisions in


(1) +2 V –2 V
vernier scale matches with 9 division in main
scale and main scale has 10 divisions in 1 cm
(2) –3 V –3 V
(3) A screw gauge having 100 divisions in the
circular scale and pitch as 1 mm
(3) 2 V 4V
(4) A screw gauge having 50 divisions in the
(4) –2 V +2 V circular scale and pitch as 1 mm

Answer (1) Answer (2)

Sol. As measured value is 3.50 cm, the least count must


be 0.01 cm = 0.1 mm
Sol.
For vernier scale with 1 MSD = 1 mm and
p n 9 MSD = 10 VSD,

For forward Bias, p-side must be at higher potential Least count = 1 MSD – 1 VSD
than n-side. = 0.1 mm

(8)

www.vedantu.com
PART–B : CHEMISTRY

31. The correct set of four quantum numbers for the Answer (3)
valence electrons of rubidium atom (Z = 37) is Sol.
1 1
(1) 5 , 0 , 0 , + (2) 5 , 1, 0 , + –
Cl Cl—
2 2 —
Cl
1 1 Cl–
(3) 5 , 1, 1, + (4) 5 , 0 , 1, +
2 2 +
Answer (1) Cs
Sol. 37 → 1s22s22p63s23p63d104s24p65s1

So last electron enters 5s orbital Cl –
Cl
— Cl
Cl
1
Hence n = 5, l = 0, ml = 0, ms = ±
2 2rCl − + 2rCs+ = 3 a
32. If Z is a compressibility factor, van der Waals
equation at low pressure can be written as 3a
rCl − + rCs+ =
2
RT a
(1) Z = 1 + (2) Z = 1 – 34. For the estimation of nitrogen, 1.4 g of an organic
Pb VRT compound was digested by Kjeldahl method and the
Pb Pb M
(3) Z = 1 – (4) Z = 1 + evolved ammonia was absorbed in 60 mL of
RT RT 10
Answer (2) sulphuric acid. The unreacted acid required 20 mL
M
PV of sodium hydroxide for complete
Sol. Compressibility factor (Z) = 10
RT neutralization. The percentage of nitrogen in the
(For one mole of real gas) compound is
van der Waal equation (1) 6% (2) 10%
a (3) 3% (4) 5%
(P + )(V − b) = RT
V2 Answer (2)
At low pressure
Sol. As per question
V−b≈ V
Normality Volume
⎛ a ⎞
⎜P + 2 ⎟ V = RT N
⎝ V ⎠ H 2 SO 4 60mL
5
a N
PV + = RT NaOH 20mL
V 10
a (n geq )H2 SO 4 = (n geq )NaOH + (n geq )NH3
PV = RT −
V
1 60 1 20
× = × + (n geq )NH3
PV a 5 1000 10 1000
= 1−
RT VRT 6 1
= + (n geq )NH3
a 500 500
So, Z = 1 − 5 1
VRT
(n geq )NH3 = =
33. CsCl crystallises in body centred cubic lattice. If ‘a’ 500 100
is its edge length then which of the following 1
(n mol )N = (n mol )NH3 = (ngeq )NH3 =
expressions is correct? 100
3a 14
(1) rCs+ + rCl − = 3a (2) rCs+ + rCl − = (Mass)N = = 0.14 g
2 100
3 0.14
(3) r + + r − = a (4) rCs+ + rCl − = 3a Percentage of "N" = × 100 = 10%
Cs Cl 2 1.4

(9)

www.vedantu.com
35. Resistance of 0.2 M solution of an electrolyte is Answer (1)
50 Ω. The specific conductance of the solution is
Sol. C2H5OH(l) + 3O2(g) → 2CO2(g) + 3H2O(l)
1.4 S m–1. The resistance of 0.5 M solution of the
same electrolyte is 280 Ω. The molar conductivity of Bomb calorimeter gives ΔU of the reaction
0.5 M solution of the electrolyte in S m2 mol–1 is
So, as per question
(1) 5 × 10–4 (2) 5 × 10–3
(3) 5 × 103 (4) 5 × 102 ΔU = –1364.47 kJ mol–1
Answer (1) Δng = –1
Sol. For 0.2 M solution ΔH = ΔU + ΔngRT
R = 50 Ω
1 × 8.314 × 298
σ = 1.4 S m–1 = 1.4 × 10–2 S cm–1 = −1364.47 −
1000
1 1 = –1366.93 kJ mol–1
⇒ ρ= = Ω cm
σ 1.4 × 10 −2
37. The equivalent conductance of NaCl at
Now, R = ρ
l concentration C and at infinite dilution are λC and
a λ∞, respectively. The correct relationship between λC
and λ∞ is given as
l R
⇒ = = 50 × 1.4 × 10 −2 (Where the constant B is positive)
a ρ
For 0.5 M solution (1) λ C = λ ∞ + (B)C (2) λ C = λ ∞ – (B)C
R = 280 Ω
σ=? (3) λ C = λ ∞ – (B) C (4) λ C = λ ∞ + (B) C

l Answer (3)
= 50 × 1.4 × 10 −2
a Sol. According to Debye Huckle onsager equation,
l
⇒ R =ρ λC = λ∞ − A C
a
Here A = B
1 1 l
⇒ = ×
ρ R a ∴ λC = λ∞ − B C
1 38. Consider separate solutions of 0.500 M C2H5OH(aq),
⇒ σ= × 50 × 1.4 × 10 −2
280 0.100 M Mg3(PO4)2(aq), 0.250 M KBr(aq) and 0.125
M Na3PO 4(aq) at 25°C. Which statement is true
1
= × 70 × 10 −2 about these solutions, assuming all salts to be strong
280 electrolytes?
= 2.5 × 10–3 S cm–1
(1) They all have the same osmotic pressure.
σ× 1000
Now, λ m = (2) 0.100 M Mg3(PO4)2(aq) has the highest osmotic
M pressure.
2.5 × 10 −3 × 1000 (3) 0.125 M Na3PO4(aq) has the highest osmotic
=
0.5 pressure.
= 5 S cm2 mol–1 (4) 0.500 M C2H5OH(aq) has the highest osmotic
= 5 × 10–4 S m2 mol–1 pressure.
36. For complete combustion of ethanol, Answer (1)
C2H5OH(l) + 3O2(g) → 2CO2(g) + 3H2O(l), Sol. π = i CRT
the amount of heat produced as measured in bomb
calorimeter, is 1364.47 kJ mol–1 at 25°C. Assuming πC2 H5OH = 1 × 0.500 × R × T = 0.5 RT
ideality the enthalpy of combustion, ΔcH, for the
πMg 3 (PO 4 )2 = 5 × 0.100 × R × T = 0.5 RT
reaction will be
(R = 8.314 kJ mol–1) π KBr = 2 × 0.250 × R × T = 0.5 RT
(1) –1366.95 kJ mol–1 (2) –1361.95 kJ mol–1
πNa3PO4 = 4 × 0.125 × RT = 0.5 RT
(3) –1460.50 kJ mol–1 (4) –1350.50 kJ mol–1

(10)

www.vedantu.com
1 1.2 × 10–3 = k [0.1]x[0.2]y ...(ii)
39. For the reaction SO 2(g ) + O 2(g ) SO 3(g) , 2.4 × 10–3 = k [0.2]x[0.1]y ...(iii)
2
x
if K P = K C(RT) where the symbols have usual Dividing equation (i) by (ii)
meaning then the value of x is (assuming ideality)
1.2 × 10 −3 k[0.1]x [0.1]y
⇒ =
1 1.2 × 10 −3 k[0.1]x [0.2]y
(1) –1 (2) −
2 y
⎡1⎤
1 ⇒ 1=⎢ ⎥
(3) (4) 1 ⎣2⎦
2
Answer (2) ⇒ y=0
Now Dividing equation (i) by (iii)
1
Sol. SO 2 (g) + O 2 (g)  SO 3 (g) 1.2 × 10 −3 k[0.1]x [0.1]y
2 =
⇒ 2.4 × 10 −3
KP = KC(RT)x k[0.2]x [0.1]y
1 x
x = Δn g = no. of gaseous moles in product ⎡1⎤ ⎡1⎤
⇒ ⎢⎣ 2 ⎥⎦ = ⎢⎣ 2 ⎥⎦
– no. of gaseous moles in reactant
⇒ x=1
⎛ 1⎞ 3 −1
= 1−⎜1+ ⎟ = 1− = d[C]
⎝ 2⎠ 2 2 Hence = k[A]1 [B]0 .
dt
40. For the non-stoichiometre reaction 2A + B → C + D, 41. Among the following oxoacids, the correct
the following kinetic data were obtained in three decreasing order of acid strength is
separate experiments, all at 298 K.
(1) HOCl > HClO2 > HClO3 > HClO4
Initial Initial Initial rate of
(2) HClO4 > HOCl > HClO2 > HClO3
Concentration Concentration formation of C
(3) HClO4 > HClO3 > HClO2 > HOCl
(A) (B) (mol L–1 s–1 )
(4) HClO2 > HClO4 > HClO3 > HOCl
0.1 M 0.1 M 1.2 × 10–3
Answer (3)
0.1 M 0.2 M 1.2 × 10–3
0.2 M 0.1 M  ClO −4 + H+
Sol. HClO 4 
2.4 × 10–3
The rate law for the formation of C is  ClO 3− + H+
HClO 3 
dC dC
(1) = k[A] [B] (2) = k[A]2 [B]  ClO −2 + H+
HClO 2 
dt dt
dC dC  ClO − + H+
(3) = k[A] [B]2 (4) = k[A] HOCl 
dt dt
Answer (4) Resonance produced conjugate base.

Sol. 2A + B ⎯⎯ –
→C + D O O O

Cl Cl Cl
−1 d[A] d[B] (i) O O

O O

O O
Rate of Reaction = =−
2 dt dt O O O

(ClO4 )
O
d[C] d[D]
= =
dt dt Cl
O O

Let rate of Reaction = k[A]x[B]y O

d[C] O O O
Or, = k[A]x [B]y
dt
Cl –
Cl Cl
Now from table, (ii) O O O

O O O
1.2 × 10–3 =k [0.1]x[0.1]y ...(i) –
(ClO3 )

(11)

www.vedantu.com

So energy order is
O O
Red < Yellow < Green < Blue
Cl Cl
(iii) –
O O The complex absorbs lower energy light lower will
– be its strength. So order of ligand strength is
(ClO ) 2

L1 < L3 < L2 < L4


(iv) ClO– is not resonance stabilized.
44. Which one of the following properties is not shown
As per resonance stability order of conjugate base is by NO?
ClO −4 > ClO 3− > ClO 2− > ClO − (1) It is diamagnetic in gaseous state

Hence acidic strength order is (2) It is a neutral oxide


(3) It combines with oxygen to form nitrogen
HClO 4 > HClO 3 > HClO 2 > HClO
dioxide
42. The metal that cannot be obtained by electrolysis of
(4) It's bond order is 2.5
an aqueous solution of its salts is
(1) Ag (2) Ca Answer (1)

(3) Cu (4) Cr Sol. Nitric oxide is paramagnetic in the gaseous state as


it has one unpaired electron in its outermost shell.
Answer (2)
The electronic configuration of NO is
Sol. On electrolysis only in case of Ca2+ salt aqueous 2 2 1
2 * σ2 σ* σ2 π2 2 *
solution H2 gas discharge at Cathode. σ 1s σ 1s 2 s 2 s 2 pz 2 px = π 2 p y π 2 px

Case of Cr However, it dimerises at low temperature to become


At cathode : Cr3+ + 2e– ⎯→ Cr diamagnetic.

So, Cr is deposited. 
2NO  N2 O2
Case of Ag Its bond order is 2.5 and it combines with O2 to give
At cathode : Ag+ + e– ⎯→ Ag nitrogen dioxide.
So, Ag is deposited.
Case of Cu 45. In which of the following reactions H2O2 acts as a
At cathode : Cu2+ + 2e– ⎯→ Cu reducing agent?

Case of Ca2+ (a) H2O2 + 2H+ + 2e– → 2H2O


(b) H2O2 – 2e– → O2 + 2H+
1
At cathode : H2O + e– ⎯→ H2 + OH– (c) H2O2 + 2e– → 2OH–
2
43. The octahedral complex of a metal ion M3+ with four (d) H2O2 + 2OH– – 2e– → O2 + 2H2O
monodentate ligands L 1, L 2 , L 3 and L 4 absorb (1) (a), (b) (2) (c), (d)
wavelengths in the region of red, green, yellow and
blue, respectively. The increasing order of ligand (3) (a), (c) (4) (b), (d)
strength of the four ligands is Answer (4)
(1) L4 < L3 < L2 < L1 (2) L1 < L3 < L2 < L4 Sol. The reducing agent oxidises itself.
(3) L3 < L2 < L4 < L1 (4) L1 < L2 < L4 < L3
(a) H 2 O −21 + 2H + + 2e − ⎯⎯
→ 2H 2 O −2
Answer (2)
0
Sol. (b) H 2 O 2−1 − 2e ⎯⎯
→ O 2 + 2H +
B
−2
V G (c) H O −1 + 2e ⎯⎯
2 2 → 2OH −
0
R Y (d) H 2 O −2 1 + 2OH − − 2e ⎯⎯
→ O 2 + H2 O
O
Note : Powers of 'O' are oxidation number of 'O' in
The energy of red light is less than that of violet
the compound.
light.
(12)

www.vedantu.com
46. The correct statement for the molecule, CsI3, is But ΔG°3 = −2FE°
(1) It is a covalent molecule
⇒ 5.38F = –2FE°
(2) It contains Cs+ and I –3 ions
⇒ E° = –2.69 V
(3) It contains Cs3+ and I– ions
As E° value is negative reaction is non spontaneous.
(4) It contains Cs+, I– and lattice I2 molecule
49. Which series of reactions correctly represents
Answer (2) chemical reactions related to iron and its
Sol. It contains Cs+ and I3– ions. compound?
47. The ratio of masses of oxygen and nitrogen in a dil.H 2 SO 4
(1) Fe ⎯⎯⎯⎯⎯ H 2 SO 4 ,O 2
→ FeSO 4 ⎯⎯⎯⎯⎯ →
particular gaseous mixture is 1 : 4. The ratio of
heat
number of their molecule is Fe 2 (SO 4 )3 ⎯⎯⎯ → Fe
(1) 1 : 4 (2) 7 : 32 O 2 ,heat
(2) Fe ⎯⎯⎯⎯ dil.H 2 SO 4
→ FeO ⎯⎯⎯⎯⎯ →
(3) 1 : 8 (4) 3 : 16 heat
FeSO 4 ⎯⎯⎯ → Fe
Answer (2)
Cl 2 ,heat heat, air
Sol. Let the mass of O2 = x (3) Fe ⎯⎯⎯⎯ → FeCl 3 ⎯⎯⎯⎯→
Zn
Mass of N2 = 4x FeCl 2 ⎯⎯→ Fe
x O 2 ,heat
(4) Fe ⎯⎯⎯⎯ CO,600ºC
→ Fe 3 O 4 ⎯⎯⎯⎯⎯ →
Number of moles of O2 =
32 CO ,700ºC
FeO ⎯⎯⎯⎯⎯ → Fe
4x x
Number of moles of N2 = = Answer (4)
28 7
O2
x x Sol. Fe ⎯⎯⎯
Heat
→ Fe 3 O 4
∴ Ratio = : = 7 : 32
32 7 This reaction is corresponding to the combustion of
48. Given below are the half-cell reactions Fe.
Mn2+ + 2e– → Mn; E° = — 1.18 V CO
Fe O ⎯⎯⎯ CO
→ FeO ⎯⎯⎯ → Fe
600ºC 700ºC
(Mn3+ + e– → Mn2+); E° = + 1.51 V
These reactions correspond to the production of Fe
The E° for 3 Mn2+ → Mn + 2Mn3+ will be by reduction of Fe3O4 in blast furnace.
(1) –2.69 V; the reaction will not occur 50. The equation which is balanced and represents the
(2) –2.69 V; the reaction will occur correct product(s) is
(3) –0.33 V; the reaction will not occur (1) Li2O + 2KCl → 2LiCl + K2O
(4) –0.33 V; the reaction will occur (2) [CoCl(NH3)5]+ + 5H+ → Co2+ + 5NH4+ + Cl—
Answer (1) excess NaOH
(3) [Mg(H 2 O) 6 ] 2+ + (EDTA) 4— ⎯⎯⎯⎯⎯⎯

Sol. (1) Mn2+ + 2e → Mn; E° = –1.18V ; [Mg(EDTA)]2+ + 6H2O
(4) CuSO4 + 4KCN → K2[Cu(CN)4] + K2SO4
ΔG°1 = −2F( −1.18) = 2.36F
Answer (2)
(2) Mn3+ + e → Mn2+ ; E° = +1.51 V;
Sol. The complex
ΔG°2 = −F(1.51) = −1.51 F [CoCl(NH3)5]+ decomposes under acidic medium, so
[CoCl(NH3)5]+ + 5H+ → Co2+ + 5NH4+ + Cl—.
(1) – 2 × (2)
51. In SN2 reactions, the correct order of reactivity for the
3Mn2+ → Mn + 2Mn3+ ; following compounds
CH3Cl, CH3CH2Cl, (CH3)2CHCl and (CH3)3CCl is
ΔG°3 = ΔG°1 − 2 ΔG°2
(1) CH3Cl > (CH3)2CHCl > CH3CH2Cl > (CH3)3CCl
= [2.36 – 2(–1.51)] F (2) CH3Cl > CH3CH2Cl > (CH3)2CHCl > (CH3)3CCl
= (2.36 + 3.02) F (3) CH3CH2Cl > CH3Cl > (CH3)2CHCl > (CH3)3CCl
= 5.38 F (4) (CH3)2CHCl > CH3CH2Cl > CH3Cl > (CH3)3CCl

(13)

www.vedantu.com
Answer (2) The major product C would be
Sol. Rate of SN2 reaction depends on steric crowding of
OCOCH3 OH
alkyl halide. So order is
CH3Cl > (CH3)CH2 – Cl > (CH3)2CH – Cl > (CH3)3CCl COOH COCH3
(1) (2)
52. On heating an aliphatic primary amine with
chloroform and ethanolic potassium hydroxide, the
COCH3
organic compound formed is OH OCOCH3
(1) An alkanol
COOCH3
(2) An alkanediol (3) (4)
COOH
(3) An alkyl cyanide
(4) An alkyl isocyanide Answer (1)

O O O
Answer (4) H O—
— O=C=O
CHCl /KOH
Sol. R – CH2 – NH2 ⎯⎯⎯⎯⎯⎯3 → R– CH – NC Sol. C
C 2 H 5 OH 2
O
53. The most suitable reagent for the conversion of
R – CH2 – OH → R – CHO is OH OH

COOH COO
(1) KMnO4 (CH3CO)2O H +

(2) K2Cr2O7
(B)
(3) CrO3
OCOCH3
(4) PCC (Pyridinium Chlorochromate) COOH
Answer (4)
Sol. PCC is mild oxidising agent, it will convert (C)
R − CH 2 − OH ⎯⎯
→ R − CHO 56. Considering the basic strength of amines in aqueous
54. The major organic compound formed by the solution, which one has the smallest pKb value?
reaction of 1, 1, 1-trichloroethane with silver powder (1) (CH3)2NH
is (2) CH3NH2
(1) Acetylene (3) (CH3)3N
(2) Ethene (4) C6H5NH2
(3) 2-Butyne Answer (1)
(4) 2-Butene Sol. Among C6H5NH2, CH3NH2, (CH3)2NH,
Answer (3) (CH3)3N . C6H5NH2 is least basic due to resonance.
+ +
Cl NH2 NH2 NH2
Ag
Sol. 2Cl—C—CH3 CH3C ≡ CCH3 + 6AgCl
Cl
1, 1, 1-trichloroethane
+
NH2 NH2
55. Sodium phenoxide when heated with CO2 under
pressure at 125°C yields a product which on
acetylation produces C.

— ONa 125° H
+ Out of (CH3)3N, CH3NH2, (CH3)2NH . (CH3)2NH is
+ CO2 B C most basic due to +I effect and hydrogen bonding in
5 Atm Ac2O
H2O.

(14)

www.vedantu.com
+I effect
CH3 HOOC COOH + HO—CH2—CH2—OH

H3C H —CO CH2—CH2—O—


+I effect
n
O Dacron
Hydrogen H H Acrylonitrile, Neoprene and Teflon are addition
bonding polymers of acrylonitrile, isoprene and tetrafluoro
57. For which of the following molecule significant ethylene respectively.
μ ≠ 0?
59. Which one of the following bases is not present in
Cl CN DNA?
(1) Quinoline
(a) (b)
(2) Adenine
Cl CN (3) Cytosine
(4) Thymine
OH SH
Answer (1)
(c) (d)
Sol. DNA contains ATGC bases

OH SH A – Adenine

(1) Only (a)] (2) (a) and (b) T – Thymine


(3) Only (c) (4) (c) and (d) G – Guanine
Answer (4) C – Cytocine

Cl CN So quinoline is not present.


60. In the reaction,
Sol. (a) (b) LiAlH PCl
Alc.KOH
CH 3 COOH ⎯⎯⎯⎯
4
→ A ⎯⎯⎯
5
→ B ⎯⎯⎯⎯ → C,
Cl CN the product C is
μ=0 μ=0
(1) Acetaldehyde
H H (2) Acetylene
O S
(3) Ethylene
(4) Acetyl chloride
(c) (d)
H Answer (3)
O S H
Sol. Ethylene
μ≠0 μ≠0 LiAlH4
CH3COOH CH3CH2OH 'A'
58. Which one is classified as a condensation polymer?
(1) Dacron (2) Neoprene PCl5

(3) Teflon (4) Acrylonitrile


CH3CH2Cl 'B'
Answer (1)
Alc. KOH
Sol. Dacron is polyester formed by condensation
polymerisation of terephthalic acid and ethylene CH2 = CH2 'C'
glycol.

(15)

www.vedantu.com
PART–C : MATHEMATICS

61. If X = {4n – 3n – 1 : n ∈ N} and Y = {9(n – 1) : n ∈ N}, 63. If a ∈ R and the equation


where N is the set of natural numbers, then X ∪ Y is
equal to –3(x – [x])2 + 2 (x – [x]) + a2 = 0

(1) X (2) Y (where [x] denotes the greatest integer ≤ x) has no


(3) N (4) Y – X integral solution, then all possible values of a lie in
the interval
Answer (2)
(1) (–2, –1)
Sol. X = {(1 + 3)n − 3n − 1, n ∈ N }
(2) (–∞, – 2) ∪ (2, ∞)
= 32 ( n C 2 + nC 3 .3 + ... + 3n − 2 ), n ∈N }
(3) (–1, 0) ∪ (0, 1)
= {Divisible by 9}
(4) (1, 2)
Y = {9(n – 1), n ∈ N}
= (All multiples of 9} Answer (3)

So, X ⊆ Y Sol. –3(x – [x])2 + 2[x – [x]) + a2 = 0


3 {x}2 – 2{x} – a2 = 0
i.e., X ∪ Y = Y
62. If z is a complex number such that |z| ≥ 2, then ⎛ 2 ⎞
a ≠ 0, 3 ⎜ { x} 2 − { x} ⎟ = a2
⎝ 3 ⎠
1
the minimum value of z + ⎛ 1⎞ 1
2
2 a 2 = 3 ⎜ { x} − ⎟ −
⎝ 3⎠ 3
5
(1) Is strictly greater than 1 1 2
2 0 ≤ { x} < 1 and − ≤ { x} − <
3 3 3
3 5
(2) Is strictly greater than but less than 2
2 2 ⎛ 1⎞ 4
0 ≤ 3 ⎜ { x} − ⎟ <
5 ⎝ 3⎠ 3
(3) Is equal to
2 1 ⎛ 1⎞ 1
2

(4) Lies in the interval (1, 2) − ≤ 3 ⎜ { x} − ⎟ − < 1


3 ⎝ 3⎠ 3
Answer (4)
For non-integral solution
Sol.
0 < a2 < 1 and a ∈(–1, 0) ∪ (0, 1)
Alternative
–3{x}2 + 2{x} + a2 = 0
1 Now, –3{x}2 + 2{x}

2

1
z+
2
1
2/3
1 1
So, | z| − ≤ z+ 1
2 2
1 1
⇒ z+ ≥ 2−
2 2
to have no integral roots 0 < a2 < 1
3
⇒ zmin. = ∴ a∈(–1, 0) ∪ (0, 1)
2
(16)

www.vedantu.com
64. Let α and β be the roots of equation Answer (1)
px2 + qx + r = 0, p ≠ 0. If p, q, r are in A.P. and
1 1 1+ 1+ 1 1+ α +β 1 + α2 + β2
+ = 4 , then the value of |α – β| is
α β Sol. 1+ α +β 1 + α2 + β2 1 + α3 + β3
34 2 13 1 + α2 + β2 1 + α3 + β3 1+ α4 + β4
(1) (2) 2
9 9 1 1 1 1 α α
61 2 17 = α β 1 × 1 β β2
(3) (4)
9 9 α2 β2 1 1 1 1
Answer (2) = [(1 – α)(1 – β)(1 – β)]2
Sol. So, k = 1
∵ p, q, r are in AP 66. If A is an 3 × 3 non-singular matrix such that
2q = p + r ...(i)
AA′ = A′A and B = A−1 A′ , then BB′ equals
1 1
Also + = 4 (1) B–1 (2) ( B−1 )′
α β
α +β (3) I + B (4) I
⇒ =4
αβ
Answer (4)
q

p Sol. BB ' = ( A−1 .A ')( A( A−1 )')
= = 4 ⇒ q = –4r
r ...(ii)
= A–1.A.A'.(A–1)1 {as AA' = A'A}
p
= I(A–1A)'
From (i)
= I.I = I2 = I
2(–4r) = p + r
67. If the coefficients of x3 and x4 in the expansion of
p = – 9r
(1 + ax + bx2) (1 – 2x)18 in powers of x are both zero,
q = – 4r then (a, b) is equal to
r=r
⎛ 272 ⎞ ⎛ 272 ⎞
Now |α – β|= (α + β)2 – 4αβ (1) ⎜ 14, ⎟ (2) ⎜ 16, ⎟
⎝ 3 ⎠ ⎝ 3 ⎠
2
⎛ –q ⎞ 4r
= ⎜ ⎟ – ⎛ 251 ⎞ ⎛ 251 ⎞
⎝ p ⎠ p (3) ⎜ 16, ⎟ (4) ⎜ 14, ⎟
⎝ 3 ⎠ ⎝ 3 ⎠
q 2 – 4 pr Answer (2)
=
| p| Sol. (1 + ax + bx2) (1 – 2x)18
16r 2 + 36r 2 (1 + ax + bx2)[18C0 – 18C (2x) + 18C (2x)2 –
= 1 2
|–9r | 18C (2x)3 + 18C (2x)4 – .......]
3 4

2 13 Coeff. of x3 = –18C3.8 + a × 4.18C2 – 2b × 18 = 0


=
9
18 × 17 × 16 4 a + 18 × 17
65. If α, β ≠ 0, and f(n) = αn + βn and = − .8 + − 36b = 0
6 2
3 1 + f (1) 1 + f (2)
= –51 × 16 × 8 + a × 36 × 17 – 36b = 0
1 + f (1) 1 + f (2) 1 + f (3)
= –34 × 16 + 51a – 3b = 0
1 + f (2) 1 + f (3) 1 + f (4)
= 51a – 3b = 34 × 16 = 544
= K(1 – α)2 (1 – β)2 (α – β)2, then K is equal to
= 51a – 3b = 544 ... (i)
(1) 1 (2) –1
Only option number (2) satisfies the equation
1
(3) αβ (4) number (i).
αβ
(17)

www.vedantu.com
68. If (10)9 + 2(11)1 (10)8 + 3(11)2 (10)7+ ... + 10(11)9 = sin( π cos 2 x )
k(10)9, then k is equal to 70. lim is equal to
x →0 x2
(1) 100 (2) 110 (1) –π (2) π

121 441 π
(3) (4) (3) (4) 1
10 100 2
Answer (1) Answer (2)
Sol. 109 + 2⋅(11)(10)8 + 3(11)2(10)7 +... + 10(11)9 = k(10)9 sin( π cos 2 x )
Sol. lim
x= 109 + 2⋅(11)(10)8 + 3(11)2(10)7+ ... +10(11)9 x →0 x2
11
x = 11⋅108 + 2⋅(11)2⋅(10)7 +... + 9(11)9 + 1110 sin( π(1 – sin 2 x )
= lim
10 x →0 x2
( π – π sin 2 x )
= lim sin
⎛ 11 ⎞ x →0 x2
x⎜1 − ⎟ = 109 + 11(10)8 + 112×(10)7 +... +119 – 1110
⎝ 10 ⎠ ( π sin 2 x )
= lim sin [∵ sin( π − θ) = sin θ]
⎛ ⎛ 11 ⎞10 ⎞ x →0 x2
⎜⎜ ⎟ − 1⎟
x 10 ( π sin 2 x ) π sin 2 x
⇒ − = 109 ⎜ ⎝ ⎠ ⎟ − 1110 = lim sin ×
10 ⎜ 11 ⎟ x →0 ( π sin 2 x ) x2
⎜⎜ −1 ⎟
10 ⎟
⎝ ⎠ 2
⎛ sin x ⎞
x = lim 1 × π ⎜ ⎟ =π
⇒ − = (1110 − 1010 ) − 1110 = − 10 10 x →0 ⎝ x ⎠
10
⇒ x = 1011 = k⋅109 1
71. If g is the inverse of a function f and f '( x ) = ,
⇒ k = 100 1 + x5
then g′(x) is equal to
69. Three positive numbers form an increasing G.P. If
the middle term in this G.P. is doubled, the new 1
(2) 1 + {g(x)}5
1 + { g( x )}
(1) 5
numbers are in A.P. Then the common ratio of the
G.P. is
(3) 1 + x5 (4) 5x 4
(1) 2 − 3 (2) 2 + 3 Answer (2)
(3) 2+ 3 (4) 3 + 2 1
Sol. f '( x ) = = f ( g( x )) = x → f '( g( x )) g '( x ) = 1
Answer (2) 1 + x5

Sol. a, ar, ar2 → G.P. 1


g '( x ) = = 1 + ( g( x ))5
f '( g( x ))
a, 2ar, ar2 → A.P.
72. If f and g are differentiable functions in [0, 1]
2 × 2ar = a + ar2
satisfying f(0) = 2 = g(1), g(0) = 0 and f(1) = 6, then
4r = 1 + r2 for some c ∈]0, 1[
⇒ r2 – 4r + 1 = 0 (1) f ′(c) = g′(c) (2) f ′(c) = 2g′(c)
(3) 2f ′(c) = g′(c) (4) 2f ′(c) = 3g′(c)
4 ± 16 − 4
r= =2± 3
2 Answer (2)
Sol. Using, mean value theorem
r =2+ 3
f (1) − f (0)
f '(c ) = =4
1−0
r = 2 − 3 is rejected
g(1) − g(0)
∵ (r > 1) g '(c ) = =2
1−0
G.P. is increasing.
so, f '(c ) = 2 g '(c )

(18)

www.vedantu.com
73. If x = –1 and x = 2 are extreme points of Answer (2)
2
f ( x ) = α log|x|+βx + x then π
x x
Sol. ∫ 1 + 4 sin 2 − 4 sin dx
0
2 2
1 1
(1) α = 2, β = − (2) α = 2, β =
2 2 ⎡ x 1 ⎤
⎢sin 2 = 2 ⎥
π ⎢ ⎥
1 1 x ⎢⇒ x = π → x = π ⎥
(3) α = −6, β = (4) α = −6, β = − = ∫ 2 sin − 1 dx
2 2 2 ⎢ 2 6 3⎥
0 ⎢ x 5π
⎢ = 5π ⎥⎥
Answer (1) →x=
⎣⎢ 2 6 3 ⎦⎥
Sol. f ( x ) = α log|x|+βx 2 + x π/3 π
⎛ x⎞ ⎛ x ⎞
= ∫ ⎜ 1 − 2 sin ⎟ dx + ∫
⎝ 2⎠
⎜ 2 sin − 1 ⎟ dx
⎝ 2 ⎠
α 0 π/3
f ′( x ) = + 2β x + 1 = 0 at x = –1, 2
x x⎤
π /3
x
π
⎡ ⎡ ⎤
= ⎢ x + 4 cos ⎥ + ⎢ −4 cos − x ⎥
−α − 2β + 1 = 0 ⇒ α + 2β = 1 ...(i) ⎣ 2 ⎦0 ⎣ 2 ⎦ π/3

α π 3 ⎛ 3 π⎞
+ 4β + 1 = 0 ⇒ α + 8β = −2 ...(ii) = +4 − 4 + ⎜⎜ 0 − π + 4 + ⎟⎟
2 3 2 ⎝ 2 3⎠

1 π
6β = −3 ⇒ β = − = 4 3−4−
2 3
∴ α=2 76. The area of the region described by
A = {(x, y) : x2 + y2 ≤ 1 and y2 ≤ 1 – x} is
1
⎛ 1 ⎞ x+ π 2 π 2
74. The integral ∫ ⎜ 1 + x − ⎟ e x dx is equal to (1) − (2) +
⎝ x⎠ 2 3 2 3

1 1 π 4 π 4
(1) ( x + 1) e x + x + c (2) − x e x + x + c (3) + (4) −
2 3 2 3

1 1 Answer (3)
(3) ( x − 1) e x + x + c (4) x e x + x + c Sol.

Answer (4)

⎧ ⎛⎜ x + 1 ⎞⎟ 1
1⎫
⎪ ⎝ x⎠ ⎛ ⎞ x+ x ⎪
Sol. I = ∫ ⎨e + x⎜1− 2
⎝ x


e ⎬ dx
⎪⎩ ⎪⎭
2 2
1 x +y =1
x+
= x.e x +c
Shaded area
As ∫ ( xf '( x ) + f ( x ))dx = xf ( x ) + c
1
π(1)2
75. The integral =
2
+2 ∫0
(1 − x ) dx

π
x x 1
∫ 1 + 4 sin 2 − 4 sin dx equals π 2(1 − x )3/2
0
2 2 = + ( −1)
2 3/2
0

π π 4
(1) 4 3 − 4 (2) 4 3 − 4 − = + (0 − ( −1))
3 2 3


π 4
−4−4 3 = +
(3) π – 4 (4) 2 3
3

(19)

www.vedantu.com
77. Let the population of rabbits surviving at a time t be 79. Let a, b, c and d be non-zero numbers. If the point of
governed by the differential equation intersection of the lines 4ax + 2ay + c = 0 and
5bx + 2by + d = 0 lies in the fourth quadrant and is
dp(t ) 1
= p(t ) − 200 . If p(0) = 100, then p(t) equals equidistant from the two axes then
dt 2
(1) 3bc – 2ad = 0
(1) 600 – 500 et/2
(2) 3bc + 2ad = 0
(2) 400 – 300 e–t/2
(3) 400 – 300 et/2 (3) 2bc – 3ad = 0

(4) 300 – 200 e–t/2 (4) 2bc + 3ad = 0


Answer (3) Answer (1)
dp(t ) 1 Sol. Let (α, -α) be the point of intersection
Sol. = p(t ) − 200
dt 2
c
d( p(t )) ∴ 4aα – 2aα + c = 0 ⇒ α=−
∫⎛1 ⎞ ∫
= dt 2a
⎜ p ( t ) − 200 ⎟
⎝2 ⎠ and 5bα – 2bα + d = 0 ⇒ α = −
d
⎛ p(t ) ⎞ 3b
2 log ⎜ − 200 ⎟ = t + c
⎝ 2 ⎠ ⇒ 3bc = 2ad
t
p(t ) ⇒ 3bc – 2ad = 0
− 200 = e2k
2
Using given condition p(t) = 400 – 300 et/2 Alternative method :

78. Let PS be the median of the triangle with vertices The point of intersection will be
P(2, 2), Q(6, –1) and R (7, 3). The equation of the line
passing through (1, –1) and parallel to PS is x –y 1
= =
2 ad – 2 bc 4 ad – 5bc 8 ab – 10 ab
(1) 4x + 7y + 3 = 0 (2) 2x – 9y – 11 = 0
2( ad – bc )
(3) 4x – 7y – 11 = 0 (4) 2x + 9y + 7 = 0 ⇒ x=
–2 ab
Answer (4)
5bc – 4 ad
⇒ y=
Sol. –2 ab
P(2,2)
∵ Point of intersection is in fourth quadrant so x
is positive and y is negative.

Also distance from axes is same

So x = – y (∵ distance from x-axis is – y as y is


negative)
Q(6,– 1) S R(7,3)

S is mid-point of QR

⎛7 +6 3–1⎞
So S = ⎜ , ⎟
⎝ 2 2 ⎠
⎛ 13 ⎞
= ⎜ , 1⎟
⎝ 2 ⎠ (x, y)

2–1 2 2( ad – bc ) –(5bc – 4 ad )
Slope of PS = =– =
13 9
2– –2 ab –2 ab
2
2
Equation of line ⇒ y – (–1) = – ( x – 1) 2ad – 2bc = – 5bc + 4ad
9
9y + 9 = – 2x + 2 ⇒ 2x + 9y + 7 = 0 ⇒ 3bc – 2ad = 0 ...(i)

(20)

www.vedantu.com
80. The locus of the foot of perpendicular drawn from If y > 0,
the centre of the ellipse x2 + 3y2 = 6 on any tangent y2 + 2 – 2y = y2 + 1 + 2y
to it is ⇒ 4y = 1
(1) (x2 + y2)2 = 6x2 + 2y2 1
⇒ y=
(2) (x2 + y2)2 = 6x2 – 2y2 4
If y < 0,
(3) (x2 – y2)2 = 6x2 + 2y2
y2 + 2 – 2y = y2 + 1 – 2y
(4) (x2 – y2)2 = 6x2 – 2y2 ⇒ 1 = 2 (Not possible)
Answer (1) 1
∴ y=
x2 y2 4
Sol. Here ellipse is + = 1 , where a2 = 6, b2 = 2 82. The slope of the line touching both the parabolas
a2 b2 y2 = 4x and x2 = –32y is
Now, equation of any variable tangent is
1 2
(1) (2)
y = mx ± a2 m2 + b 2 ...(i) 8 3
1 3
where m is slope of the tangent (3) (4)
2 2
So, equation of perpendicular line drawn from Answer (3)
centre to tangent is
Sol. y2 = 4x …(1)
−x 2
x = –32y …(2)
y= ...(ii)
m m be slope of common tangent
Eliminating m, we get Equation of tangent (1)

( x 2 + y 2 )2 = a2 x 2 + b 2 y 2 1
y = mx + …(i)
m
⇒ ( x 2 + y 2 )2 = 6 x 2 + 2 y 2 Equation of tangent (2)
y = mx + 8m2 …(ii)
81. Let C be the circle with centre at (1, 1) and radius = 1.
If T is the circle centred at (0, y), passing through (i) and (ii) are identical
origin and touching the circle C externally, then the 1
radius of T is equal to = 8m2
m
1
1 1 ⇒ m3 =
(1) (2) 8
2 4
1
m=
3 3 2
(3) (4) Alternative method :
2 2

Answer (2) Let tangent to y 2 = 4x be


Sol. 1
C y = mx +
m
as this is also tangent to x 2 = −32 y
(1, 1)
(0, y)
2 32
T Solving x + 32mx + =0
m
Since roots are equal
∴ D=0

C ≡ ( x − 1)2 + ( y − 1)2 = 1 2 32
⇒ (32) − 4 × =0
m
Radius of T = |y|
3 4
T touches C externally ⇒ m =
32
(0 – 1)2 + (y – 1)2 = (1 + |y|)2
1
⇒ 1 + y2 + 1 – 2y = 1 + y2 + 2|y| ⇒ m=
2
(21)

www.vedantu.com
83. The image of the line 84. The angle between the lines whose direction
consines satisfy the equations l + m + n = 0 and
x−1 y−3 z−4 l2 = m2 + n2 is
= = in the plane 2x – y + z + 3 = 0
3 1 −5 π π
(1) (2)
is the line 6 2
π π
x−3 y+5 z−2 (3) (4)
(1) = = 3 4
3 1 −5
Answer (3)
x−3 y+5 z−2 Sol. l + m + n = 0
(2) = = l2 = m2 + n2
−3 −1 5
Now, (–m – n)2 = m2 + n2
x+3 y−5 z−2 ⇒ mn = 0
(3) = =
3 1 −5 m = 0 or n = 0
If m = 0 If n = 0
x+3 y−5 z+2
(4) = = then l = –n then l = –m
−3 −1 5
l2 + m2 + n2 =1 l2 + m2 + n2 = 1
Answer (3) Gives ⇒ 2m2 = 1
1 1
⇒ n=±
2
⇒ m =
A (1,3, 4) 2 2
Sol.
1
3iˆ + ˆj + kˆ i.e. (l1, m1, n1) ⇒ m=±
2
⎛ 1 1 ⎞ 1
= ⎜− , 0, ⎟ Let m =
P ⎝ 2 2⎠ 2
1
3iˆ + ˆj + 5kˆ l=−
A′ 2
(a, b, c) n=0
(l2, m2, n2)
⎛ 1 1 ⎞
a−1 b−3 c−4 = ⎜− , ,0⎟
= = =λ ⎝ 2 2 ⎠
2 −1 1
1
∴ cos θ =
⇒ a = 2λ + 1 2
b=3–λ π
θ=
c=4+λ 3
      2
⎛ λ λ⎞ 85. If ⎡⎣ a × b b × c c × a ⎤⎦ = λ[ a b c ] then λ is equal to
P ≡ ⎜ λ + 1, 3 − , 4 + ⎟
⎝ 2 2⎠ (1) 0 (2) 1
⎛ λ⎞ ⎛ λ⎞ (3) 2 (4) 3
2( λ + 1) − ⎜ 3 − ⎟ + ⎜ 4 + ⎟ + 3 = 0
⎝ 2⎠ ⎝ 2⎠ Answer (2)
Sol. L.H.S.
λ λ    
2λ + 2 − 3 + +4+ +3 = 0
2 2 = ( a × b ) ⋅ [(b × c ) × (c × a )]
        
3λ + 6 = 0 ⇒ λ = – 2 = ( a × b ) ⋅ [(b × c ⋅ a )c – (b × c ⋅ c )a ]
     
a = – 3, b = 5, c = 2 = ( a × b ) ⋅ [[b c a ]c ] [∵ b × c . c = 0]
So the equation of the required line is = [ a b c ] ⋅ ( a × b ⋅ c ) = [ a b c ]2
     
x+3 y−5 z−2 [ a × b b × c c × a ] = [ a b c ]2
= =
3 1 −5 So λ = 1

(22)

www.vedantu.com
86. Let A and B be two events such that Answer (2)
1 1 1 1
P( A ∪ B) = , P( A ∩ B) = and P( A) = , where k k
6 4 4 Sol. f k ( x ) = (sin x + cos x )
k
A stands for the complement of the event A. Then 1 1
the events A and B are f 4 (x) − f6 (x) = (sin 4 x + cos 4 x ) − (sin 6 x + cos 6 x )
4 6
(1) Independent but not equally likely
1[ 1
(2) Independent and equally likely = 1 − 2 sin 2 x cos 2 x ] − [ 1 − 3 sin 2 x cos 2 x ]
4 6
(3) Mutually exclusive and independent
1 1 1
(4) Equally likely but not independent = − =
Answer (1) 4 6 12
89. A bird is sitting on the top of a vertical pole 20 m
1 1 5 high and its elevation from a point O on the ground
Sol. P( A ∪ B) =⇒ P( A ∪ B) = 1 – =
6 6 6 is 45°. It flies off horizontally straight away from the
1 1 3 point O. After one second, the elevation of the bird
P( A) = ⇒ P( A) = 1 – =
4 4 4 from O is reduced to 30°. Then the speed (in m/s)
∵ P( A ∪ B) = P( A) + P( B) – P( A ∩ B) of the bird is

5 3
= + P ( B) –
1 (1) 20 2 (2) 20 ( 3 −1 )
6 4 4
1
(3) 40 ( 2 −1 ) (4) 40 ( 3− 2 )
P( B) =
3 Answer (2)
∵ P(A) ≠ P(B) so they are not equally likely. Sol. A B
3 1 1
Also P(A) × P(B) = × =
4 3 4
= P(A ∩ B)
45° 20 20
∵ P( A ∩ B) = P( A) ⋅ P( B) so A & B are independent. 30°
87. The variance of first 50 even natural numbers is x y
437
(1) 437 (2) t=1s
4
20
833 From figure tan 45° =
(3) (4) 833 x
4
Answer (4) 20
and tan 30° =
x+y
Σxi2
Sol. Variance = − ( x )2 so, y = 20( 3 − 1)
N
2
2 2 2 + 4 2 + ... + 100 2 ⎛ 2 + 4 + ... + 100 ⎞ i.e., speed = 20( 3 − 1) m/s.
⇒ σ = −⎜ ⎟
50 ⎝ 50 ⎠ 90. The statement ~(p ↔ ~q) is
2 2 2 2
4(1 + 2 + 3 + .... + 50 ) (1) A tautology
= − (51)2
50 (2) A fallacy
⎛ 50 × 51 × 101 ⎞ 2 (3) Equivalent to p ↔ q
= 4⎜ ⎟ − (51)
⎝ 50 × 6 ⎠ (4) Equivalent to ~ p ↔ q
= 3434 – 2601 Answer (3)
⇒ σ = 833
2
Sol. ~(p ↔ ~ q)
1 p q ~q p ↔ ~q ~ (p ↔ ~q)
k k
88. Let f k ( x ) = (sin x + cos x ) where x ∈ R and k ≥ 1 . F F T F
k T
Then f4(x) – f6(x) equals F T F T F
1 1 T F T T F
(1) (2)
4 12
T T F F T
1 1
(3) (4) Clearly equivalent to p ↔ q
6 3
  
(23)

www.vedantu.com
Set - 03 35
59. _uQ¡_pdp„\u L$ey„ A¡L$ b¡¼V¡$qfep_piL$ ârsÆhuAp¡
(antibiotic) R>¡ ?
(1) Cqf\°p¡dpeku_
(2) V¡$V²$pkpe¼gu_
(3) ¼gp¡fA¡ça¡r_L$p¡g
(4) Ap¡ãgp¡¼k¡rk_

60. ""fyldp_ Å„byX$uep¡''(``®g) (Rhumann’s purple)


61. Let P = {θ : sinθ − cosθ = 2 cosθ } and 61. ◊ÊŸÊ P = {θ : sinθ − cosθ = 2 cosθ } ÃÕÊ Ahgp¡L$_ A¡ r_Zp®eL$ L$kp¡V$u _uQ¡_pdp„\u L$p¡_u lpS>fu
Q = {θ : sinθ + cosθ = 2 sinθ } be two Q = {θ : sinθ + cosθ = 2 sinθ } ŒÊ k|Qh¡ R>¡ ?
sets. Then : ‚◊Èëøÿ „Ò¥, ÃÊ — (1) fuX$éyk]N iL®$fp
(1) P ⊂ Q and Q−P ≠ φ
(1) P ⊂ Q ÃÕÊ Q−P ≠ φ (2) ¼eyâuL$ Ape_
(2) Q ⊄ P
(2) Q⊄P (3) âp¡V$u_
(3) P ⊄ Q
(3) P⊄Q (4) õV$pQ®
(4) P = Q
(4) P=Q

61. ^pfp¡ L¡ $ P = {θ : sinθ − cosθ = 2 cosθ }


A_¡ Q = {θ : sinθ + cosθ = 2 sinθ } b¡
NZp¡ R>¡. sp¡ :
(1) P ⊂ Q A_¡ Q−P ≠ φ
(2) Q⊄P
(3) P⊄Q
(4) P=Q

SET - 03 ENGLISH SET - 03 HINDI SET - 03 GUJARATI

www.vedantu.com 1
Set - 03 36
62. If x is a solution of the equation, 62. ÿÁŒ ‚◊Ë∑§⁄UáÊ 62. Å¡ kduL$fZ
 1  1  1
2 x + 1 − 2x − 1 = 1,  x   , then 2 x + 1 − 2x − 1 = 1,  x   _p¡ A¡L$
 2 2 x + 1 − 2x − 1 = 1,  x   ,
 2
∑§Ê x  2
4x 2 − 1 is equal to : DL¡$g x R>¡, sp¡
∞∑§ „‹ „Ò, ÃÊ 4x 2 − 1 ’⁄UÊ’⁄U „Ò — 4x 2 − 1 =__________ \pe.
3 3 3
(1) (1) (1)
4 4 4
1 1 1
(2) (2) (2)
2 2 2
(3) 2 (3) 2 (3) 2
(4) 2 2 (4) (4) 2 2
2 2

63. Let z=1+ai be a complex number, a > 0, ^pfp¡ L¡$ z=1+ai, a > 0, A¡L$ A¡hu k„L$f k„¿ep R>¡ L¡$
such that z3 is a real number. Then the sum
63. ◊ÊŸÊ ∞∑§ ∞‚Ë ‚Áê◊üÊ ‚¥ÅÿÊ „Ò,
z=1+ai, a > 0 63.

1+z+z2+.....+z11 is equal to : Á∑§z ∞∑§ flÊSÃÁfl∑§ ‚¥ Å ÿÊ „Ò , ÃÊ ÿÊ ª


3 S>¡ \ u z 3 hpõsrhL$ k„ ¿ ep \pe. sp¡ kfhpmp¡
1+z+z2+.....+z11 ’⁄UÊ’⁄U „Ò — 1+z+z2+.....+z11 =__________ \pe.
(1) −1250 3i
(1) −1250 3i (1) −1250 3i
(2) 1250 3i
(2) 1250 3i (2) 1250 3i
(3) 1365 3i
(3) 1365 3i (3) 1365 3i
(4) −1365 3i
(4) −1365 3i (4) −1365 3i

SET - 03 ENGLISH SET - 03 HINDI SET - 03 GUJARATI

www.vedantu.com 2
Set - 03 37
64. Let A be a 3×3 matrix such that 64. ◊ÊŸÊ ∑§Ê ∞∑§ ∞ ‚ Ê •Ê√ÿÍ „ „Ò Á∑§
A, 3×3 64. ^pfp¡ L¡ $ A A¡ L $ 3×3 î¡ r ZL$ R>¡ L¡ $ S>¡ \ u
A2−5A+7I=O.
A2−5A+7I=O „Ò– A2−5A+7I=O.
1
Statement - I : A−1 = (5I − A ). −1 1 1
7 ∑§ÕŸ - I :A = (5I − A ). rh^p_ - I : A−1 = (5I − A ).
7 7
Statement - II : The polynomial
A3−2A2−3A+I can be ∑§ÕŸ - II : ’„È ¬ Œ A 3 −2A 2 −3A+I ∑§Ê rh^p_ - II : blº ` v$u A 3 −2A 2 −3A+I _¡
reduced to 5(A−4I). 5(A−4I) ◊¥ ¬Á⁄UflÁøà Á∑§ÿÊ ¡Ê ‚∑§ÃÊ 5(A−4I) dp„ ê$`p„sqfs L$fu iL$pe R>¡.
Then : „Ò– sp¡ __________.
(1) Statement-I is true, but Statement-II ÃÊ, (1) rh^p_-I kÐe R>¡, `f„sy rh^p_-II AkÐe R>¡.
is false.
(2) Statement-I is false, but Statement-II (1) ∑§ÕŸ - I ‚àÿ „Ò ‹Á∑§Ÿ ∑§ÕŸ - II •‚àÿ „Ò– (2) rh^p_-I AkÐe R>¡, `f„sy rh^p_-II kÐe R>¡.
is true. (2) ∑§ÕŸ - I •‚àÿ „Ò ‹Á∑§Ÿ ∑§ÕŸ - II ‚àÿ „Ò– (3) b„_¡ rh^p_p¡ kÐe R>¡.
(3) Both the statements are true. (3) ŒÊŸÊ¥ ∑§ÕŸ ‚àÿ „Ò¥– (4) b„_¡ rh^p_p¡ AkÐe R>¡.
(4) Both the statements are false.
(4) ŒÊŸÊ¥ ∑§ÕŸ •‚àÿ „Ò¥–
−4 −1
−4 −1 65. Å¡ A = , sp¡ î¡ r ZL$
65. If A = , then the determinant of −4 −1 3 1 
 3 1  65. ÿÁŒ A = „Ò , ÃÊ •Ê√ÿÍ „
 3 1  (A 2016 −2A 2015 −A 2014 ) _p¡ r_òpeL$
the matrix (A2016−2A2015−A2014) is :
(1) 2014 (A2016−2A2015−A2014) ∑§Ê ‚Ê⁄UÁáÊ∑§ „Ò — __________ R>¡.
(1) 2014 (1) 2014
(2) −175
(2) −175 (2) −175
(3) 2016
(3) 2016 (3) 2016
(4) −25
(4) −25 (4) −25

SET - 03 ENGLISH SET - 03 HINDI SET - 03 GUJARATI

www.vedantu.com 3
Set - 03 38
n+2 n+2 n+2
C6 C6 C6
66. If
n−2
= 11, then n satisfies the 66. ÿÁŒ n−2
= 11, „Ò, ÃÊ n ÁŸêŸ ◊¥ ‚ Á∑§‚ 66. Å¡ n−2
= 11, sp¡ n _uQ¡_pdp„\u L$ey„ kduL$fZ
P2 P2 P2
equation : ‚◊Ë∑§⁄UáÊ ∑§Ê ‚¥ÃÈc≈U ∑§⁄UÃÊ „Ò? k„`p¡j¡?
(1) n2+3n−108=0 (1) n2+3n−108=0 (1) n2+3n−108=0
(2) n2+5n−84=0 (2) n2+5n−84=0 (2) n2+5n−84=0
(3) n2+2n−80=0 (3) n2+2n−80=0 (3) n2+2n−80=0
(4) n2+n−110=0 (4) n2+n−110=0 (4) n2+n−110=0

67. If the coefficients of x−2 and x−4 in the


18 18
ÿÁŒ  13 1 
, (x > 0), ∑§ ¬˝‚Ê⁄U ◊¥ x−2 Å¡  13 1 
, (x > 0) _p rhõsfZdp„ x−2
 13 1 18 67.
x + 1 
67.
x + 1 
expansion of + , (x > 0), are  2x 3   2x 3 
x 1 
 2x 3 
m A_¡ x−4 _p klNyZL$p¡ A_y¾$d¡ m A_¡ n lp¡e sp¡,
m ÃÕÊ x−4 ∑§ ªÈáÊÊ¥∑§ ∑˝§◊‡Ê— m ÃÕÊ n „Ò¥, ÃÊ
m and n respectively, then is equal to : n m
n =__________ \pe.
’⁄UÊ’⁄U „Ò — n
(1) 182
(1) 182 (1) 182
4
(2) 4 4
5 (2) (2)
5 5
5
(3) 5 5
4 (3) (3)
4 4
(4) 27
(4) 27 (4) 27

68. Let a1, a2, a3, ......, an, ..... be in A.P. If


a3+a7+a11+a15=72, then the sum of its 68. ◊ÊŸÊ a1, a2, a3, ......, an, ..... ∞∑§ ‚◊Ê¥Ã⁄U üÊ…∏Ë ◊¥ „Ò¥– 68. ^pfp¡ L¡$ a1, a2, a3, ......, an, ..... kdp„sf î¡Zudp„ R>¡.
first 17 terms is equal to : ÿÁŒ a3+a7+a11+a15=72 „Ò, ÃÊ ©‚∑§ ¬˝Õ◊ 17 Å¡ a3+a7+a11+a15=72, sp¡ s¡_p â\d 17 `v$p_¡ p¡
(1) 306 ¬ŒÊ¥ ∑§Ê ÿÊª ’⁄UÊ’⁄U „Ò — kfhpmp¡ __________ \pe.
(2) 153 (1) 306 (1) 306
(3) 612 (2) 153 (2) 153
(4) 204 (3) 612 (3) 612
(4) 204 (4) 204
SET - 03 ENGLISH SET - 03 HINDI SET - 03 GUJARATI

www.vedantu.com 4
Set - 03 39
10 10 10
69. The sum ∑ ( r 2+ 1) × (r!) is equal to : 69. ÿÊª»§‹ ∑ ( r 2+ 1 ) × (r!) ’⁄UÊ’⁄U „Ò — 69. ∑ ( r 2+ 1) × (r!) =__________.
r =1 r =1 r =1

(1) (11)! (1) (11)! (1) (11)!


(2) 10×(11!) (2) 10×(11!) (2) 10×(11!)
(3) 101×(10!) (3) 101×(10!) (3) 101×(10!)
(4) 11×(11!) (4) 11×(11!) (4) 11×(11!)

( 1 − cos2 x )2 ( 1 − cos2 x )2 ( 1 − cos2 x )2


70. lim is : 70. lim ’⁄UÊ’⁄U „Ò — 70. lim =__________.
x →0 2 x tan x − x tan 2 x x →0 2 x tan x − x tan 2 x x →0 2 x tan x − x tan 2 x
(1) −2 (1) −2 (1) −2
1 1 1
(2) − (2) − (2) −
2 2 2
1 1 1
(3) (3) (3)
2 2 2
(4) 2 (4) 2 (4) 2

SET - 03 ENGLISH SET - 03 HINDI SET - 03 GUJARATI

www.vedantu.com 5
Set - 03 40
71. Let a, b e R, (a ≠ 0). If the function f defined 71. ◊ÊŸÊ a, b e R, (a ≠ 0)– ÿÁŒ »§‹Ÿ f ¡Ê, ÁŸêŸ mÊ⁄UÊ 71. ^pfp¡ L¡$ a, b e R, (a ≠ 0).
as
¬Á⁄U÷ÊÁ·Ã „Ò —
 2x2
 2x2  , 0≤x < 1
 , 0≤x < 1  2x2  a
 a  , 0≤x <1 
  a Å¡ rh^¡e f (x) =  a , 1≤x < 2
f (x) =  a , 1≤x < 2   2
 2 f (x) =  a , 1≤x < 2
 2b −4b ,
 2b −4b ,  2  x 3
2 ≤x < ∞
2 ≤x < ∞  2b −4b ,
 x 3 2 ≤x < ∞
 x 3
A„sfpg dp„ kss lp¡e, sp¡ ¾$dey¼s Å¡X$
[0, ∞)
is continuous in the interval [0, ∞), then an
•¥Ã⁄UÊ‹ [0, ∞) ◊¥ ‚Ãà „Ò, ÃÊ ∞∑§ ∑˝§Á◊à ÿÈÇ◊ (a, b) = __________ R>¡.
ordered pair (a, b) is :
(a, b) „Ò — (1) ( 2, 1− 3)
(1) ( 2, 1− 3)
(1) ( 2, 1− 3) (2) (− 2, 1+ 3)
(2) (− 2, 1+ 3)
(2) (− 2, 1+ 3) (3) ( 2 , −1 + 3)
(3) ( 2 , −1 + 3)
(3) ( 2 , −1 + 3) (4) (− 2, 1− 3)
(4) (− 2, 1− 3)
(4) (− 2, 1− 3)
72. Let f(x)=sin 4 x+cos 4 x. Then f is an 72. ^pfp¡ L¡$ f(x)=sin4x+cos4x. sp¡ _uQ¡_pdp„\u L$ep
increasing function in the interval : 72. ◊ÊŸÊ f(x)=sin4x+cos4x „Ò, ÃÊ ÁŸêŸ ◊¥ ‚ Á∑§‚ A„sfpgdp„ f h^sy„ rh^¡e R>¡ ?
•¥Ã⁄UÊ‹ ◊¥ f ∞∑§ flœ¸◊ÊŸ »§‹Ÿ „Ò?
 π  π
(1)  0, 4   π (1)  0, 4 
(1)  0, 4 
π π  π π 
(2)  4 , 2  π π  (2)  4 , 2 
(2)  4 , 2 
 π 5π   π 5π 
(3)  2 , 8   π 5π  (3)  2 , 8 
(3)  2 , 8 
 5π 3π   5π 3π 
(4)  8 , 4   5π 3π  (4)  8 , 4 
(4)  8 , 4 

SET - 03 ENGLISH SET - 03 HINDI SET - 03 GUJARATI

www.vedantu.com 6
Set - 03 41
73. Let C be a curve given by
73. ◊ÊŸÊ C ∞∑§ fl∑˝§ „Ò ¡Ê y (x) = 1 + 4x − 3 , 73. ^pfp¡ L¡$ A¡L$ h¾$ C A¡ y ( x ) = 1 + 4x − 3,
3
y ( x) = 1 + 4x − 3, x > . If P is a point 3 3
4 x> mÊ⁄UÊ ¬˝ŒûÊ „Ò– ÿÁŒ C ¬⁄U P ∞∑§ ∞‚Ê Á’¥ŒÈ „Ò x> Üpfp v$ip®h¡g R>¡. Å¡ C `f A¡L$ A¡hy„ tbvy$ P R>¡
on C, such that the tangent at P has slope 4 4
2 , then a point through which the normal Á∑§ P ¬⁄U πË¥øË ªß¸ S¬‡Ê¸ ⁄UπÊ ∑§Ë …Ê‹ 2 „Ò, ÃÊ fl„ L¡$ S>¡\u tbvy$ P ApNm_p õ`i®L$_p¡ Y$pm 2 \pe, sp¡ P
3 3 3
at P passes, is : Á’¥ŒÈ Á¡‚‚ P ¬⁄U πË¥øÊ ªÿÊ •Á÷‹¥’ ªÈ$¡⁄UÃÊ „Ò, „Ò — ApNm_p¡ Arcg„b __________ tbvy$dp„\u `kpf \pe
(1) (2, 3) (1) (2, 3) R>¡.
(2) (4, −3) (2) (4, −3) (1) (2, 3)
(3) (1, 7) (3) (1, 7) (2) (4, −3)
(4) (3, −4) (4) (3, −4) (3) (1, 7)
(4) (3, −4)

dx dx
74. The integral is equal 74. ‚◊Ê∑§‹ ’⁄UÊ’⁄U „Ò —
(1 + x) x− x 2 (1 + x) x − x2 dx
74. =__________.
(1 + x) x − x2
to : (¡„Ê° C ∞∑§ ‚◊Ê∑§‹Ÿ •ø⁄U „Ò–)
(where C is a constant of integration.)
(Äep„ C k„L$g__p¡ AQmp„L$ R>¡.)
1+ x
1+ x (1) −2 +C
(1) −2 +C 1− x 1+ x
1− x (1) −2 +C
1− x
1− x
1− x (2) −2 +C
(2) −2 +C 1+ x 1− x
1+ x (2) −2 +C
1+ x
1− x
1− x (3) − +C
(3) − +C 1+ x 1− x
1+ x (3) − +C
1+ x
1+ x
1+ x (4) 2 +C
(4) 2 +C 1− x 1+ x
1− x (4) 2 +C
1− x

SET - 03 ENGLISH SET - 03 HINDI SET - 03 GUJARATI

www.vedantu.com 7
Set - 03 42
75. The value of the integral
 x 2  dx  x 2  dx
75. ‚◊Ê∑§‹   , ¡„Ê° 75.   =_________.
 x 2  dx
  , where [x]  x 2− 28x + 196  +  x 2   x 2− 28x + 196  +  x 2 
       
 x 2− 28x + 196  +  x 2 
   
[x], x ‚ ∑§◊ ÿÊ x ∑§ ’⁄UÊ’⁄U ◊„ûÊ◊ ¬ÍáÊÊZ∑§ „Ò, ∑§Ê Äep„ [x] A¡ x \u _p_p A\hp x _¡ kdp_ sdpd
denotes the greatest integer less than or ◊ÊŸ „Ò — `|Zp¯L$p¡dp„ kp¥\u dp¡V$p¡ `|Zp¯L$ v$ip®h¡ R>¡.
equal to x, is :
(1) 6 (1) 6
(1) 6
(2) 3 (2) 3
(2) 3
(3) 7 (3) 7
(3) 7
1 1
1 (4) (4)
(4) 3 3
3
76. x e R, x ≠ 0, ∑§ Á‹∞, ÿÁŒ y(x) ∞∑§ ∞‚Ê •fl∑§‹ŸËÿ
76. For x e R, x ≠ 0, if y(x) is a differentiable 76. Å¡ x e R, x ≠ 0 dpV¡$, y(x) A¡L$ rhL$g_ue rh^¡e R>¡ L¡$
function such that »§‹Ÿ „Ò Á∑§
x x
x x x x S>¡ \ u x ∫ y (t ) dt = ( x + 1) ∫ t y (t ) dt , sp¡
x ∫ y (t ) dt = ( x + 1) ∫ t y (t ) dt , then y(x) x ∫ y (t ) dt = ( x + 1) ∫ t y (t ) dt „Ò, ÃÊ y (x) 1 1
1 1 1 1
y(x)=__________.
equals : ’⁄UÊ’⁄U „Ò — (Äep„ C AQm R>¡.)
(where C is a constant.) (¡„Ê° C ∞∑§ •ø⁄U „Ò–)
1
1 C −x
C − 1 (1) e
(1) e x C −
x
x (1) e x
x
1
1
1
C −x
C −
C
(2) e
(2) x
x2
e −
2 (2) e x
x2
x
1 1
C −x 1 C −x
(3) e C − (3) e
x3 (3) 3
e x
x3
x
1 1
(4) 1
3 x
Cx e (4) (4) 3
3 x
Cx e Cx e x

SET - 03 ENGLISH SET - 03 HINDI SET - 03 GUJARATI

www.vedantu.com 8
Set - 03 43
77. The solution of the differential equation dy y dy y
tanx tanx
dy y tanx 77. •fl∑§‹ ‚◊Ë∑§⁄UáÊ + secx = , ¡„Ê° 77. rhL$g kduL$fZ + secx = , Äep„
+ secx = , where 0 ≤ x < π , dx 2 2y dx 2 2y
dx 2 2y 2
and y(0)=1, is given by : 0≤x <
π
„Ò ÃÕÊ y(0)=1 „Ò, ∑§Ê „‹ „Ò — 0≤x <
π
, A_¡ y(0)=1, _p¡ DL¡$g _________
2 2

(1) y = 1−
x
x R>¡.
sec x + tan x (1) y = 1−
sec x + tan x x
(1) y = 1−
2 x sec x + tan x
(2) y = 1+ x
sec x + tan x (2) y2 = 1 +
sec x + tan x x
(2) y2 = 1 +
2 x sec x + tan x
(3) y = 1− x
sec x + tan x (3) y2 = 1 −
sec x + tan x x
(3) y2 = 1 −
x sec x + tan x
(4) y = 1+ x
sec x + tan x (4) y = 1+
sec x + tan x x
(4) y = 1+
sec x + tan x
78. A ray of light is incident along a line which
meets another line, 7x−y+1=0, at the 78. ¬˝∑§Ê‡Ê ∑§Ë ∞∑§ Á∑§⁄UáÊ ∞∑§ ⁄UπÊ ∑§Ë ÁŒ‡ÊÊ ◊¥ •Ê¬ÁÃÃ
point (0, 1). The ray is then reflected from „Ò ¡Ê ∞∑§ •ãÿ ⁄UπÊ 7x−y+1=0 ∑§Ê Á’¥ŒÈ 78. âL$pi_y„ A¡L$ qL$fZ f¡Mp 7x−y+1=0 `f Ap`ps
this point along the line, y+2x=1. Then (0, 1) ¬⁄U Á◊‹ÃË „Ò– fl„ Á∑§⁄UáÊ Á»§⁄U ß‚ Á’¥ŒÈ ‚ ⁄UπÊ \pe R>¡ S>¡ tbv$y$ (0, 1) ApNm dm¡ R>¡. Ðepfbpv$ Ap
the equation of the line of incidence of the
y+2x=1 ∑§Ë ÁŒ‡ÊÊ ◊¥ ¬Á⁄UflÁøà „ÊÃË „Ò, ÃÊ •Ê¬ÁÃà qL$fZ Ap tbvy$dp„\u `fphrs®s \C f¡Mp y+2x=1
ray of light is :
(1) 41x−38y+38=0 ¬˝∑§Ê‡Ê ∑§Ë Á∑§⁄UáÊ ∑§Ê ‚◊Ë∑§⁄UáÊ „Ò — `f fl¡ R>¡. sp¡ Ap`ps qL$fZ_y„ kduL$fZ __________
(2) 41x+25y−25=0 (1) 41x−38y+38=0 R>¡.
(3) 41x+38y−38=0 (2) 41x+25y−25=0 (1) 41x−38y+38=0
(4) 41x−25y+25=0 (3) 41x+38y−38=0 (2) 41x+25y−25=0
(4) 41x−25y+25=0 (3) 41x+38y−38=0
(4) 41x−25y+25=0

SET - 03 ENGLISH SET - 03 HINDI SET - 03 GUJARATI

www.vedantu.com 9
Set - 03 44
79. A straight line through origin O meets the 79. ◊Í‹ Á’¥ŒÈ O ‚ „Ê∑§⁄U ¡ÊŸ flÊ‹Ë ∞∑§ ‚⁄U‹ ⁄UπÊ ⁄UπÊ•Ê¥ 79. ENdtbvy$ O dp„\u `kpf \su A¡L$ f¡Mp, f¡MpAp¡
lines 3y=10−4x and 8x+6y+5=0 at
3y=10−4x ÃÕÊ 8x+6y+5=0 ∑§Ê ∑˝ § ◊‡Ê— 3y=10−4x A_¡ 8x+6y+5=0 _¡ A_y ¾ $ d ¡
points A and B respectively. Then O
divides the segment AB in the ratio : Á’¥ŒÈ•Ê¥ A ÃÕÊ B ¬⁄U Á◊‹ÃË „Ò¥, ÃÊ Á’¥ŒÈ O ⁄UπÊπ¥«U tbvy$Ap¡ A A_¡ B dp„ dm¡ R>¡. sp¡ O A¡ f¡MpM„X$ AB _¡
(1) 2 : 3 AB ∑§Ê Á¡‚ •ŸÈ¬Êà ◊¥ Áfl÷ÊÁ¡Ã ∑§⁄UÃÊ „Ò, fl„ „Ò — __________ NyZp¡Ñfdp„ rhcpS>_ L$f¡ R>¡.
(2) 1 : 2 (1) 2:3 (1) 2:3
(3) 4 : 1 (2) 1:2 (2) 1:2
(4) 3 : 4 (3) 4:1 (3) 4:1
(4) 3:4 (4) 3:4
80. Equation of the tangent to the circle, at the
point (1, −1), whose centre is the point of 80. ©‚ flÎûÊ Á¡‚∑§Ê ∑§ãŒ˝ ‚⁄U‹ ⁄UπÊ•Ê¥ x−y=1 ÃÕÊ 80. S>¡_„y L¡$ÞÖ f¡MpAp¡ x−y=1 A_¡ 2x+y=3 _y„ R>¡v$tbvy$
intersection of the straight lines x−y=1
2x+y=3 ∑§Ê ¬˝ÁÃë¿UŒ Á’¥ŒÈ „Ò, ∑§ Á’¥ŒÈ (1, −1) ¬⁄U lp¡e s¡hp hsy®m_¡ tbvy$ (1, −1) ApNm_p õ`i®L$_y„
and 2x+y=3 is :
(1) 4x+y−3=0 πË¥øË ªß¸ S¬‡Ê¸ ⁄πÊ ∑§Ê ‚◊Ë∑§⁄UáÊ „Ò — kduL$fZ __________ R>¡.
(2) x+4y+3=0 (1) 4x+y−3=0 (1) 4x+y−3=0
(3) 3x−y−4=0 (2) x+4y+3=0 (2) x+4y+3=0
(4) x−3y−4=0 (3) 3x−y−4=0 (3) 3x−y−4=0
(4) x−3y−4=0 (4) x−3y−4=0
81. P and Q are two distinct points on the
parabola, y2=4x, with parameters t and t1 81. P ÃÕÊ Q ¬⁄Ufl‹ÿ y2=4x ¬⁄U ÁSÕà ŒÊ Á÷㟠Á’¥ŒÈ „Ò 81. P A_¡ Q `fhge y2=4x `f Aph¡gp b¡ rcÞ_
respectively. If the normal at P passes Á¡Ÿ∑§ ¬˝Êø‹ ∑˝§◊‡Ê— t ÃÕÊ t1 „Ò¥– ÿÁŒ P ¬⁄U πË¥øÊ tbv$y$Ap¡ R>¡ S>¡_p âpQgp¡ A_y¾$d¡ t A_¡ t1 R>¡. Å¡ P
through Q, then the minimum value of t12
ªÿÊ •Á÷‹¥’ Q ‚ „Ê∑§⁄U ¡ÊÃÊ „Ò, ÃÊ t12 ∑§Ê ãÿÍŸÃ◊ ApNm_p¡ Arcg„b Q dp„\u `kpf \sp¡ lp¡e, sp¡ t12
is :
◊ÊŸ „Ò — _y„ Þe|_sd d|ëe __________ R>¡.
(1) 2
(1) 2 (1) 2
(2) 4
(2) 4 (2) 4
(3) 6
(3) 6 (3) 6
(4) 8
(4) 8 (4) 8

SET - 03 ENGLISH SET - 03 HINDI SET - 03 GUJARATI

www.vedantu.com 10
Set - 03 45
82. A hyperbola whose transverse axis is along 82. ∞∑§ •Áì⁄Ufl‹ÿ, Á¡‚∑§Ê •ŸÈ¬˝SÕ •ˇÊ ‡ÊÊ¥∑§fl 82. A¡ L $ Arshge S>¡ _ p¡ dy ¿ e An ip„ L $h
x2 y2 x2 y2 x2 y2
the major axis of the conic, + =4 + =4 ∑§ ŒËÉʸ •ˇÊ ∑§Ë ÁŒ‡ÊÊ ◊¥ „Ò ÃÕÊ + =4 _p â^p_ An `f R>¡ s\p s¡_p
3 4 3 4 3 4
and has vertices at the foci of this conic. If
Á¡‚∑§ ‡ÊË·¸ ß‚ ‡ÊÊ¥∑§fl ∑§Ë ŸÊÁ÷ÿÊ¥ ¬⁄U „Ò– ÿÁŒ rifp¡ t bvy $ A p¡ Ap ip„ L $ h _u _prcAp¡ `f R>¡ . Å¡
3
the eccentricity of the hyperbola is , then 3 3
2 •Áì⁄Ufl‹ÿ ∑§Ë ©à∑§ãŒ˝ÃÊ 2
„Ò, ÃÊ ÁŸêŸ ◊¥ ‚ ∑§ÊÒŸ Arshge_u DÐL¡$ÞÖsp 2
lp¡e, sp¡ _uQ¡_pdp„\u L$ey„
which of the following points does NOT
lie on it ? ‚Ê Á’¥ŒÈ ß‚ ¬⁄U ÁSÕà Ÿ„Ë¥ „Ò? tbvy$ s¡_p `f Aph¡gy„ _l] lp¡e ?
(1) (0, 2) (1) (0, 2) (1) (0, 2)

(2) ( 5, 2 2 ) (2) ( 5, 2 2 ) (2) ( 5, 2 2 )


(3) ( 10 , 2 3 ) (3) ( 10 , 2 3 ) (3) ( 10 , 2 3 )
(4) (5, 2 3 ) (4) (5, 2 3 ) (4) (5, 2 3 )

83. ABC is a triangle in a plane with vertices 83. ∞∑§ ‚◊Ë ◊¥ ∞∑§ ÁòÊ÷È¡ ABC „Ò Á¡‚∑§ ‡ÊË·¸ 83. A¡L$ kdsgdp„ A¡L$ rÓL$p¡Z ABC R>¡. S>¡_p rifp¡tbvy$Ap¡
A(2, 3, 5), B(−1, 3, 2) and C(λ, 5, µ). If the A(2, 3, 5), B(−1, 3, 2) ÃÕÊ C(λ, 5, µ) „Ò¥– ÿÁŒ A(2, 3, 5), B(−1, 3, 2) A_¡ C(λ, 5, µ) R>¡. Å¡ A
median through A is equally inclined to the
A ‚ „Ê∑§⁄U ¡ÊÃË ◊ÊÁäÿ∑§Ê ÁŸŒ¸‡ÊÊ¥∑§ •ˇÊÊ¥ ¬⁄U ‚◊ÊŸ dp„\u _uL$msu dÝeNp epdpnp kp\¡ kdp_ fus¡ Y$m¡g
coordinate axes, then the value of
(λ3+µ3+5) is : M§¬ ‚ ¤ÊÈ∑§Ë „Ò, ÃÊ (λ3+µ3+5) ∑§Ê ◊ÊŸ „Ò — R>¡, sp¡ (λ3+µ3+5) _u qL„$ds __________ R>¡.
(1) 1130 (1) 1130 (1) 1130
(2) 1348 (2) 1348 (2) 1348
(3) 676 (3) 676 (3) 676
(4) 1077 (4) 1077 (4) 1077

SET - 03 ENGLISH SET - 03 HINDI SET - 03 GUJARATI

www.vedantu.com 11
Set - 03 46
84. The number of distinct real values of λ for 84. λ ∑§ fl„ Á÷㟠flÊSÃÁfl∑§ ◊ÊŸÊ¥ ∑§Ë ‚¥ÅÿÊ Á¡Ÿ∑§ x− 1 y− 2 z+ 3
x− 1 y− 2 z+ 3
84. S>¡_p dpV¡$ f¡MpAp¡ = =
which the lines x− 1 y− 2 z+ 3 1 2 λ2
1
=
2
=
λ2
Á‹∞ ⁄ π Ê∞° = = ÃÕÊ
1 2 λ2 x− 3 y− 2 z− 1
A_¡ = = kdsgue \pe
x− 3 y− 2 z− 1 x− 3 y− 2 z− 1 1 λ 2 2
and = = are ‚◊ËËÿ „Ò¥, „Ò —
1 2 2 = 2
=
λ 1 λ 2 s¡ h u _u rcÞ_ hpõsrhL$ qL„ $ dsp¡ _ u k„ ¿ ep
λ
coplanar is :
(1) 4 __________ R>¡.
(1) 4
(2) 1 (1) 4
(2) 1
(3) 2 (2) 1
(3) 2
(4) 3 (3) 2
(4) 3
(4) 3
85. ◊ÊŸÊ ABC ∞∑§ ÁòÊ÷È¡ „Ò Á¡‚∑§Ê ¬Á⁄U∑§ãŒ˝ P ¬⁄U „Ò–
85. Let ABC be a triangle whose circumcentre
is at P. If the position vectors of A, B, C ÿÁŒ Á’¥ŒÈ•Ê¥ A, B, C ÃÕÊ P ∑§ ÁSÕÁà ‚ÁŒ‡Ê ∑˝§◊‡Ê— 85. ^pfp¡ L¡$ ABC A¡L$ rÓL$p¡Z R>¡ S>¡_y„ `qfL¡$ÞÖ P ApNm
→ → → → → → R>¡. Å¡ A, B, C A_¡ P _p õ\p_kqv$ip¡ A_y¾$d¡
→ → → → → →
ÃÕÊ a + b + c „Ò¥, ÃÊ ß‚ ÁòÊ÷È¡
and P are a , b , c and a + b + c a, b, c → → →
→ → →
4 4 a, b, c A_¡ a + b + c lp¡e, sp¡ Ap
respectively, then the position vector of the ∑§ ‹¥’ - ∑§ãŒ˝ ∑§Ê ÁSÕÁà ‚ÁŒ‡Ê „Ò — 4
orthocentre of this triangle, is :
→ → → rÓL$p¡Z_p g„bL¡$ÞÖ_p¡ õ\p_kqv$i __________ R>¡.
→ → → (1) a + b + c
→ → →
(1) a + b + c (1) a + b + c
→ → →
a b c 
−
→ → → (2) + +
a + b + c    → → →
(2) − 2 (2)  a + b + c 
 2    −
 
2
   

→ (3) 0

(3) 0 (3) 0
→ → →
→ → (4) a + b + c

  → → →
(4) a + b + c (4) a + b + c
  2  
2 2

SET - 03 ENGLISH SET - 03 HINDI SET - 03 GUJARATI

www.vedantu.com 12
Set - 03 47
86. The mean of 5 observations is 5 and their 86. 5 ¬˝ˇÊáÊÊ¥ ∑§Ê ◊Êäÿ 5 „Ò ÃÕÊ ©Ÿ∑§Ê ¬˝‚⁄UáÊ 124 „Ò– 86. 5 Ahgp¡L$_p¡_p¡ dÝeL$ 5 A_¡ rhQfZ 124 R>¡. Å¡
variance is 124. If three of the observations
ÿÁŒ ©Ÿ◊¥ ‚ ÃËŸ ¬˝ˇÊáÊ 1, 2 ÃÕÊ 6 „Ò¥, ÃÊ ߟ •Ê°∑§«∏Ê¥ s¡dp„\u ÓZ Ahgp¡L$_p¡ 1, 2 A_¡ 6 lp¡e, sp¡ dprlsu_y„
are 1, 2 and 6 ; then the mean deviation
from the mean of the data is : ∑§Ê ◊Êäÿ ‚ ◊Êäÿ Áflø‹Ÿ „Ò — dÝeL$\u kf¡fpi rhQg_ __________ \pe.
(1) 2.4 (1) 2.4 (1) 2.4
(2) 2.8 (2) 2.8 (2) 2.8
(3) 2.5 (3) 2.5 (3) 2.5
(4) 2.6 (4) 2.6 (4) 2.6

87. An experiment succeeds twice as often as 87. ∞∑§ ¬˝ÿÊª ∑§ ‚»§‹ „ÊŸ ∑§Ê ‚¥ÿÊª ©‚∑§ Áfl»§‹ 87. A¡L$ âep¡N S>¡V$gp hMs Akam \pe R>¡ s¡_p\u b¡
it fails. The probability of at least 5 „ÊŸ ∑§ ‚¥ÿÊª ∑§Ê ŒÈªÈŸÊ „Ò– ß‚ ¬˝ÿÊª ∑§ 6 ¬⁄UˡÊáÊÊ¥ NZp¡ kam \pe R>¡. Ap âep¡N_p R> âeÐ_p¡dp„\u
successes in the six trials of this experiment ◊¥ ‚ ∑§◊ ‚ ∑§◊ ¬Ê°ø ∑§ ‚»§‹ „ÊŸ ∑§Ë ¬˝ÊÁÿ∑§ÃÊ
is :
Ap¡ R >pdp„ Ap¡ R >u 5 kamsp dmhp_u k„ c ph_p
„Ò — __________ R>¡.
240
(1) 240 240
729 (1) (1)
729 729
192
(2) 192 192
729 (2) (2)
729 729
256
(3) 256 256
729 (3) (3)
729 729
496
(4) 496 496
729 (4) (4)
729 729

SET - 03 ENGLISH SET - 03 HINDI SET - 03 GUJARATI

www.vedantu.com 13
Set - 03 48
π π π
88. If A>0, B>0 and Α + B = , then the 88. ÿÁŒ A>0, B>0 ÃÕÊ Α+B= „Ò , ÃÊ 88. Å¡ A>0, B>0 A_¡ Α+B= lp¡ e , sp¡
6 6 6
minimum value of tanA+tanB is : tanA+tanB ∑§Ê ãÿÍŸÃ◊ ◊ÊŸ „Ò — tanA+tanB _y„ Þe|_sd d|ëe __________ R>¡.
(1) 3− 2 (1) 3− 2 (1) 3− 2
(2) 2− 3 (2) 2− 3 (2) 2− 3
(3) 4 −2 3 (3) 4 −2 3 (3) 4 −2 3
2 2 2
(4) (4) (4)
3 3 3

89. The angle of elevation of the top of a 89. Á’¥ŒÈ A ‚, ¡Ê ∞∑§ ™§äflʸœ⁄U ◊ËŸÊ⁄U ∑§ ¬Ífl¸ ∑§Ë •Ê⁄U
vertical tower from a point A, due east of it „Ò, ◊ËŸÊ⁄U ∑§ ‡ÊË·¸ ∑§Ê ©ãŸÿŸ ∑§ÊáÊ 458 „Ò– Á’¥ŒÈ B, 89. A¡L$ rifp¡g„b V$phf_u `|h® sfa_p tbvy$ A \u V$phf_u
is 458. The angle of elevation of the top of
¡Ê Á’¥ŒÈ A ∑§ ŒÁˇÊáÊ ◊¥ „Ò, ‚ ©‚Ë ◊ËŸÊ⁄U ∑§ ‡ÊË·¸ ∑§Ê V$p¡Q_p¡ DÐk¡^L$p¡Z 458 R>¡. tbvy$ A _u v$rnZ¡ Aph¡g
the same tower from a point B, due south
©ãŸÿŸ ∑§ÊáÊ 308 „Ò– ÿÁŒ A ÃÕÊ B ∑§ ’Ëø ∑§Ë ŒÍ⁄UË tbvy$ B \u V$phf_u V$p¡Q_p¡ DÐk¡^L$p¡Z 308 R>¡. Å¡ A
of A is 308. If the distance between A and
B is 54 2 m , then the height of the tower 54 2 ◊Ë. „Ò, ÃÊ ◊ËŸÊ⁄U ∑§Ë ™§°øÊ߸ (◊Ë. ◊¥) „Ò — A_¡ B hÃQ¡_y„ A„sf 54 2 m lp¡e, sp¡ V$phf_u
(in metres), is : KQpC (duV$fdp„) __________ R>¡.
(1) 36 3
(1) 36 3 (2) 54 (1) 36 3
(2) 54 (2) 54
(3) 54 3
(3) 54 3 (4) 108 (3) 54 3
(4) 108 (4) 108

SET - 03 ENGLISH SET - 03 HINDI SET - 03 GUJARATI

www.vedantu.com 14
Set - 03 49
90. The contrapositive of the following 90. ÁŸêŸ ∑§ÕŸ ∑§Ê ¬˝ÁÃœŸÊà◊∑§ (contrapositive) 90. _uQ¡_p rh^p__y„ kdp_p\} â¡fZ L$ey„ R>¡ ?
statement,
„Ò — ""Å¡ L$p¡C Qp¡fk_u bpSy> bdZu L$fhpdp„ Aph¡, sp¡ s¡_y„
“If the side of a square doubles, then its area
increases four times”, is : ““ÿÁŒ Á∑§‚Ë flª¸ ∑§Ë ÷È¡Ê ŒÈªÈŸË „Ê ¡Ê∞, ÃÊ ©‚∑§Ê n¡Óam QpfNÏ„ h^¡.''
(1) If the side of a square is not doubled, ˇÊòÊ»§‹ øÊ⁄U ªÈŸÊ ’…∏ ¡ÊÃÊ „Ò”” — (1) Å¡ L$p¡C Qp¡fk_u bpSy> bdZu L$fhpdp„ _ Aph¡,
then its area does not increase four (1) ÿÁŒ ∞∑§ flª¸ ∑§Ë ÷È¡Ê ŒÈªÈŸË Ÿ ∑§Ë ¡Ê∞, ÃÊ sp¡ s¡_y„ n¡Óam QpfNÏ„ h^i¡ _rl.
times. ©‚∑§Ê ˇÊòÊ»§‹ øÊ⁄U ªÈŸÊ Ÿ„Ë¥ ’…∏ÃÊ– (2) Å¡ L$pC ¡ Qp¡fk_y„ n¡Óam QpfNÏ„ h^pfhpdp„ Aph¡,
(2) If the area of a square increases four
(2) ÿÁŒ Á∑§‚Ë flª¸ ∑§Ê ˇÊòÊ»§‹ øÊ⁄U ªÈŸÊ ’…∏ ¡Ê∞, sp¡ s¡_u bpSy> bdZu \pe.
times, then its side is doubled.
(3) If the area of a square increases four
ÃÊ ©‚∑§Ë ÷È¡Ê ŒÈªÈŸË „Ê ¡ÊÃË „Ò– (3) Å¡ L$p¡C Qp¡fk_y„ n¡Óam QpfNÏ„ h^pfhpdp„
times, then its side is not doubled. (3) ÿÁŒ Á∑§‚Ë flª¸ ∑§Ê ˇÊòÊ»§‹ øÊ⁄U ªÈŸÊ ’…∏ ¡ÊÃÊ Aph¡, sp¡ s¡_u bpSy> bdZu _ \pe.
(4) If the area of a square does not „Ò, ÃÊ ©‚∑§Ë ÷È¡Ê ŒÈªÈŸË Ÿ„Ë¥ „ÊÃË–
increase four times, then its side is not (4) Å¡ L$p¡C Qp¡fk_y„ n¡Óam QpfNÏ„ h^pfhpdp„ _
(4) ÿÁŒ Á∑§‚Ë flª¸ ∑§Ê ˇÊòÊ»§‹ øÊ⁄U ªÈŸÊ Ÿ„Ë¥ ’…∏ÃÊ,
doubled. Aph¡, sp¡ s¡_u bpSy> bdZu _ \pe.
ÃÊ ©‚∑§Ë ÷È¡Ê ŒÈªÈŸË Ÿ„Ë¥ „ÊÃË–
-oOo- -oOo-
-oOo-

SET - 03 ENGLISH SET - 03 HINDI SET - 03 GUJARATI

www.vedantu.com 15
Question and Answer Key - April 10 Online

Question No. Answer Key


Q61 4
Q62 1
Q63 4
Q64 3
Q65 4
Q66 1
Q67 1
Q68 1
Q69 2
Q70 1
Q71 1
Q72 2
Q73 3
Q74 2
Q75 2
Q76 3
Q77 3
Q78 1
Q79 3
Q80 2
Q81 4
Q82 4
Q83 2
Q84 4
Q85 4
Q86 2
Q87 3
Q88 3
Q89 2
Q90 4

www.vedantu.com
JEE Mains 2018

Physics

1. The density of a material in the shape of a cube is determines by measuring three sides of
the cube and its mass. If the relative errors in measuring the mass and length are respectively
1.5% and 1%, the maximum error in determining the density is :
(1) 2.5%
(2) 3.5%
(3) 4.5%
(4) 6%

Solution:
M

L3
 M 3L
 
  L
 1.5  31
 4.5%
Hence the Solution is Option (3)

2. All the graphs below are intended to represent the same motion. One of them does it
incorrectly. Pick it up.
(1)

(2)

(3)

(4)

Solution:

Set: A www.vedantu.com 1
JEE Mains 2018

(1),(3),(4) uniform retardation & then uniform acceleration.


(2) Normal uniform acceleration & retardation.
Hence the Solution is Option (2)

3. Two masses m1 = 5 kg and m2 = 10 kg, connected by an inextensible string over a


frictionless pulley, are moving as shown in the figure. The coefficient of friction of horizontal
surface is 0.15. The minimum weight m that should be put on top of m2 to stop the motion is :

(1) 18.3 g
(2) 27.3 kg
(3) 43.3 kg
(4) 10.3 kg

Solution:

To stop the motion,


m1 g  T    m1  m2  g
m1
  m2  m

5
  10  m
0.15
 m  23.3kg
Hence the nearest Solution is Option (2)

4. A particle is moving in a circular path of radius a under the action of an attractive potential
k
U   2 . Its total energy is :
2r
k
(1)  2
4a
k
(2)
2a2
(3) Zero
3 k
(4)
2 a2

Set: A www.vedantu.com 2
JEE Mains 2018

Solution:
K
u
2r 2
du  K
But F   3
dr r
‘ – ‘ sign of force implies attractive force
So,
K mv 2

r3 r
2
mv K
  2  EK
2 2r
K K
UT  2  2
2r 2r
0
Hence the Solution is Option (3)

5. In a collinear collision, a particle with an initial speed  0 strikes a stationary particle of the
same mass. If the final total kinetic energy is 50% greater than the original kinetic energy, the
magnitude of the relative velocity between the two particles, after collision, is :
0
(1)
4
(2) 20

(3) 0
2
0
(4)
2

Solution:
By cons of linear momentum,
mv0  mv1  mv2
v0  v1  v2 .....  i 
1 2 1 2 1 2
 mv1  mv2    mv0 
Also,   2  100
2 2
1 2
mv
2 0
 50
1
 v12  v22  v02  v02
2
3
 v12  v22  v02
2
Using (i)

Set: A www.vedantu.com 3
JEE Mains 2018

3
 v0  v2   v22  v02
2

2
1
v22  2v0v2  v12  v02
2
 1 
2v22   2v0  v2   v02   0
 2 
2v0  4v02  4v02
v2 
4
v  2v0
 0
2
v  2v0
If v2  0
2
v  2v0
 v1  0
2
So relative velocity :
 v2  v1  2v0
Hence the Solution is Option (2)

6. Seven identical circular planar disks, each of mass M and radius R are welded
symmetrically as shown. The moment of inertia of the arrangement about the axis normal to
the plane and passing through the point P is :

19
(1) MR2
2
55
(2) MR2
2
73
(3) MR2
2
181
(4) MR2
2

Solution:
Moment of inertia of each disc about the given axis is,

Set: A www.vedantu.com 4
JEE Mains 2018

1 3
I1  MR 2  MR 2  MR 2
2 2
1 19
I 4  MR 2  M  3R   MR 2
2

2 2
1 51
I 7  MR 2  M  3R   MR 2
2

2 2
I net  I1  I 2  I 3  I 4  I 5  I 6  I 7
I net  I1  I 4  I 7
73
I net  MR 2
2
Hence the Solution is Option (3)

R
7. From a uniform circular disc of radius R and mass 9 M, a small disc of radius is
3
removed as shown in the figure. The moment of inertia of the remaining disc about an axis
perpendicular to the plane of the disc and passing through centre of disc is :

(1) 4MR2
40
(2) MR2
9
(3) 10MR2
37
(4) MR2
9

Solution:
Mass of disc = Volume x Density
9M = A  T   (Area x thickness x density)
9M =  R2  t  ......... i 
For the disc which is cut off.
2
R
M '      t  ......  ii 
3

Set: A www.vedantu.com 5
JEE Mains 2018

(i)Divided by (ii)
9M
9M ' M
M
Moment of inertia of complete disc about an axis passes through O.
1
I1   9M   R2
2

Moment of inertia of cut off disc about an axis passes through O


2 2
1 R  2R 
I2  M     M   
2 3  3 
1 MR2 4MR2
 
2 9 9
So, moment of inertia of remaining disc = I1  I 2
9MR 2 MR 2 4MR 2
  
2 18 9
 81  1  8 
  MR
2

 18 
 4MR 2
Hence the Solution is Option (1)

8. A particle is moving with a uniform speed in a circular orbit of radius R in a central force
inversely proportional to the nth power of R. If the period of rotation of the particle is T, then :
(1) T R3/2 for any n.
n
1
(2) T R 2
(3) T R n1/2
(4) T Rn/2

Solution:
F  KR n 1
mv 2 K
So,  n
R R
K 1
v   n 1 /2
M R
2 R 2 R n 1
Now, T   R 2
v K
M
 T  R n 1 /2
Hence the Solution is Option (3)

9. A solid sphere of radius r made of a soft material of bulk modulus K is surrounded by a


liquid in a cylindrical container. A massless piston of area a floats on the surface of the

Set: A www.vedantu.com 6
JEE Mains 2018

liquid, covering entire cross section of cylindrical container. When a mass m is placed on the
surface of the piston to compress the liquid, the fractional decrement in the radius of the
 dr 
sphere,   is :
 r 
Ka
(1)
mg
Ka
(2)
3mg
mg
(3)
3Ka
mg
(4)
Ka

Solution: Bulk Modulus,

P
K
 V / V 
4
For sphere, V   r 3
3
V 3r
 
V r
r P Mg
  
r 3K 3Ka
Hence the Solution is Option (3)

10. Two moles of an ideal monoatomic gas occupies a volume V at 270 C. The gas expands
adiabatically to a volume 2 V. Calculate (a) the final temperature of the gas and (b) change in
its internal energy.
(1) (a) 189 K (b) 2.7 kJ
(2) (a) 195 K (b) -2.7 kJ
(3) (a) 189 K (b) -2.7 kJ
(4) (a) 195 K (b) 2.7 kJ

Solution:
Initially n = 2, v, T= 300k
Finally Vd = 2v
Gas is monoatomic, So, r = 5/3
So,

Set: A www.vedantu.com 7
JEE Mains 2018

nR Ti  T f 
dw 
r 1
r 1
TV
i i  T f V fr 1
2/3
1
Ti     Tf
2
300 300
 Tf    189k
41/3 1.6
Since gas undergoes expensed.
So, d w  0
& d q  du  d w
 du  0.
Hence the Solution is Option (3)

11. The mass of a hydrogen molecule is 3.32 1027 kg. If 1023 hydrogen molecules strike, per
second, a fixed wall of area 2 cm2 at an angle of 450 to the normal, and rebound elastically
with a speed of 103 m/s, then the pressure on the wall is nearly :
(1) 2.35 103 N / m2
(2) 4.70 103 N / m2
(3) 2.35 102 N / m2
(4) 4.70 102 N / m2

Solution:
Change in momentum normal to the wall

 2mv cos 45o

So, Force 
 2mv cos 45   N o

t
1
2  3.32 1027 103  1023
Pr essure  2
2 104 1
P  2.35 103 N / m2
Hence the Solution is Option (1)

12. A silver atom in a solid oscillates in simple harmonic motion in some direction with a
frequency of 1012/sec. What is the force constant of the bonds connecting one atom with the
other ? (Mole wt. of silver = 108 and Avagadro number = 6.02 x 10 23 gm mole-1)
(1) 6.44 N/m

Set: A www.vedantu.com 8
JEE Mains 2018

(2) 7.1 N/m


(3) 2.2 N/m
(4) 5.5 N/m

Solution:
M
T  2
K
4  M
2
K
T2
108 103
 4 
2

6.023 1023
 10 
12 2

10.8
 4 2  102
6.023
 7.1N / m
Hence the Solution is Option (2)

13. A granite rod of 60 cm length is clamped at its middle point and is set into longitudinal
vibrations. The density of granite is 2.7 103 kg/m3 and its young’s modulus is 9.27 1010 Pa.
What will be the fundamental frequency of the longitudinal vibrations ?
(1) 5 kHz
(2) 2.5 kHz
(3) 10 kHz
(4) 7.5 kHz

Solution:
Since rod is clamped at centre. So centre it behaves as node & end it behave as antinode.
So,

 30cm
4
   1.2m
9.27 1010
y
 
 2.7 103
 5.85 103 m / s
5.85 103
So, 
1.2
 5kHz
Hence the Solution is Option (1)

14. Three concentric metal shells A, B and C of respective radii a, b and c (a < b < c) have
surface charge densities  ,  and  respectively. The potential of shell B is :
  a 2  b2 
(1)   c
0  a 

Set: A www.vedantu.com 9
JEE Mains 2018

  a 2  b2 
(2)   c
0  b 
  b2  c 2 
(3)   a
0  b 
  b2  c 2 
(4)   a
0  c 

Solution:
Charge in sphere
A  qA    4 a 2
B  qB    4 b2
C  qc    4 c 2
Potential of B:

qA qb qc
  
4 0 b 4 0 b 4 0 c
 a2 b 2  c 2
  
0 b 0 b 0 c
  a2 
  b  c
0  b 
  a b
2 2

   c
0  b 
Hence the Solution is Option (2)

15. A parallel plate capacitor of capacitance 90 pF is connected to a battery of emf 20 V. If a


5
dielectric material constant K = is inserted between the plates, the magnitude of the
3
induced charge will be :
(1) 1.2 n C
(2) 0.3 n C
(3) 2.4 n C
(4) 0.9 n C

Solution:

Set: A www.vedantu.com 10
JEE Mains 2018

qi  C0V
 90 1012  20
 1800 1012
 1.8nC
5
q f  1.8nC
3
 3nC
So, qind  3nC  1.8nC
 1.2nC
Hence the Solution is Option (1)

16. In an a.c. circuit, the instantaneous e.m.f. and current are given by e = 100 sin 30 t
 
i  20sin  30t   . In one cycle of a.c., the average power consumed by the circuit and the
 4
wattless current are, respectively :
(1) 50, 10
1000
(2) ,10
2
50
(3) ,0
2
(4) 50, 0

Solution:
Pqv  rms  I rms  cos 
100 20  
   cos  
2 2 4
2000
 watt
2 2
1000
 w
2
I w  I rms  cos 
20 1
 
2 2
 10 A
Hence the Solution is Option (2)

17. Two batteries with e.m.f. 12 V and 13 V are connected in parallel across a load resistor of
10𝛺. The internal resistances of the two batteries are 1𝛺 and 2𝛺 respectively. The voltage
across the load lies between :
(1) 11.6 V and 11.7 V
(2) 11.5 V and 11.6 V
(3) 11.4 V and 11.5 V

Set: A www.vedantu.com 11
JEE Mains 2018

(4) 11.7 V and 11.8 V

Solution:

Total
 1  2 
  
emf   1
r r2 
1 1
  
 r1 r2 
 12 13 
  

1 2
1 1 
  
1 2 
37
 Volt
3
So, V    Ir
r

Rr
 11.53V
Hence the Solution is Option (2)

18. An electron, a proton and an alpha particle having the same kinetic energy are moving in
circular orbits of radii re, rp, r𝛼 respectively in a uniform magnetic field B. The relation
between re, rp, r𝛼 is :
(1) re > rp = r𝛼
(2) re < rp = r𝛼
(3) re < rp < r𝛼
(4) re < r𝛼 < rp

Solution:
For change moving in circular orbit is a uniform magnetic field,
mv 2mEK
r 
Bq Bq
B2q2r 2
 EK 
2m
Since all particles came same EK & B,
So,

Set: A www.vedantu.com 12
JEE Mains 2018

q2r 2
 Cons tan t
2m
qe  q p & me  m p
 re  rp
For proton &  -particle
12  rp2 22  r2

2 1 2 4
 rp  r
Hence the Solution is Option (2)

19. The dipole moment of a circular loop carrying a current I, is m and the magnetic field at
the centre of the loop is B1. When the dipole moment is doubled by keeping the current
B
constant, the magnetic field at the centre of the loop is B2. The ratio 1 is :
B2
(1) 2
(2) 3

(3) 2

1
(4)
2

Solution:
Magnetic moment, M  IA  I   r and magnetic field at the centre of circle
2

I
B 0
2R
M f  I   rf2  2M
M i  I   ri 2  M
rf
  2
ri
B1 rf
So,   2
B2 ri
Hence the Solution is Option (3)

1
20. For an RLC circuit driven with voltage of amplitude  m and frequency 0  the
LC
current exhibits resonance. The quality factor, Q is given by :

0 L
(1)
R

Set: A www.vedantu.com 13
JEE Mains 2018

0 R
(2)
L
R
(3)
0C 
CR
(4)
0

Solution:
Qfactor = (voltage across L or C at Resonance)/(voltage across R)

I  X L 0 L
= 
I R R
Hence the Solution is Option (1)

  z 
21. An EM wave from air enters a medium. The electric fields are E1  E01xˆ cos 2 v   t 
  c 
in air and E2  E02 xˆ cos k  2z  ct  in medium, where the wave number k and frequency 𝜐
refer to their values in air. The medium is non-magnetic. If r1 and r2 refer to relative
permitivities of air and medium respectively, which of the following options is correct ?

(1) r1  4
r2
r1
(2) 2
r2
r1 1
(3) 
r2 4
r1 1
(4) 
r2 2

Solution:
  z 
E1  E01xˆ cos 2 v   t 
  c 
E2  E02 xˆ cos k  2z  ct 
Where
2  1
k & 
 c 
So volume in medium 1 = C
Volume in medium 2 = C/2
1
C
 0r1

Set: A www.vedantu.com 14
JEE Mains 2018

C 1

2  0r2
r2
2
r1
r1 1
 
r2 4
Hence the Solution is Option (3)

22. Unpolarized light of intensity I passes through an ideal polarizer A. Another identical
1
polarizer B is placed behind A. The intensity of light beyond B is found to be .Now
2
another identical polarizer C is placed between A and B. The intensity beyond B is now
1
found to be . The angle between polarizer A and C is :
8
0
(1) 0
(2) 300
(3) 450
(4) 600

Solution:
I
Unpolarized light of intensity I, when passed through a polarizer A, its intensity becomes
2
I
Since intensity of light emerging from polarizer B =
2
So, A & B are parallel placed.
Let, C makes angle  with A.

So,
I I
cos 4  
2 8
1
 cos 2  
2
1
 cos  
2
  45 

Hence the Solution is Option (3)

23. The angular width of the central maximum in a single slit diffraction pattern is 60 0. The
width of the slit is 1 𝜇m. The slit is illuminated by monochromatic plane waves. If another
slit of same width is made near it, Young’s fringes can be observed on a screen placed at a

Set: A www.vedantu.com 15
JEE Mains 2018

distance of 50 cm from the slits. If the observed fringe width is 1 cm, what is slit separation
distance ?
(i.e. distance between the centres of each slit.)
(1) 25𝜇m
(2) 50𝜇m
(3) 75𝜇m
(4) 100 𝜇m

Solution:
2 
 600 
d 180
 1
     m
 3 2
In Ydse,
D
B'  106
d'
 1 1
  
 10  
2 2 2 2
106
d'

d' 2
106
10 12
 26.16  25 m
Hence the Solution is Option (1)

24. An electron from various excited states of hydrogen atom emit radiation to come to the
ground state. Let n,g be the de Broglie wavelength of the electron in the nth state and the
ground state respectively. Let  n be the wavelength of the emitted photon in the transition
from the nth state to the ground state. For large n, (A, B are constants)
B
(1)  n  A  2
n
(2) n  A  Bn
(3) 2n  A  Bn2
(4) 2n  

Solution:

Set: A www.vedantu.com 16
JEE Mains 2018

h h
 
mv 2MEk
h2
Ek 
2m 2
For emitted photon,
En  E1
hc

n
h2 h2 hc
  
2mn 2mg  n
2 2

hc h2  h 2 
  K K  
 n 2mn2  2mg2 
  2 n  A  Bn2
Hence the Solution is Option (3)

25. If the series limit frequency of the Lyman series is 𝜐L , then the series limit frequency of
the Pfund series is :
(1) 25vL
(2) 16vL
v
(3) L
16
v
(4) L
25

Solution:
1  1 1
 R 2  2   v
  n1 n2 
1 1
R  2   vl & R  2   v p
1  5 
v 1
 l 
v p 25
vl
 vp 
25
Hence the Solution is Option (4)

26. It is found that if a neutron suffers an elastic collinear collision with deuterium at rest,
fractional loss of its energy is pd; while for its similar collision with carbon nucleus at rest,
fractional loss of energy is pc. The values of pd and pc are respectively :
(1) (.89, .28)
(2) (.28, .89)

Set: A www.vedantu.com 17
JEE Mains 2018

(3) (0, 0)
(4) (0, 1)

Solution: U1 , U2=0
Since collision is elastic,
u  M  M 2   2 M 1M 2
1  1 1
M1  M 2
But , u2  0
1 M1  M 2
  .............  i 
u1 M1  M 2
Fractional loss in kinetic energy of neutron,
1 1
M 1u12  M 112
2 2
1
M 1u12
2
2
 
 1  1 
 u1 
So, A.TQ;
 1 2  8
2

Pd  1      0.89
 1 2  9
 1  12 
2

Pc  1     0.28
 1  12 
Hence the Solution is Option (1)

27. The reading of the ammeter for a silicon diode in the given circuit is :

(1) 0
(2) 15 mA
(3) 11.5 mA
(4) 13.5 mA

Solution:
The given diode is a forward bias & hence behaves as a perfect conductor i.e., it offers zero
resistance.
So,
V 3
I   1102 A
R 200
 15mA

Set: A www.vedantu.com 18
JEE Mains 2018

Hence the Solution is Option (2)

28. A telephonic communication service is working at carrier frequency of 10 GHz. Only


10% of it is utilized for transmission. How many telephonic channels can be transmitted
simultaneously if each channel requires a bandwidth of 5 kHz ?
(1) 2 103
(2) 2 104
(3) 2 105
(4) 2 106

Solution:
Required number of channels
10
10 109
 100
5 103
 2 105
Hence the Solution is Option (3)

29. In a potentiometer experiment, it is found that no current passes through the galvanometer
when the terminals of the cell are connected across 52 cm of the potentiometer wire. If the
cell is shunted by a resistance of 5𝛺, a balance is found when the cell is connected across 40
cm of the wire. Find the internal resistance of the cell.
(1) 1𝛺
(2) 1.5𝛺
(3) 2𝛺
(4) 2.5𝛺

Solution:

l 
r   1  1 R
 l2 
 52 
   1  5
 40 
12  5

40
 1.5
Hence the Solution is Option (2)

Set: A www.vedantu.com 19
JEE Mains 2018

30. On interchanging the resistances, the balane point of a meter bridge shifts to the left by 10
cm. The resistance of their series combinations is 1 k𝛺. How much was the resistance on the
left slot before interchanging the resistances ?
(1) 990𝛺
(2) 505𝛺
(3) 550𝛺
(4) 910𝛺

Solution:

P l

Q 100  l
Q l  10
& 
P 110  l
l 110  l
 
100  l l  10
 l 2  10l  11000  100l  100l  l 2
 200l  11000
 l  55cm
l  100  l 
  1000
A A

  10
A
l
So,  10  55
A
 550
Hence the Solution is Option (3)

Chemistry

31. The ratio of mass present of C and H of an organic compound (CXHYOZ) is 6 : 1. If one
molecule of the above compound (CXHYOZ) contains half as much oxygen as required to burn
one molecule of compound CXHY completely to CO2 and H2O. The empirical formula of
compound CXHYOZ is :
(1) C3H6O3
(2) C2H4O
(3) C3H4O2
(4) C2H4O3

Set: A www.vedantu.com 20
JEE Mains 2018

Solution:
CxHyOz – Organic compound
mass of C
 6 :1
mass of H
mass of C / 12 6 1
 :
mass of H / 1 12 2
No.of moles of C 1
  :1
No.of moles of H 2
moles of C
Cx H yOz 
Moles of H
 y y
Cx H y   x   o2  xco2  H 2o
 x 2
 y x y
cx hy oz  x    o2  x co 2  H 2O
 4 2 x
1 y y z
x  x 
2 4 4 2
x y y
 2x   Z
4 2
y
Z  x
4
4
c2 h4o3 Z  2   3
4
Hence the answer is option (4).

32. Which type of ‘defect’ has the presence of cations in the interstitial sites ?
(1) Schottky defect
(2) Vacancy defect
(3) Frenkel defect
(4) Metal deficiency defect

Solution:
Frankel defect has the presence of cation in interstitial site.
Hence the answer is option (3).

33. According to molecular orbital theory, which of the following will not be a viable
molecule ?
(1) H22
(2) He2

Set: A www.vedantu.com 21
JEE Mains 2018

(3) H 2
(4) H22

Solution :
*
H22 electronic configuration  1S 2 ,  1S 2
1 1
Number of electrons = 4 ; Bo   Nb  Na    2  2  0
2 2
Molecule does not exist.
Hence the answer is option (4).

34. Which of the following lines correctly show the temperature dependence of equilibrium
constant K, for an exothermic reaction?

(1) A and B
(2) B and C
(3) C and D
(4) A and D

Solution:
H o S o
ln K    Re lationbetween Rate cons tan t 
RT R
For exothermic reaction H = -ve.
H o
Slope  0
R
Hence the answer is option (1).

35. The combustion of benzene (1) gives CO2(g) and H2O(I). Given that heat of combustion
of benzene at constant volume is -3263.9 kJ mol-1 at 250 C; heat of combustion (in kJ mol-1)
of benzene at constant pressure will be :
(1) 4152.6
(2) -452.46
(3) 3260
(4) -3267.6

Solution:

Set: A www.vedantu.com 22
JEE Mains 2018

15
C6 H 6  l  O2  g   6CO2  3H 2O  l 
2
ng  6  7.5  1.5
H  U  ngRT
 3263.9  1.5  8.134 103  298
 3267.6

Hence the answer is option (4).

36. For 1 molal aqueous solution of the following compounds, which one will show the
highest freezing point?
(1) Co  H 2O 6  Cl3
(2) Co  H2O5 Cl  Cl2 .H2O

(3) Co  H2O 4 Cl2  Cl.2H 2O


(4) Co  H2O3 Cl3  .3H2O

Solution:
Tf  i  kf  m
Tf  i
1
Tf 
i
Co  H2O3 Cl3  .3H2O ; i = 1
Co  H 2O 6  Cl3 ; i = 2
Co  H2O5 Cl  Cl2 .H2O ; i = 4
Co  H2O 4 Cl2  Cl.2H 2O ; i = 3
Hence the answer is option (4).

37. An aqueous solution contains 0.10 M H2S and 0.20 M HCl. If the equilibrium constants
for the formation of HS- from H2S is 1.0 x 10-7 and that of S2- from HS- ions is 1.2 x 10-13 then
the concentration of S2- ions in aqueous solution is :
(1) 5 x 10-8
(2) 3 x 10-20
(3) 6 x 10-21
(4) 5 x 10-19

Solution:

Set: A www.vedantu.com 23
JEE Mains 2018


H 2 S  2 H   S 2
ka1 .ka2

0.1  x 0.2 x
 0.2 
 52
2

ka1.ka2 
0.1
2
1.2 10  0.1
  S 2 
0.04
 S 2   3 1020
Hence the answer is option (2).

38. An aqueous solution contains an unknown concentration of Ba2+. When 50 mL of a 1M


solution of Na2SO4 is added, BaSO4 just begins to precipitate. The final volume is 500 mL.
The solubility product of BaSO4 is 1 x 10-10. What is the original concentration of Ba2+ ?
(1) 5 x 10-9 M
(2) 2 x 10-9 M
(3) 1.1 x 10-9 M
(4) 1.0 x 10-10 M

Solution:
KS p  BaSO4   1010
QS p  KS p
 Ba  SO   10
2 2
4
10

50
Ba 2  1010
500
  Ba 2   109
m1v1  m2 v2
m  450  109  500
500
m   109
450
m  1.1109
Hence the answer is option (3).

39. At 5180 C, the rate of decomposition of a sample of gaseous acetaldehyde, initially ar a


pressure of 363 Torr, was 1.00 Torr s-1 when 5% had reacted and 0.5 Torr s-1 when 33% had
reacted. The order of the reaction is :
(1) 2
(2) 3
(3) 1
(4) 0

Solution:

Set: A www.vedantu.com 24
JEE Mains 2018

r  k  p  m = order of reaction
m

m
 95 
1  k 363  ............1
 100 
 67 
0.5  k  362   ........... 2 
 100 
equation  1 / Equation  2
m
1  95 

0.5  67 
m
 95 
2 
 67 
2   0.4 
m

log 2  m log1.4
0.3010
m
log1.4
0.3010
m
0.15
m2
Hence the answer is option (1).

40. How long (approximate) should water be electrolysed by passing through 100 amperes
current so that the oxygen released can completely burn 27.66 g of diborane?
(Atomic weight of B = 10.8 u)
(1) 6.4 hours
(2) 0.8 hours
(3) 3.2 hours
(4) 1.6 hours

Solution:

Cathode: H2O  2e  H 2  2OH

 Re duction
Anode:
2 H 2O  4 H   O2  4e  oxidation 
B2 H 6  3O2  B2O3  3H 2O
27.66 g
27.66
n 1
276
Moles of O2 required = 3

Set: A www.vedantu.com 25
JEE Mains 2018

Gram equivalent of O2 = 3  4  12
Gram equivalent
it
w
96500
100  t
12 
96500
12  965
t sec
60  60
t  3.2hr
Hence the answer is option (3).

41. The recommended concentration of fluoride ion in drinking water is up to 1 ppm as


fluoride ion is required to make teeth enamel harder by converting 3Ca3  PO4 2 .Ca OH 2 
:
(1) CaF2 
(2) 3  CaF2  .Ca  OH 2 
(3) 3Ca3  PO4 2 .CaF2 

 
(4) 3 Ca  OH 2 .CaF2 
 

Solution:
3Ca3  PO4 2 .CaF2 
Hence the answer is option (3).

42. Which of the following compounds contain(s) no covalent bond(s) ?


KCl, PH3, O2, B2H6, H2SO4
(1) KCl, B2H6, PH3
(2) KCl, H2SO4
(3) KCl,
(4) KCl, B2H6

Solution:
KCl is an ionic compound. It cannot from covalent bond. Elements of s – block & p – block
combine to form ionic compounds.
Hence the answer is option (3).

43. Which of the following are Lewis acids ?


(1) PH3 and BCl3
(2) AlCl3 and SiCl4
(3) PH3 and SiCl4

Set: A www.vedantu.com 26
JEE Mains 2018

(4) BCl3 and AlCl3

Solution:

BCl3 and AlCl3 are electron deficient compounds. Boron and aluminium has 6 electrons in
their valence shell in BCl3 & AlCl3,. PH3, SiCl4 have 8 electrons in their valence shell. They
are not Lewis acids.
Hence the answer is option (4).

44. Total number of lone pair of electrons in I 3 ion is :


(1) 3
(2) 6
(3) 9
(4) 12

Solution:
The total number of lone pair of electrons is I3 is 9

Hence the answer is option (1).

45. Which of the following salts is the most basic in aqueous solution?
(1) Al(CN)3
(2) CH3COOK
(3) FeCl3
(4) Pb(CH3COO)2

Solution: CH3COOK is a salt of weak acid and strong base


Hydrolysis of potassium acetate gives strong base KOH
Hence the answer is option (2).

4 3
46. Hydrogen peroxide oxides  Fe  CN 6  to  Fe CN 6  in acidic medium but reduces
3 4
 Fe CN 6  to  Fe  CN 6  in alkaline medium. The other products formed are,
respectively:
(1)  H 2O  O2  and H2O


(2)  H 2O  O2  and H2O  OH  
(3) H2O and  H 2O  O2 


(4) H2O and H2O  OH  

Set: A www.vedantu.com 27
JEE Mains 2018

Solution:
4 3
 Fe 2  CN   H 2O2
  Fe   CN 6   H 2O
3
 6 Oxidizing agent

reducing agent
 oxidi sin g agent  H 2O21  H 2O2  Re duction 
 Re ducing agent  H2O21  O2  oxidation 
3 4
 Fe 3  CN   H 2O2
  Fe 2  CN    O2
 6  Oxidizing agent  6

 oxiding agent 
Hence the answer is option (3).

47. The oxidation states of Cr in Cr  H2O6  Cl3 , Cr C6 H6 2  , and
K2 Cr  CN 2  O 2  O2  NH3  respectively are :
(1) +3, +4, and +6
(2) +3, +2, and +4
(3) +3, 0, and +6
(4) +3, 0, and +4

Solution:
The Oxidation state of Cr in
Cr  H 2O 6  Cl3
x  6  0  3  0 .
x  3

Cr  C6 H 6 2 
x  2  0  0
x0

K 2 Cr  CN 2  O 2  O2  NH 3 
2  x  2  1  2  2   1  2   0
6
 x  6
Hence the answer is option (3).

48. The compound that does not produce nitrogen gas by the thermal decomposition is :
(1) Ba(N3)2
(2) (NH4)2Cr2O7

Set: A www.vedantu.com 28
JEE Mains 2018

(3) NH4NO2
(4) (NH4)2SO4

Solution:

NH 4 NO2   N 2  H 2O
 NH 4 2 Cr2O7 
 N 2  H 2O  Cr2O3
Ba  N 3 2 

 Ba  N 2
Hence the answer is option (4).

49. When metal ‘M’ is treated with NaOH, a white gelatinous precipitate ‘X’ is obtained,
which is soluble in excess of NaOH. Compound ‘X’ when heated strongly gives an oxide
which is used in chromatography as an adsorbent. The metal ‘M’ is :
(1) Zn
(2) Ca
(3) Al
(4) Fe

Solution:
The Gelatinous precipitate formed in Al(OH)3 ; Al(OH)3 on strong heating gives Al2O3 which
is used in chromatography as an adsorbent. So the metal is Al.
Hence the answer is option (3).

50. Consider the following reaction and statements:



Co  NH3 4 Br2   Br   Co  NH3 3 Br3   NH3
(I) Two isomers are produced if the reactant complex ion is a cis-isomer.
(II) Two isomers are produced if the reactant complex ion is a trans-isomer.
(III)Only one isomer is produced if the reactant complex ion is a trans-isomer.
(IV) Only one isomer is produced if the reactant complex ion is a cis-isomer.
The correct statements are :
(1) (I) and (II)
(2) (I) and (III)
(3) (III) and (IV)
(4) (II) and (IV)

Solution:

Set: A www.vedantu.com 29
JEE Mains 2018

As all the NH3 positions are identical


only one product can be formed.

Hence the answer is option (2).

51. Glucose on prolonged heating with HI gives :


(1) n-Hexane
(2) 1-Hexane
(3) Hexanoic acid
(4) 6-iodohexanal

Solution:
C6 H12O6 
HI
CH3   CH 2 4  CH3
Glu cos e n  hexane
Hence the answer is option (1).

52. The trans-alkenes are formed by the reduction of alkynes with :


(1) H2 – pd/C, BaSO4
(2) NaBH4
(3) Na/liq. NH3
(4) Sn-HCl

Solution:
Trans alkenes are formed by the reaction of alkynes with Na/liq. NH3
(birch Reduction)

Set: A www.vedantu.com 30
JEE Mains 2018

Hence the answer is option (3).

53. Which of the following compounds will be suitable for Kjeldahl’s method for nitrogen
estimation?

Solution:
Kjeldahl’s method:-

Organic compounds nitrogen +


Conc.H 2 SO4   NH 4 2 SO4
 alkali
NH3
Nitro compounds, A2O compounds & Nitrogen part of the aromatic ring will not give
positive result for Kjeldahl’s method.
Aniline is the best suitable to estimate nitrogen using Kjeldahl’s method.
Hence the answer is option (2).

54. Phenol on treatment with CO2 in the presence of NaOH followed by acidification
produces compound X as the major product. X on treatment with (CH3CO)2O in the presence
of catalytic amount of H2SO4 produces :
(1)

(2)

Set: A www.vedantu.com 31
JEE Mains 2018

(3)

(4)

Solution:

Hence the answer is option (1).

55. An alkali is titrated against an acid with methyl orange as indicator, which of the
following is a correct combination?

Solution:
When a weak base is titrated with string acid with methyl orange as an indicator then at end
point
The colour change will be yellow to pinkish red.
Hence the answer is option (3).

Set: A www.vedantu.com 32
JEE Mains 2018

56. The predominant form of histamine present in human blood is (pk a , Histidine = 6.0)
(1)

(2)

(3)

(4)

Solution:

Hence the answer is option (4).

57. Phenol reacts with methyl chloroformate in the presence of NaOH to form product A.
A reacts with Br2 to form product B. A and B are respectively :
(1)

(2)

(3)

(4)

Set: A www.vedantu.com 33
JEE Mains 2018

Solution:

Hence the answer is option (3).

58. The increasing order of basicity of the following compounds is :


(a)

(b)

(c)

(d)

(1) (a) < (b) < (c) < (d)


(2) (b) < (a) < (c) < (d)
(3) (b) < (a) < (d) < (c)
(4) (d) < (b) < (a) < (c)

Solution:

The lone pair on the nitrogen atom is in conjugation with the  bond hence it can involve in
resonance.
during resonance the other nitrogen attains a negative charge.
Hence ‘c’ is a strong base.

Set: A www.vedantu.com 34
JEE Mains 2018

The +I group of CH3 and C2H5 makes ‘d’ more basic but less basic than ‘c’

Due to -I of  bond, basic character decreases.

The nitrogen involves sp2 hybridization which is highly electro negative have least basic.
Hence the answer is option (4).

59. The major product formed in the following reaction is :

(1)

(2)

(3)

(4)

Solution:

Set: A www.vedantu.com 35
JEE Mains 2018

Hence the answer is option (4).

60. The major product of the following reaction is :

(1)

(2)

(3)

(4)

Solution:


NaOMe
MeOH

E2 reaction

Hence the answer is option (2).

Set: A www.vedantu.com 36
JEE Mains 2018

Mathematics

61. Two sets A and B are as under :


A={(a, b)  R  R : a  5  1 and b  5  1};
B={(a, b)  R  R : 4(a  6)2  9(b  5)2  36 }. Then :
(1) B A
(2) A B
(3) A  B   (an empty set)
(4) neither A  B nor B  A

62. Let S= {x  R : x  0 and 2 x  3  x ( x  6)  6  0} . Then S :


(1) is an empty set.
(2) contains exactly one element.
(3) contains exactly two element.
(4) contains exactly four elements.

Solution:
2 | x  3|  x  
x 6 6 0
Case-I x 3
2  
x  3  x  6 x  6  0  x  4 x  0  x  0,6
As x  9  x  16
Case  II x 3 2 x 6 x6 x 6  0

 x 6  
x  2  0  x  36,4

As, x  3  x  4
There are exactly two elements in the given set.

63. If  ,   c are the distinct roots, of the equation x2  x 1  0 , then  101   107 is equal
to :
(1) -1
(2) 0
(3) 1
(4) 2

Solution:
x2 – x+ 1 = 0

Set: A www.vedantu.com 37
JEE Mains 2018

1  3
x   ,  2
2
    and    2
       2   101   214     2     1
101 107

x  4 2x 2x
64. If 2 x x  4 2 x = ( A  Bx)( x  A)2 , then the ordered pair (A, B) is equal to :
2x 2x x  4
(1) (-4, -5)
(2) (-4, 3)
(3) (-4, 5)
(4) (4, 5)

65. If the system of linear equations


x  ky  3z  0
3x  ky  2 z  0
2 x  4 y  3z  0
xz
has a non-zero solution (x, y, z), then is equal to:
y2
(1) -10
(2) 10
(3) -30
(4) 30

66. From 6 different novels and 3 different dictionaries, 4 novels and 1 dictionary are to
be selected and arranged in a row on a shelf so that the dictionary is always in the
middle. The number of such arrangements is :
(1) at least 1000
(2) less than 500
(3) at least 500 but less than 750
(4) at least 750 but less than 1000

67. The sum of the co-efficients of all odd degree terms in the expansion of

x    
5 5
x3  1  x  x3  1 ,( x  1) is:
(1) -1
(2) 0
(3) 1
(4) 2

Set: A www.vedantu.com 38
JEE Mains 2018

12
68. Let a1 , a2 , a3 ,......, a49 be in A.P. such that a
k 0
4 k 1  416 and a9  a43  66 .

If a12  a22  .....  a172  140m , then m is equal to:


(1) 66
(2) 68
(3) 34
(4) 33

Solution:
a1  a5  a9  416 a  24d  32 ..............i 
a9  a43  66  a  25d  33 ...............ii 
From (i) and (ii) d = 1 and a = 8
Now, a12  a22  .......  a172  140m
17 17
 8   r  1   7  r   140m  m  34
2

r 1 r 1

69. Let A be the sum of the first 20 terms and B be the sum of the first 40 terms of
the series
12  2 22  32  2 42  52  2 62  .....
If B-2A=100  , then  is equal to:
(1) 232
(2) 248
(3) 464
(4) 496

Solution:
A = 12 + 2.22 + 32 + 2.42+ ………+ A2+2.202
= (12 + 2.22 + 32 + 42+ ………+202)+ (22+ 42+ ………+ 202)
20  21 41 10 11 21
  4  2870  1540  4410  2870  1540  4410
6 6
40  41 81 4  20  21 41
B   540  41  41 280  41 820  33620
6 6
33620  8820  100
100  24800
  248

70. For each t  R, let [t] be the greatest integer less than or equal to t. Than
 1 2 15  
lim x        ......    
x0
 x  x  x 

(1) is equal to 0

Set: A www.vedantu.com 39
JEE Mains 2018

(2) is equal to 50
(3) is equal to 120
(4) does not exist (in R)

71. Let S={t  R:f(x)= x   • (e  1)sin x is not differentiable at t}. Then the set S is
x

equal to:
(1)  (an empty set)
(2) {0}
(3) {}
(4) {0, }

72. If the curve y2 = 6x, 9x2 + by2 = 16 intersect each other at right angles, then the
value of b is:

(1) 6
7
(2)
2
(3) 4
9
(4)
2

Solution:
2yy’= 6
6 3
y'  
2 y y1
18 x1 9 x1 27 x 27 x
18 x1     21  1  b  2 1
2by1 by1 by1 y1
9
y12  6 x1  b 
2

1 1 f ( x)
73. Let f ( x)  x2 
2
and g ( x)  x  , x  R  1,0,1 . If h( x)  , then the local
x x g ( x)
minimum value of value of h( x) is :

(1) 3
(2) -3
(3) 2 2
2
(4) 2

Set: A www.vedantu.com 40
JEE Mains 2018

sin 2 x cos2 x
 dx
sin x  cos3 x sin 2 x  sin 3 x cos2 x  cos5 x 
2
74. The integral 5 is equal to :

1
(1) c
3 1  tan 3 x 
1
(2) c
3 1  tan 3 x 
1
(3) c
1  cot 3 x
1
(4) c
1  cot 3 x

Solution:
sin 2 x cos2 x
 dx
sin  cos x sin 2 x  sin3 x cos2 x  cos2 x  cos5 x 
5 3 2

tan 2 x sec6 x
  tan5 x  tan2 x  tan3 x  1 dx
dt
Put tanx = t  sec2 x 
dx
t 2 1  t 2 
2

 dt
t 3  1 t 2  1
2 2

T3 +1=y
dy
3t 2 
dt
1 dy 1 1

3 y 2

3 y 
C  
3  tan 3 x  1
C


2
sin 2 x
75. The value of 
 1+2x
dx is:
2


(1)
8

(2)
2
(3) 4

(4)
4

Set: A www.vedantu.com 41
JEE Mains 2018

76. Let g ( x)  cos x2 , f ( x)  x , and  ,      be the roots of the quadratic


equation 18x2  9 x   2  0 . Then the area (in sq. units) bounded by the curve
y  ( gof )( x) and the lines x   , x   and y  0 , is :

(1)
1
2
 3 1 
(2)
1
2
 3 1 
(3)
1
2
 3 2 
(4)
1
2
 2 1 
Solution:
g  x   cos x 2
f  x  x
g  f  x    cos x
given, 18 x 2  9 x   2  0   6 x    3x     0
 
x ,
6 3
 /3
3 1
Area 

 cos x dx 
/6 2

77. Let y=y(x) be the solution of the differential equation


dy  
sin x  y cos x =4 x , x  (0, ) . If y    0 , then y   is equal to :
dx 2 6
4
(1) 2
9 3
8 2
(2) 
9 3
8
(3)  2
9
4
(4)  2
9

Solution:
dy
 y cot x  4x cos ec x  d  y sin x   4xdx
dt
Integrating both sides we get y sin x  2x2  C
   2
Also, y    0  c  
2 2

Set: A www.vedantu.com 42
JEE Mains 2018

2   8 2
 y sin x  2x2   y   
2 6 9

78. A straight line through a fixed point (2, 3) intersects the coordinate axes at distinct
points P and Q. If O is the origin and the rectangle OPRQ is completed, then the locus of
R is:
(1) 3x  2 y  6
(2) 2x  3y  xy
(3) 3x  2 y  xy
(4) 3x  2 y  6xy

Solution:
Let R = (h,k)
P = (0, k)
Q = (h,0)
Equation of line would be,

x y
  1 ........ i 
h k
2 3
  1
h k
2k + 3h = hk
Locus of (h, k) is 2y + 3x = xy

79. Let the orthocenter and centroid of a triangle be A(-3, 5) and B(3, 3) respectively. If
C is the circumcentre of this triangle, than the radius of the circle having line segment
AC as diameter, is :
(1) 10
2
(2) 10

5
(3) 3
2

3 5
(4)
2

Set: A www.vedantu.com 43
JEE Mains 2018

80. If the tangent at (1,7) to the curve x2  y  6 touches the circle


x2  y2  16x  12 y  c  0 than the value of c is :

(1) 195
(2) 185
(3) 85
(4) 95

81. Tangent and normal are drawn at P(16, 16) on the parabola y 2  16 x , which intersect
the axis of the parabola at A and B, respectively. If C is the centre of the circle through
the points P, A and B and CPB   , then a value of tan  is :
1
(1)
2
(2) 2
(3) 3
4
(4)
3

Solution:
The equation of tangent at P

1
y  16   x  16  A   16, 0
2
The normal is y = y – 16 = -2(x – 16)
B = (24, 0)

Since APB 
2
AB is the diameter
Centre of the circle C = (4, 0)
Slope of PB = -2 = m1
4 m  m1
Slope of CP =  m2  tan  2 2
3 1  m2m1

82. Tangents are drawn to the hyperbola 4x2  y2  36 at the points P and Q. If these
tangents intersects at the point T(0, 3) then the area (in sq. units) of  PTQ is :
(1) 45 5
(2) 54 3
(3) 60 3
(4) 36 5

Set: A www.vedantu.com 44
JEE Mains 2018

Solution:
Equation of PQ,
4x.(0) -3y = 36

Y = -12
1
Area of TPQ   15  6 5  45 5
2

83. If L1 is the line of intersection of the planes 2x  2 y  3z  0, x  y  z  0 and L2 is


the line of intersection of the planes x  2 y  z  3  0,3x  y  2z  0 then the distance of
the origin from the plane, containing the lane L1 and L2 , is :

1
(1) 4 2
1
(2) 3 2
1
(3) 2 2
1
(4) 2

84. The length of the projection of the line segment joining the points (5, -1, 4) and (4, -
1, 3) on the plane, x + y + z = 7 is :
2
(1)
3
2
(2)
3
1
(3)
3
2
(4)
3

85. Let u be a vector coplanar with the vectors a  2i  3 j  k and b  j  k . If u is


perpendicular to a and u . b = 24, than equal to :

(1) 336
(2) 315
(3) 256
(4) 84

Set: A www.vedantu.com 45
JEE Mains 2018

Solution:
 
u a  b  0; u.a  0 and u.b  24

   
Let b  b.a a.  b.u u

  b.a    b. u 
2 2 2
b
2

  b. u
2 2
b  b.a  2
u

2  24 
2
2
2  2
 u  336
7 u

86. A bag contains 4 red and 6 black balls. A ball is drawn at random from the bag, its
colour is observed and this ball along with two additional balls of the same colour is
returned to the bag. If now a ball is drawn at random from the bag, then probability that
this drawn ball is red, is:

3
(1)
10
2
(2)
5
1
(3)
5
3
(4)
4

9 9
87. If  (x  5)  9 and  (x  5)
i 1
i
i 1
i
2
 45 , then the standard deviation of the 9 items

x1, x2, …., x9 is:


(1) 9
(2) 4
(3) 2
(4) 3

     1
88. If sum of all the solution of equation 8cos x.  cos   x  .cos   x     1 in
 6  6  2
[0,  ] is k ,then k is equal to :
2
(1)
3
13
(2)
9

Set: A www.vedantu.com 46
JEE Mains 2018

8
(3)
9
20
(4)
9

89. PQR is a triangular park with PQ=PR=200 m. AT.V. tower stands at the mid-point
of QR. If the angle of elevation of the top of the tower at P,Q and R are respectively 450 ,
300 and 300 then the height of tower (in m) is :
(1) 100
(2) 50
(3) 100 3
(4) 50 2

Solution:

h 1

x 3
x  3h
200  3h2  h2
4h2   200 
2

4h2  40000
h  100

90. The Boolean expression ( p  q)  ( p  q) is equivalent to:


(1) p
(2) p
(3) q
(4) q

Solution:
( p  q)  ( p  q)
P q ( p  q) pq
T F F F
T F F F
F T F T
F F T F

Set: A www.vedantu.com 47
JEE (Main) 2018
BOOKLET CODE : A
Q.No. Answer Q.No. Answer Q.No. Answer
1. (3) 31. (4) 61. (2)
2. (2) 32. (3) 62. (3)
3. (2) 33. (4) 63. (3)
4. (3) 34. (1) 64. (3)
5. (2) 35. (4) 65. (2)
6. (4) 36. (4) 66. (1)
7. (1) 37. (2) 67. (4)
8. (3) 38. (3) 68. (3)
9. (3) 39. (1) 69. (2)
10. (3) 40. (3) 70. (3)
11. (1) 41. (3) 71. (1)
12. (2) 42. (3) 72. (4)
13. (1) 43. (4)* 73. (4)
14. (2) 44. (3) 74. (2)
15. (1) 45. (2) 75. (4)
16. (2) 46. (3) 76. (1)
17. (2) 47. (3) 77. (3)
18. (2) 48. (4) 78. (3)
19. (3) 49. (3) 79. (3)
20. (1) 50. (2) 80. (4)
21. (3) 51. (1) 81. (2)
22. (3) 52. (3) 82. (1)
23. (1) 53. (2) 83. (2)
24. (1) 54. (1) 84. (4)
25. (4) 55. (3) 85. (1)
26. (1) 56. (4) 86. (2)
27. (3) 57. (3) 87. (3)
28. (3) 58. (3) 88. (2)
29. (2) 59. (4) 89. (1)
30. (3) 60. (2) 90. (1)
Vedantu Innovations Pvt. Ltd.
Score high with a personal teacher, Learn LIVE Online!
www.vedantu.com
JEE Main - 2018 (CBT)
Exam Test Date: 16/04/2018

Part - A (Physics)

1. The relative uncertainty in the period of a satellite orbiting around the earth is 10–2. If the relative
uncertainty in the radius of the orbit is negligible the relative uncertainty in the mass of the earth is :

(1) 2 × 10–2 (2) 6 × 10–2 (3) 3 × 10–2 (4) 10–2


Ans. (1)
Sol. From kepler's Law
4 2 3
T2  r
GM
M T
2 = 2 × 10–2
M T

2. At some instant a radioactive sample S1 having an activity 5 Ci has twice the number of nuclei as
another sample S2 which has an activity of 10 Ci. The half lives of S1 and S2 are :
(1) 5 years and 20 years, respectively (2) 20 years and 5 years, respectively
(3) 20 years and 10 years, respectively (4) 10 years and 20 years, respectively
Ans. (1)
Sol. Given : N1 = 2N2
n2
1N1 =  N1  5ci
t1
n2
2N2 =  N2  10ci
t2
t 2 N1 1
 
t1 N2 2
t2 1

t1 4
Hence 5years and 20 year

CP 3
3. Two moles of helium are mixed with an moles of hydrogen. If  for the mixture then the value of n is
CV 2
(1) 1 (2) 3 (3) 2 (4) 3/2
Ans. (3)
Cp fmix  2 3
Sol.  
Cv fmix 2
 f mix = 4
n f n f
fmix  11 2 2
n1  n2
4  2  3  n2  5
  n2 = 2mole
2  n2

www.vedantu.com 1
4. Unpolarized light of intensity I is incident on a system of two polarizers, A followed by B. The intensity of
emergent light is I/2. If a third polarizer C is placed between A and B the intensity of emergent light is
reduced to I/3. The angle between the polarizers A and C is , then
1/ 4 1/ 4 1/ 2 1/ 2
 2  1  1  2
(1) cos  =   (2) cos  =   (3) cos  =   (4) cos  =  
3
  3 3 3
Ans. (1)
Sol. A and B have same alignment of transmission axis.
Lets assume c is introduced at  angle
I I
cos2   cos2  
2 3
1/ 4
2 2
cos4 =  cos =  
3 3

5. The de-Broglie wavelength (B) associated with the electron orbiting in the second excited state of
hydrogen atom is related to that in the ground state (G) by :
(1) B = 3G (2) B = 2G (3) B = 3G/3 (4) B = 3G/2
Ans. (1)
B Pa mv G
Sol.  
 G PB mvB
z B nB 3
V× So  
n  G nG 1
B = 3G
Length of Orbit = n × 
2r 1
=  
n n

6. In the given circuit the current through zener diode is :

R1 500

15V

1500 R2 VZ = 10V

(1) 3.3mA (2) 2.5mA (3) 5.5mA (4) 6.7mA


Ans. (1)
5
Sol. Current in R1 = I1 =
500
I1 = 10 mA
10 20
Current in R2 = I2 =  I2 =
mA
1500 3
 20  10
Current in zener diode = I1 – I2 =  10   mA  mA
 3  3

www.vedantu.com 2
7. The end correction of a resonance column is 1cm. If the shortest length resonating with the tuning fork
is 10cm, the next resonating length should be :
(1) 32cm (2) 40cm (3) 28cm (4) 36cm
Ans. (1)
Sol. Given : e = 1 cm
For first resonance

  1  e  11cm
4
For second resonance
3
  1  e  2 = 3 × 11 – 1 = 32 cm
4

8. Two sitar strings A and B playing the note 'Dha' are slightly out of tune and produce beats of frequency
5Hz. The tension of the string B is slightly increased and the beat frequency is found to decrease by
3Hz. If the frequency of A is 425 Hz. the original frequency of B is :
(1) 428 Hz (2) 430 Hz (3) 422 Hz (4) 420 Hz
Ans. (4)
Sol. Frequency of B is either 420Hz or 430Hz As tension in B is increased its frequency will increase.
If frequency is 430Hz, beat frequency will increase
If frequency is 420 Hz beat frequency will decrease, hence correct answer is 420Hz

9. A power transmission line feeds input power at 2300V to a step down transformer with its primary
windings having 4000 turns giving the output power at 230V. If the current in the primary of the
transformer is 5A and its efficiency is 90% the output current would be :
(1) 45A (2) 50A (3) 20A (4) 25A
Ans. (1)
P
Sol. Efficiency n = 0.9 = s
PP
Vs Is = 0.9 × VP IP
0.9  2300  5
Is = = 45A
230

10. A body of mass m starts moving from rest along x-axis so that its velocity varies as  = a s where a is
a constant and s is the distance covered by the body. The total work done by all the forces acting on the body
in the first t seconds after the start of the motion is :
1 1
(1) 8ma4t2 (2) ma4t2 (3) 4ma4t2 (4) ma4t2
4 8
Ans. (4)
ds
Sol. v= a s
dt
2 s  at
a2t 2
S
4
a2
F = m
2
ma 2 a2 t 2 1
Work =  = ma 4 t 2
2 4 8

www.vedantu.com 3
11. Suppose that the angular velocity of rotation of earth is increased. Then as a consequence:
(1) Weight of the object every where on the earth will decrease
(2) Weight of the object every where on the earth will increase
(3) Except at poles weight of the object on the earth will decrease
(4) There will be no change in weight anywhere on the earth.
Ans. (3)
Sol. g' = g – 2Rcos2
Where  is latitude there will be no change in gravity at poles as  = 90°
At all other points as  increases g' will decrease.

12. Both the nucleus and the atom of some element are in their respective first excited states. They get de-

excited by emitting photons of wavelengths N, A respectively. The ratio N is closest to :
A
(1) 10–1 (2) 10–6 (3) 10 (4) 10–10
Ans. (2)
 N Ea
Sol. 
 a EN
where Ea and EN are energies of photons from atom and nucleus respectively. EN is of the order of MeV
and Ea in few eV.
N
So  106
a

13. A plane electromagnetic wave of wavelength  has an intensity I. It is propagating along the positive
Y-direction. The allowed expressions for the electric and magnetic fields are given by :
 2I  2   I  2 ˆ
E cos   y – ct   k; E cos   y – ct   k;
0c     c   
(1) (2) 0
 1  1
B   Eiˆ B   Eiˆ
c c
 2I  2 ˆ  I  2 
E cos   y – ct   k; E cos   y – ct   i;
0c   0c  
(3) (4)
 1  1 
B  Eiˆ B  Ek
c c
Ans. (1)
1
Sol. If E0 is magnitude of electric field then 0E2  C  I
2
2I
E0 
C0
E0
B0 
C
 
direction of E  B will be along + ĵ .

14. A charge q is spread uniformly over an insulated loop of radius r. If it is rotated with an angular velocity
 with respect to normal axis then magnetic moment of the loop is :
3 1 4
(1) qr 2 (2) qr 2 (3) qr2 (4) qr 2
2 2 3
Ans. (2)

www.vedantu.com 4
Sol.

M q

L 2m
q
M=  mr 2 
2m
qr 2
M=
2

15. A heating element has a resistance of 100 at room temperature. When it is connected to a supply of
220V a steady current of 2A passes in it and temperature is 500°C more than room temperature. What
is the temperature coefficient of resistance of the heating element?
(1) 5 × 10–4 °C–1 (2) 2 × 10–4 °C–1 (3) 1 × 10–4 °C–1 (4) 0.5 × 10–4 °C–1
Ans. (2)
220
Sol. Resistance after temperature increases by 500°C = = 110
2
110 = 100 (1 + 500)
10
=
100  500
 = 2 × 10–4 °C–1

16. A coil of cross-sectional area A having n turns is placed in a uniform magnetic field B. When it is rotated
with an angular velocity  the maximum e.m.f. induced in the coil will be :
3 1
(1) nBA (2) 3nBA (3) nBA (4) nBA
2 2
Ans. (3)
Sol.  = BAn sint
max = BAn

17. A ray of light is incident at an angle of 60° on one face of a prism of angle 30°. The emergent ray of
light makes an angle of 30° with incident ray. The angle made by the emergent ray with second face of
prism will be :
(1) 0° (2) 90° (3) 30° (4) 45°
Ans. (2)
Sol. =I+e–A
30 = 60 + e – 30°
 e=0
So angle with face = 90°

30°
30°

60°

www.vedantu.com 5
18. A galvanometer with its coil resistance 25 requires a current of 1mA for its full deflection. In order to
construct an ammeter to read up to a current of 2A the approximate value of the shunt resistance
should be :
(1) 1.25 × 10–2 (2) 2.5 × 10–3 (3) 2.5 × 10–2 (4) 1.25 × 10–3
Ans. (1)
Sol. IgRg = (I – Ig)S
10 –3  25
S ~

2

S~
–3
– 12.5 × 10
or 1.25 × 10–2
Ig
G

I–Ig S
1
19. An oscillator of mass M is at rest in the equilibrium position in a potential V = k(x – X)2. A particle of
2
mass m comes from right with speed u and collides completely inelastically with M and sticks to it. This
process repeats every time the oscillator crosses its equilibrium position. The amplitude of oscillations after 13
collisions is : (M = 10, m = 5, u = 1, k = 1)
2 1 3 1
(1) (2) (3) (4)
3 3 5 2
Ans. (2)
Sol. In first collision mu momentum will be imparted to system. In second collision when momentum of
(M + m) is in opposite direction mu momentum of particle will make its momentum zero.
on 13th collision
m M +12m

M +13m V
mu = (M + 13m)v
mu u
v= 
M  13m 15
v = A
u K 1 75 1
  A  A= 
15 M  13m 15 1 3

20. One mole of an ideal monatomic gas is taken along the path ABCA as shown in the PV diagram. The
maximum temperature attained by the gas along the path BC is given by :
P

B
3P0

P0 C
A

V0 2V0 V
25 P0 V0 5 P0 V0 25 P0 V0 25 P0 V0
(1) (2) (3) (4)
4 R 8 R 8 R 16 R

www.vedantu.com 6
Ans. (3)
Sol. Equation of line BC
2P0
P = P0 – (V – 2V0)
V0
2P0 V 2
P0 V –  4P0 V
V0
Temperature =
1 R
P0  2V 2 
T= 5V – 
R  V0 
dT 4V 5
=0  5– =0  V= V0
dV V0 4
P0  5V0 2 25 2 
T= 5  –  V0 
R 4 V0 16 
25 P0 V0
T=
8 R

21. In a circuit for finding the resistance of a galvanometer by half deflection method a 6V battery and a
high resistance of 11k are used. The figure of merit of the galvanometer produces a deflection of  = 9
divisions when current flows in the circuit. The value of the shunt resistance that can cause the
deflection of /2 is
(1) 550 (2) 220 (3) 55 (4) 110
Ans. (4)
 1
Sol. I= G = K
RG 9
I  S I S
   
2 GS SG 2 R(S  G)  GS
R
GS
I
RG 
S= 2
–
R  G I
2
1
11 103   103  270  10–6
S= 9
6
6– 
2
S = 110
S

G G
I I/2

R R

E
E E

www.vedantu.com 7
22. In the following circuit the switch S is closed at t = 0. The charge on the capacitor C1 as a function of
 C1C2 
time will be given by  Ceq  
 C1  C2 

C1 C2

S ER
(1) C1E[1 – exp(–tR/C1)] (2) CeqE exp(–t/RCeq)
(3) CeqE [1 – exp(–t/RCeq)] (4) C2E[1 – exp(–t/RC2)]
Ans. (3)
– t / Rc eq
Sol. q = Ceq E 1– e 

Both capacitor will have same charge as they are connected in series.
Ceq

E R

        
23.    
Let A  ˆi  ˆj and B  2iˆ – ˆj . The magnitude of a coplanar vector C such that A.C  B.C  A.B is given
by :
9 20 5 10
(1) (2) (3) (4)
12 9 9 9
Ans. (3)

Sol. If C  a ˆi  bjˆ then
   
A.C  A.B
a+ b=1 …(i)
   
B.C  A.B
2a – b = 1 …(ii)
Solving equation (i) and (ii) we get
1 2
a= ,b=
3 3
 1 4 5
C   
9 9 9

24. A particle executes simple harmonic motion and is located at x = a, b and c at times t0, 2t0 and 3t0
respectively. The frequency of the oscillation is :
1 ac 1  a  2b  1 ab 1  2a  3c 
(1) cos–1   (2) cos–1   (3) cos–1   (4) cos–1  
2t 0  2b  2t 0  3c  2t 0  2c  2t 0  b 
Ans. (1)

www.vedantu.com 8
Sol. a = Asint0
b = Asin2t0
c = Asin3t0
a + c = A[sint0 + sin3t0] = 2Asin2t0cost0
ac
= 2cost0
b
1 ac 1 ac
 = cos–1    f= cos–1  
t0  2b  2t 0  2b 

25. A thin circular disk is in the xy plane as shown in the figure. The ratio of its moment of inertia about z
and z' axes will be :
z
z'

o y

x
(1) 1 : 4 (2) 1 : 5 (3) 1 : 3 (4) 1 : 2
Ans. (3)
mR2
Sol. Iz =
2
3 I 1
Iz' = mR2 z' 
2 Iz 3

26. Two identical conducting spheres A and B carry equal charge. They are separated by a distance much
larger than their diameters and the force between them is F. A third identical conducting sphere C is
uncharged. Sphere C is first touched to A then to B and then removed. As a result the force between A
and B would be equal to :
3F F 3F
(1) (2) (3) (4) F
4 2 8
Ans. (3)
Sol.
q q

r
2
kq q
F= 2
when A and C are touched charge on both will be
r 2
Then when B and C are touched
q
q
3q
qB = 2 
2 4
q 3q
k  2
kqA qB 2 4  3 kq  3 F
F' = 2
 2 2
r r 8 r 8

www.vedantu.com 9
27. Two particles of the same mass m are moving in circular orbits because of force given by
–16 3
F(r) = – r . The first particle is at distance r = 1 and the second at r = 4. The best estimate for the ratio
r
of kinetic energies of the first and the second particle is closest to :
–3 2 –2 –1
(1) 3 × 10 (2) 6 × 10 (3) 6 × 10 (4) 10
Ans. (3)
mV 2 16 3
Sol.  r
r r
1
KE0 = mV2
2
1
= [16 + r4]
2
16  1
K1 2 17
 
K2 16  256 272
2
K1
~
– 6  10 –2
K2

28. The percentage errors in quantities P, Q, R and S are 0.5%, 1%, 3% and 1.5% respectively in the
P 3 Q2
measurement of a physical quantity A = . The maximum percentage error in the value of A will be :
RS
(1) 6.5% (2) 7.5% (3) 6.0% (4) 8.5%
Ans. (1)
A 3P 2Q 1 R S
Sol.    
A P Q 2 R S
1
= 3 × 0.5 + 2 × 1 + × 3 + 1.5
2
= 1.5 + 2 + 1.5 + 1.5
A
= 6.5%
A

29. A carrier wave of peak voltage 14V is used for transmitting a message signal given to achieve a
modulation index of 80% will be :
(1) 22.4V (2) 7V (3) 11.2V (4) 28V
Ans. (3)
A
Sol. m= m
Ac
Am = 0.8 × 14
= 11.2V

30. A small soap bubble of radius 4cm is trapped inside another bubble of radius 6cm without any contact.
Let P2 be the pressure inside the inner bubble and P0 the pressure outside the outer bubble. Radius of
another bubble with pressure difference P2 – P0 between its inside and outside would be :
(1) 2.4cm (2) 12cm (3) 4.8cm (4) 6cm
Ans. (1)

www.vedantu.com 10
4T 4T 4T
Sol. P2 = P0 +  P2 = P0 +
6 4 r
1 1 1
    r = 2.4cm
r 6 4

P2
P2
r
4

www.vedantu.com 11
JEE Main - 2018 (CBT)
Exam Test Date: 16/04/2018

Part - B (Chemistry)

1. For standardizing NaOH solution, which of the following is used as a primary standard ?
(1) Sodium tetraborate (2) Ferrous Ammonium Sulfate
(3) Oxalic acid (4) dil. HCl
Ans. (3)
Sol. Oxalic acid is used as a primary standard for NaOH standardizing.

2. Products A and B formed in the following reactions are respectively :


 
NH3CH3COO

C6H5NH2
+ HNO2  A  B

SO3H
O
N=N–O–CCH3 N=N NH2

(1) and

SO3H SO3H
O
H
N=N–OCCH3 N=N–N

(2) and

SO3H SO3H

N=NCOCH3 N=N

(3) and

SO3H SO3H
N=NCOCH3

(4) and HO3S N NH2


H
SO3H

www.vedantu.com 1
Ans. (1)
N O O
  

NH3 CH3COO N O–C–CH3 N=N–O–C–CH3

HNO2
Sol.

SO3H SO3H SO3H


(A)

N=N NH2
H2 N

(B)
SO3H

3. When XO2 is fused with an alkali metal hydroxide in presence of an oxidizing agent such as KNO3 ; a
dark green product is formed which disproportioates in acidic solution to afford a dark purple solution. X
is :
(1) Mn (2) Cr (3) V (4) Ti
Ans. (1)
Acidic
Sol. MnO2 + KOH  K 2MnO4   KMnO4
solution
(dark green) (dark purple)

4. The major product B formed in the following reaction sequence is :


CHO
(i) C H MgBr
HCl

2 5
 A   B
(ii) H2O
MeO

(1) (2)
Cl
Cl MeO
Cl

(3) (4)
MeO MeO
Ans. (4)
O OH Cl
C–H
(i) C H MgBr HCl
Sol. 
2 5
  
(ii) H2O SN1
CH3O CH3O CH3O

www.vedantu.com 2
5. In a complexometric titration of metal ion with ligand
M (Metal ion) + L (Ligand)  C (Complex) end point is estimated spectrophotometrically (through light
absorption). If 'M' and 'C' do not absorb light and only 'L' absorbs, then the titration plot between
absorbed light (A) versus volume of ligand 'L' (V) would look like :

A A A A
(1) (2) (3) (4)

V (Ligand) V (Ligand) V (Ligand) V (Ligand)


Ans. (1)
Sol. Initially ligand consumed by metal due to formation of complex. So absorbed light (A) remain constant,
after complex formation is completed, extra volume of ligand solution increases ligand concentration
and also increases absorbed light.

6. The major product of the following reaction is :


CH=CHCH3

HBr
 

CH2CH2CH2Br BrCHCH2CH3 CH2CHCH3 CH=CHCH3


Br
(1) (2) (3) (4)

Br
Ans. (2)
Br
CH=CH–CH3 CH–CH CH–CH2–CH3
2–CH3


HBr Br
Sol.    

7. Among the following, the incorrect statement is :


(1) Cellulose and amylose has 1,4-glycosidic linkage.
(2) Lactose contains -D-galactose and -D-glucose.
(3) Maltose and lactose has 1,4-glycosidic linkage.
(4) Sucrose and amylose has 1,2-glycosidic linkage.
Ans. (4)
Sol. In amylose 1,4-glycosidic linkage is present.

www.vedantu.com 3
8. In the extraction of copper from its sulphide ore, metal is finally obtained by the oxidation of cuprous
sulphide with :
(1) SO2 (2) Fe2O3 (3) Cu2O (4) CO
Ans. (3)
Sol. Cu2S + 2Cu2O  6Cu + SO2

9. Among the oxides of nitrogen :


N2O3, N2O4 and N2O5 ; the molecule(s) having nitrogen-nitrogen bond is/are :
(1) N2O3 and N2O4 (2) N2O4 and N2O5 (3) N2O3 and N2O5 (4) Only N2O5
Ans. (1)
N 2O 3 N2O4
O O O O
Sol. N—N N—N
O O O

10. Which of the following conversions involves change in both shape and hybridisation ?
+
(1) H2O  H3O (2) BF3  BF4– (3) CH4 C2H6 (4) NH3  NH4

Ans. (2)

Sol. BF3  BF4–

F –
F B
F F
B F
F F
2 3
sp sp
Triangle planar Tetrahedral

11. The most polar compound among the following is :


F F F

(1) (2) (3) (4)


F
F F
Ans. (3)
F

Sol. In , the bond dipole vector of C–F bond is not subtractive.


F

www.vedantu.com 4
12. In Wilkinson's catalyst, the hybridization of central metal ion and its shape are respectively :
3 2 3
(1) sp d, trigonal bipyramidal (2) d sp , octahedral
2 3
(3) dsp , square planar (4) sp , tetrahedral
Ans. (3)
Sol. Wilkinson catalyst
[RhCl(PPh3)3]

13. At 320 K, a gas A2 is 20 % dissociated to A(g). The standard free energy change at 320 K and 1 atm in
–1 –1 –1
J mol is approximately : (R = 8.314 JK mol ; ln 2 = 0.693 ; ln 3 = 1.098)
(1) 1844 (2) 2068 (3) 4281 (4) 4763
Ans. (3)
Sol. A2(g) 2A(g)
1 0
20 20
1 – 1× 2×
100 100
0.8 0.4

(p A )2 0.4  0.4
Kp = = = 0.2
(p A 2 ) 0.8

Gº = –2.303 × 8.314 × 320 log10 0.2 = 4281 J/mole

14. Which of the following complexes will show geometrical isomerism ?


(1) Potassium tris(oxalato)chromate(III)
(2) Pentaaquachlorochromium(III)chloride
(3) Aquachlorobis(ethylenediamine)cobalt(II) chloride
(4) Potassium amminetrichloroplatinate(II)
Ans. (3)
Sol. [Co(H2O)Cl(en)2]Cl

+ H2O +
en
H2O

Co en Co en

Cl
en
Cl

(Geometrical Isomer)

www.vedantu.com 5
15. Which of the following statements is false ?
(1) Splitting of spectral lines in electrical field is called Stark effect.
(2) Frequency of emitted radiation from a black body goes from a lower wavelength of higher
wavelength as the temperature increases.
(3) Photon has momentum as well as wavelength.
(4) Rydberg constant has unit of energy.
Ans. (2) and (4) [both are false]
Sol. When temperature is increased, black body emit high energy radiation, from higher wavelength to lower
wavelength.
–1 –1
Rydberg constant has unit length (i.e. cm )

16. When 9.65 ampere current was passed for 1.0 hour into nitrobenzene in acidic medium, the amount of
p-aminophenol produced is :
(1) 109.0 g (2) 98.1 g (3) 9.81 g (4) 10.9 g
Ans. (3)
OH

Sol. C6H5NO2 

NH2
– +
4e + 4H + C6H5NO2  C6H4 (OH)(NH2 ) + H2O
M.W.  109 g

E  M
(v.f.) = 4 W = ZIt =
F
×I×t E  4 
 
109  9.65  60  60
W=
4  96500
W = 9.81 g

17. For which of the following processes, S is negative ?


(1) C(diamond)  C(graphite) (2) N2(g, 1 atm)  N2(g, 5 atm)
(3) N2(g, 273 K)  N2(g, 300 K) (4) H2(g)  2H(g)
Ans. (2)
Sol. N2 (g, 1 atm)  N2 (g, 5 atm)

 T  V V2 P
S =  nCp ln 2  + nR ln 2 for isothermal process T1 = T2 and = 1
 T1 V1 V1 P2

P1 1
= 0 + nR ln = nR ln
P2 5

S < 0

www.vedantu.com 6
18. An unknown chlorohydrocarbon has 3.55 % of chlorine. If each molecule of the hydrocarbon has one
chlorine atom only ; chlorine atoms present in 1 g of chlorohydrocarbon are :
23 –1
(Atomic wt. of Cl = 35.5 u ; Avogadro constant = 6.023 × 10 mol )
9 23 21 20
(1) 6.023 × 10 (2) 6.023 × 10 (3) 6.023 × 10 (4) 6.023 × 10
Ans. (4)
Sol. CxHyCl
% Cl = 3.55
3.55
Weight of Cl = 1 ×
100
1 3.55
nCl– =
100  35.5

– 1 3.55 23
No of Cl ion = × 6.023 × 10
100  35.5
20
= 6.023 × 10

19. The incorrect statement is :


2+
(1) Cu ion gives chocolate coloured precipitate with potassium ferrocyanide solution.
2+ 2+
(2) Cu and Ni ions give black precipitate with H2S in presence of HCl solution.
(3) Ferric ion gives blood red colour with potassium thiocyanate.
2+
(4) Cu salts give red coloured borax bead test in reducing flame.
Ans. (2)
2–
Sol. Due to common ion effect, sufficient S concentration not produce and not formed ppt of NiS.

–1
20. The mass of a non-volatile, non-electrolyte solute (molar mass = 50 g mol ) needed to be dissolved in
114 g octane to reduce its vapour pressure to 75 %, is :
(1) 37.5 g (2) 75 g (3) 150 g (4) 50 g
Ans. (Bonus)

Po – Ps n
Sol. =
Ps N

W
100P – 75P 50
=
75P 1

25 W
=
75 50
50
W= g
3

www.vedantu.com 7
21. The incorrect geometry is represented by :
(1) NF3 – trigonal planar (2) BF3 – trigonal planar
(3) AsF5 – trigonal bipyramidal (4) H2O – bent
Ans. (1)
Sol. NF3
N 3
F F sp
F Trigonal pyramidal

22. Assuming ideal gas behaviour, the ratio of density of ammonia to that of hydrogen chlroide at same
temperature and pressure is : (Atomic wt. of Cl 35.5 u)
(1) 1.46 (2) 1.64 (3) 0.46 (4) 0.64
Ans. (3)
P(M.w.)
Sol. d=
RT
dNH3 (M.w.)NH3 17
= = = 0.46
dHCl (M.w.)HCl 36.5

23. The correct match between items of List-I and List-II is :


List-I List-II
(A) Phenelzine (P) Pyrimidine
(B) Chloroxylenol (Q) Furan
(C) Uracil (R) Hydrazine
(D) Ranitidine (S) Phenol
(1) (A)-(S), (B)-(R), (C)-(Q), (D)-(P) (2) (A)-(R), (B)-(S), (C)-(P), (D)-(Q)
(3) (A)-(R), (B)-(S), (C)-(Q), (D)-(P) (4) (A)-(S), (B)-(R), (C)-(P), (D)-(Q)
Ans. (2)
Sol.  Phenelzine contains hydrazine
 Chloroxylenol contains phenol
 Uracil is the pyrimidine base
 Ranitidine contains furan ring

24. The gas phase reaction 2NO2(g)  N2O4(g) is an exothermic reaction. The decomposition of N2O4, in
equilibrium mixture of NO2(g) and N2O4(g), can be increased by :
(1) addition of an inert gas at constant pressure.
(2) lowering the temperature
(3) increasing the pressure
(4) addition of an inert gas at constant volume.
Ans. (1)
Sol. 2NO2(g)  N2O4(g) H = (–)
By addition of an inert gas at constant pressure, volume increases, so reaction moving in backward
direction and decomposition of N2O4 increases.

www.vedantu.com 8
25. Which one of the following is not a property of physical adsorption ?
(1) Higher the pressure, more the adsorption
(2) Greater the surface area, more the adsorption
(3) Lower the temperature, more the adsorption
(4) Unilayer adsorption occurs
Ans. (4)
Sol. Physical adsorption is multilayer adsorption.

26. A group 13 element 'X' reacts with chlorine gas to produce a compound XCl3. XCl3 is electron deficient
and easily reacts with NH3 to form Cl3X  NH3 adduct; however, XCl3 does not dimerize. X is :
(1) B (2) Al (3) In (4) Ga
Ans. (1)
Sol. BCl3
Cl H
NH3
B + Cl2 BCl3 B N
[not dimerise due to Cl Cl H H
(p-p) back bonding]

27. The major product of the following reaction is :


Br

KOH
 
SN 2
NH2

OH OH OH OH

(1) (2) (3) (4)


NH2 NH2 H2N H2N

Ans. (3)
Br OH OH

KOH
Sol.   or
SN 2
NH2 NH2 H2N

Inversion takes place at the carbon containing bromine atom.

www.vedantu.com 9
28. If 50 % of a reaction occurs in 100 second and 75 % of the reaction occurs in 200 second, the order of
this reaction is :
(1) 2 (2) 3 (3) Zero (4) 1
Ans. (4)
Co
100 second

75 % Co
Sol. complete 2
200 second

Co
4
First order reaction as half life is constant.

29. The major product of the following reaction is :


(i) OHCCH2COCl
 
(ii) H2SO4 , heat
MeO OH
OMe O

(1) (2)
O MeO O O
O OMe

(3) (4)
MeO O O O
Ans. (2)
O O O O
H H
H–C–CH2–C–Cl
Sol. C=O –H C=O
CH3O OH CH3O O CH3O O
H H Electrophilic
substitution

OH

H, 
–H2O
CH3O O O Dehydration CH3O O O

30. Which of the following compounds will most readily be dehydrated to give alkene under acidic
condition?
(1) 4-Hydroxypentan-2-one (2) 3-Hydroxypentan-2-one
(3) 1-Pentanol (4) 2-Hydroxycyclopentanone
Ans. (1)
OH O
Sol. will most readily be dehydrated to give conjugated alkene.

www.vedantu.com 10
JEE Main - 2018 (CBT)
ExamTest Date : 16/04/2018

Part - C (Mathematics)

1 1 dy
1. If x = 2cos ec t
and y = 2sec t
(|t|  1), then is equal to :
dx
y y x x
(1) (2*)  (3)  (4)
x x y y
Ans. (2)
1 1  1 
2s ec t  n2  
1  2
dy dy / dt 2 2sec t
 t t 1 
Sol. = =
dx dx / dt 1 1  1 
2cos ec t  n2 
1  2 
2 2cos ec t
 t t 1 
1
2sec t
y
=– =
2cos ec
1
t x

2. Let N denote the set of all natural numbers. Define two binary relations on N as
R1 = {(x, y)  N × N : 2x + y = 10} and R2 = {(x, y)  N × N : x + 2y = 10}. Then
(1) Both R1 and R2 are transitive relations
(2) Range of R2 is {1, 2, 3, 4}.
(3) Range of R1 is {2, 4, 8}
(4) Both R1 and R2 are symmetric relations.
Ans. (2)
Sol. R1 = {(1, 8), (2, 6), (3, 4), (9, 2)}
R2 = {(8, 1), (6, 2), (4, 3), (2, 4)}
Range of R2 = {1, 2, 3, 4}

3. The coefficient of x2 in the expansion of the product (2– x2)· ((1 + 2x + 3x2)6 + (1 – 4x2)6) is :
(1) 107 (2) 108 (3) 155 (4*) 106
Ans. (4)
2 2 2 6 2 6
Sol. coefficient of x = 2 coefficient of x in ((1 + 2x + 3x ) + (1 – 4x ) ) – constant term
6
(1 + 2x + 3x2)6 =  6
c r (2x  3x 2 )r
r 0
6 6 2 6 2 2
= C0 + C1 (2x + 3x ) + C2 (2x + 3x ) + ….
2
coefficient of x = 2(18 + 60 – 24) – 2
= 108 – 2 = 106

www.vedantu.com 1
1
4. If the area of the region bounded by the curves, y = x2, y = and the lines y = 0 and x = t (t > 1) is 1 sq.
x
unit, then t is equal to :
2 3
3 4
(1*) e 3 (2) e 2 (3) (4)
2 3
Ans. (1)
1 1
Sol.  x2 dx    dx = 1
x x

2
1
+ nt = 1  t = e3
3
5. If the length of the latus rectum of an ellipse is 4 units and the distance between a focus and its nearest
3
vertex on the major axis is units, then its eccentricity is :
2
2 1 1 1
(1) (2) (3) (4)
3 2 9 3
Ans. (4)
2b2
Sol. = 4 b2 = 2a
a
3
b2 = a2 (1 – e2) , a (1 – e) =
2
2 = a(1 – e) (1 + e)
3 1
2 = (1 + e) e=
2 3
6. The number of numbers between 2,000 and 5,000 that can be formed with the digits 0, 1, 2, 3, 4
(repetition of digits is not allowed) and are multiple of 3 is :
(1) 36 (2) 30 (3) 24 (4) 48
Ans. (2)
Sol. number can be formed y (0, 1, 2, 3) or (0, 2, 3, 4)
number of 4 digits number = 2 × 3 ! + 3 × 3 ! = 30
7. Two different families A and B are blessed with equal number of children. There are 3 tickests to be
distributed amongst the children of these families so that no child gets more than one ticket. If the
1
probability that all the tickets go to the children of the family B is , then the number of children in each
12
family is :
(1) 6 (2) 5 (3) 3 (4) 4
Ans. (2)
Sol. Let n number of children are there in each family
n
1 C .3!
= 2n 3
12 C3 .3!
n
C3 1
2n
= n=5
C3 12

www.vedantu.com 2
1
(27  x) 3  3
8. lim 2
equals :
x 0
3
9  (27  x)
1 1 1 1
(1)  (2) (3) (4) 
6 6 3 3
Ans. (1)
1
 
 x  3
3  1   1 
 27  
Sol. lim  
2
x 0  
 x 3 
9 1   1  
  27  
 
 x 
1  81  1
lim
x 0 3
 2 x  =
 .  6
 3 27 
1 1 1
9. Let p, q and r be real numbers (p  q, r  0), such that the roots of the equation + = are
x p x q r
equal in magnitude but opposite in sign, then the sum of squares of these roots is equal to :
p2  q2
(1) p2 + q2 (2) (3) 2(p2 + q2) (4) p2 + q2 + r2
2
Ans. (1)
Sol. (2x + p + q) r = (x + p) (x + q)
x2 + (p + q – 2r) x + pq – pr – qr = 0
p + q = 2r ……..(i)
2 + 2 = ( + )2 – 2
= 0 – 2 [pq – pr – qr] = – 2pq + 2r (p + q) = – 2pq + (p + q)2 = p2 + q2

1 1 1
10. Let , ,…, (xi 0 for i = 1, 2, …., n) be in A.P. such that x1 = 4 and x21 = 20. If n is the least positive
x1 x 2 xn
n  1
integer for which xn > 50, then  x  is equal to :
i1  i 

1 13 13
(1) 3 (2) (3) (4)
8 4 8
Ans. (3)
1 1
Sol. + 20.d =
4 20
1
d=
100
1 1
<
xn 50
1 n 1 1
– <  n > 24
4 100 50
n = 25
25 
1  25  1 1  13
  =
x
i1  i  2  2  4  100  24  = 4
 

www.vedantu.com 3
11. The differential equation representing the family of ellipses having foci either on the x-axis or on the
y-axis, centre at the origin and passing through the point (0, 3) is :
(1*) xy y– y2 + 9 = 0 (2) xy y + x (y)2 – y y = 0
2
(3) xy y + y – 9 = 0 (4) x + y y = 0
Ans. (1)
Sol. Equation of ellipse
x2 y2
+ =1
a2 b2
x2 y2 x y dy
Passes (0, 3)  2
+ =1 ………(1) 2
=–
a 9 a 9 dx
2x 2y dy 1 y 1
+ =0 ………(2) = y
a2 9 dx a2 9x
By (1) & (2) D. equation is
xy 1 y2
– y + =1
9 9
 xy y1 – y2 + 9 = 0

12. The sum of the intercepts on the coordinate axes of the plane passing trhough the point (–2, –2, 2) and
containing the line joining the points (1, –1, 2) and (1, 1, 1), is :
(1) 4 (2) 12 (3) –8 (4) –4
Ans. (4)
Sol. Equation plane
x2 y2 z2
3 1 0 =0
3 3 1
 – (x + 2) + 3 (y + 2) + 6 (z – 2) = 0
 x – 3y – 6z + 8 = 0
8 8
sum of intercepts = – 8 + + = – 4
3 6

1 0 0 
13. Let A = 1 1 0  and B = A20. Then the sum of the elements of the first column of B is :
1 1 1
(1) 210 (2) 211 (3) 251 (4) 231
Ans. (4)
1 0 0 
Sol. A = 1 1 0 
1 1 1
 1 0 0 1 0 0
A2 =  2 1 0  ; A3 = 3 1 0

3 2 1  6 3 1
 1 0 0  1 0 0
A =  4 1 0  ….. A20 =
4 
 20 1 0

10 4 1  210 20 1
Sum of the elements of first column = 231

www.vedantu.com 4
14. Let A, B and C be three events, which are pair-wise independent and E denotes the complement of an
event E. If P(A BC) = 0 and P(C) > 0, then P [( A  B )| C] is equal to :
(1) P( A ) – P(B) (2) P( A ) – P( B ) (3) P( A ) + P( B ) (4) P(A) + P( B )
Ans. (1)
P[(A  B)  C]
Sol. P[( A  B )|C] =
P(C)
P(C) – P(A  C) – P(B  C)  P(A  B  C)
=
P(C)
P(C) – P(A)P(C)  P(B)P(C)
=
P(C)
= 1 –P (A) – P(B)
 
= P A – P(B) or P B – P(A) 
15. If p  (p q) is false, then the truth values of p and q are respectively :
(1) F, F (2) T, T (3) F, T (4) T, F
Ans. (2)
Sol. P  (~p  ~q)
p q ~ p ~ q p  (~ p ~ q)
T T F F
T F T T
F T T T
F F T T

1 k 1
16. If the function f defined as f(x) = – 2x , x  0, is continuous at x = 0, then the ordered pair (k, f(0)) is
x e 1
equal to :
1 
(1) (2, 1) (2) (3, 1) (3) (3, 2) (4)  ,2 
3 
Ans. (2)
1 k –1
Sol. f(x) = – 2x ;x0
x e –1
f(x) is continuous at x = 0
1 k –1
 f(0) = Lim – 2x
x 0 x e –1
1
(1  (2x)  (2x) 2  .........(–1– x(k – 1)
= Lim 2!
x 0  e2x – 1 
2x 2  
 2x 
Clearly k = 3 and f(0) = 1

www.vedantu.com 5
x y z 5  x 7y  14 z  3 2
17. If the angle between the lines, = = and = = is cos–1  3  , then p is equal to :
2 2 1 2 p 4  
2 7 4 7
(1) (2) (3)  (4) 
7 2 7 4
Ans. (2)
x y z x–5 y–2 z–3
Sol. = = and = =
2 2 1 2 P/7 4
 2P 
 4 4 
2
  7
Angle between both lines is cos–1   = cos–1  
3  P 2 
 3. 4   16 
 49 
2 56  2P 7
 =  P2  980 = P + 28  P2 + 980 = P2 + 56P + 784  56P = 196  P =
3 2
3 P  980 2

18. The locus of the point of intersection of the lines, 2 x – y + 4 2 k = 0 and 2 kx + ky – 4 2 = 0


(k is any non-zero real parameter), is :
1
(1) an ellipse whose eccentricity is .
3
(2) a hyperbola whose eccentricity is 3
(3) a hyperbola with length of its transverse axis 8 2
(4) an ellipse with length of its major axis 8 2 .
Ans. (3)
Sol. 2 x–y+4 2 k=0 ..............(i)
2 kx + ky – 4 2 = 0 .............(ii)
Eliminating k by (i) and (ii)
 2x – y 
 
2x  y 
 –4 2 
=4 2
 
2x2 – y2 = –32
y2 x2
– =1 Hyperbola
32 16
16 3
e = 1 = and length of transverse axis = 8 2
32 2

19. A man on the top of a vertical tower observes a car moving at a uniform speed towards the tower on a
horizonatal road. If it takes 18 min. for the angle of depression of the car to change from 30 to 45° ;
then after this, the time taken (in min.) by the car to reach the foot of the tower, is :
9
(1) ( 3 – 1) (2) 18(1 + 3 ) (3) 18 ( 3 – 1) (4) 9 (1 + 3 )
2
Ans. (4)
Sol. Let length of tower = h B

 AC' = AB = h
and AC = AB cot 30º = 3 h  CC' = ( 3 –1) h
Time taken by car form C to C' = 18 min
18 45º 30º
 time take by car to reach the foot of the tower = min. A 1
C
3 –1 C
= 9 ( 3 + 1) min

www.vedantu.com 6
20. If an angle A of a ABC satisfies 5 cosA + 3 = 0, then the roots of the quadratic equaiton,
9x2 + 27x + 20 = 0 are :
(1) sec A, cotA (2) secA, tanA (3) tanA, cosA (4) sinA, secA
Ans. (2)
3
Sol. 5cosA + 3 = 0  cosA = – clearly A  (90º, 180º)
5
2 5 4
Now roots of equation 9x + 27x + 20 = 0 are – and –
3 3
 Roots secA and tanA

21. If a circle C, whose radius is 3, touches externally the circle, x 2 + y2 + 2x – 4y – 4 = 0 at the point (2, 2),
then the length of the intercept cut by this circle C, on the x-axis is equal to :
(1) 2 3 (2) 5 (3) 3 2 (4) 2 5
Ans. (4)
Sol. Centre of given circle = (–1, 2)
and radius = 1  4  4 = 3
centre of required circle (5,2)
or (–4,2)
length of intercept on x-axis will be square in both circle
so one required circle (x – 5)2 + (y –2)2 = 32
x2 + y2 – 10x – 4y + 20 = 0
Length of x intercept = 2 g2 – c
= 2 25 – 20 = 2 5

22. Let P be a point on the parabola, x2 = 4y. If the distance of P from the centre of the circle,
x2 + y2 + 6x + 8 = 0 is minimum, then the equation of the tangent to the parabola at P, is :
(1) x + y + 1 = 0 (2) x + 4y – 2 = 0 (3) x + 2y = 0 (4) x – y + 3 = 0
Ans. 1
Sol. Let P (2t, t2)
equation normal at P to x2 = 4y be
2 1
y –t =– (x – 2t)
t
it passes through (–3,0)
1
0 – t2 = – (–3 – 2t)
t
t3 + 2t + 3 = 0
2
(t + 1) (t – t + 3) = 0
t=–1
Point P is (–2,1)
2
equation of tangent to x = 4y at (–2,1)
x(–2) = 2 (y + 1)
x+y+ 1=0

www.vedantu.com 7
x
23. If f(x) =  t(sin x  sin t) dt then :
0
(1) f (x) – f(x) = cosx – 2x sinx (2) f(x) + f(x) – f(x) = cosx
(3) f(x) + f(x) = sinx (4) f(x) + f(x) = cosx – 2x sinx
Ans. 4
x
Sol. f(x) =  t(sin x – sin t)dt
0
x x
f(x) = sinx  t dt –  t sin tdt
0 0
x
f'(x) = (sinx) x + cosx  t dt – x sinx
0
x
f'(x)= cosx  tdt
0
x
f''(x) = (cosx) x – (sinx)  tdt
0
x
f'''(x) = x (–sinx) + cosx – (sinx)x – (cosx)  tdt
0
f'''(x) + f'(x) = cosx – 2x sinx

24. The number of values of k for which the system of linear equations,
(k + 2)x + 10y = k
kx + (k + 3) y = k – 1 has no soution, is :
(1) 1 (2) 2 (3) 3 (4) 4
Ans. (1)
Sol. For no solution
k2 10 k
= 
k k3 k –1
(k + 2) (k + 3) = 10 k
k2 – 5k + 6 = 0  k = 2,3
k  2 for k = 2 both lines identical
so k = 3 only
so number of values of k is 1

tan x K  K tan x  1 
25. If  1  tan x  tan dx = x –
2
tan–1   + C, (C is a constant of integration), then the ordered
x A  A 
pair (K, A) is equal to
(1) (2, 1) (2) (2, 3) (3) (–2, 1) (4) (–2, 3)
Ans. (2)
tan
Sol. I= 
1  tan x  tan 2 x
dx


1  sec 2 x 
dx
  1  tan x  tan x 

2

dt
=x–  , where tanx = t  sec2 x dx = dt
1  t  t2
  
 t   
dt 1 1 2   C = x – 2 tan1 2 tan x  1   C
= 
 1
2
3
= x–
3 /2
tan
 3 /2 3 
3 

t      
2 4  

= k =2 , A = 3.

www.vedantu.com 8
n
 1 i 3 
26. The least positive integer n for which  = 1, is
 1  i 3 
 
(1) 2 (2) 5 (3) 6 (4) 3
Ans. (4)
n
 1 i 3 
Sol.   1
 1 i 3 
 
n
 22 
   1
 2 
n = 1
least positive integer value of n is 3.

3 7 15 31
27. The sum of the first 20 terms of the series 1 + + + + + …., is :
2 4 8 16
1 1 1 1
(1) 39 + (2) 38 + (3) 38 + 19 (4) 39 + 20
219 220 2 2
Ans. (3)
3 7 15 31
Sol. 1     ......
2 4 8 16
 1  1  1  1 
= (2–1) +  2     2     2     2    ……….upto 20 terms
 2   4   8   16 
 1 1 1 
= 40 – 1     .....up to 20 terms 
 2 4 8 
  1  20 
 1   
 2  1 1
= 40      = 40 – 2 + 19 = 38 + 19
 1 1  2 2
 2 
 
        
28. Let a = î + ĵ + k̂ , c = ĵ – k̂ and a vector b be such that a × b = c and a · b = 3. Then | b | equals :
11 11 11 11
(1) (2) (3) (4)
3 3 3 3
Ans. (3)
  
Sol. a b  c
    
 
a  a b  ac
      
 
a . b a  a . a b  a  c


3a  3b  2 î  ĵ  k̂
3i + 3j + 3k̂ – 3 b̂ = – 2 î + ĵ + k̂
 1

b  5 î  2 ĵ  2k̂
3

 25  4  4
|b | =
3
 11
|b | =
3

www.vedantu.com 9

You might also like